You are on page 1of 148

yoursmahboob.wordpress.

com

CONTENT
Topics Page No.
Seating Arrangements 2
Puzzles 12
Coding Decoding 22
Input-Output 32
Alphanumeric 44
Direction-Ranking 54
Equality-Inequality 71
Syllogism 84
Blood Relation 111
Critical Reasoning 122
www.ibpsguide.com | www.ibpsguide.in | mock.ibpsguide.in | www.sscexamguide.com
Copyright © 2016 IBPS Guide 1
yoursmahboob.wordpress.com

1. SEATING ARRANGEMENTS
Q (1 – 5) L, M, N, O, P, Q, R and S are sitting around a square table in such a way that four of them sit at four
corners of the square while four sit in the middle of each of the four sides. The ones who sit at the four corners
face the centre while those who sit in the middle of the sides face outside. Two females sit in the middle of the
sides and two at the corners. L sits second to the left of R. R sits in the middle of one of the sides. N sits fourth
to the right of his wife and his wife is not an immediate neighbour of L or R. M sits third to right of her
husband. M does not sit at any of the corners. Only Osits between M and S. S is the husband of L. P is a male.

Q 1) Which of the following is true with respect to the given seating arrangement?
a) Q is a male and sits diagonally opposite to P
b) L sits in the centre of one of the sides of the square table
c) R and S do not face each other in the seating arrangement
d) No two males are immediate neighbor of each other
e) P and O are immediate neighbours of each other

Q 2) What is the position of P with respect to N?


a) Immediately to the left
b) Second to the left
c) Third to the right
d) Immediately to the right
e) Second to the right

Q 3) How many people sit between M and N when counted in anti – clockwise direction from M?
a) Four b) Three c) Two d) None e) One

Q 4) Who amongst the following is the wife of N?


a) O b) Q c) M d) R e) Cannot be determined

Q 5) Who among the following is M‘s husband?


a) N b) R c) P d) Q e) Cannot be determined

Q (6 – 10) There are eight persons – Rajeev, Mahendra, Narendra, Orijeet, Pranjal, Ramesh, Ankur, Somesh –
sitting around a circular area at equal distance from one another, but not necessarily in the same order. Some of
them are facing the center while some others are facing outside(ie in a direction just opposite to the center).
Somesh sitting fourth to the left of Pranjal, Orijeet is sitting second to the right of Somesh. Somesh faces
outside. Rajeev is sitting third to the right of Ankur. Ankur is not an immediate neighbour of Pranjal. Both the
immediate neighbours of Ramesh face toward the center. Both the immediate neighbours of Mahendra face
opposite directions(ie if one person faces towards the center, the other person faces outside). Both the
www.ibpsguide.com | www.ibpsguide.in | mock.ibpsguide.in | www.sscexamguide.com
Copyright © 2016 IBPS Guide 2
yoursmahboob.wordpress.com

immediate neighbours both the immediate neighbours of Somesh face the same directions as that of Orijeet.
Both the immediate neighbours of Pranjal face just the opposite direction of Pranjal(ie if Pranjal faces towards
the center, bot the immediate neighbour of Pranjal faces outside and vice versa). Narendra is immediate
neighbour of Ankur. Rajeev is facing inside and Narendra is facing the same direction as that of Orijeet.

Q 6) Which of the following statements is/are true regarding Narendra as per the given seating arrangement?
a) Narendra is sitting to the immediate right of Mahendra
b) Narendra faces outside
c) Rajeev is sitting second to the right of narendra
d) Somesh is one of the immediate neighbours of Nahendra

Q 7) Who among the following sitting to the immediate left of Pranjal?


a) Ankur b) Mahendra c) Orijeet d) None of these

Q 8) How many persons are facing outside?


a) One b) Three c) Five d) Four

Q 9) What is the position of Ankur with respect to the Orijeet in the given sitting arrangement?
a) Fourth to the left b) Third to the right
c) Fifth to the left d) Second to the right

Q 10) Four of the following three are alike in a certain way on the basis of given seating arrangement, and
hence form a group. Which one of the following does not belong to that group?
a) Narendra b) Orijeet c) Ankur d) Pranjal

Directions (Q. 11-14): Study the following information carefully and answer the given questions.
K, L, M, P, Q, R, S and T are sitting around a square table in such a way that four of them sit at four corners of
the square while four sit in the middle of each of the four sides. The one who sit at the four corners faces outside
while those who sit in the middle of the sides face the centre of the table. P sits third to the right of S. S faces
the centre. Q sits third to the left of M who does not sit in the middle of the sides. Only one person sits between
Q and R. R is not an immediate neighbor of M. T faces the centre. K is not an immediate neighbour of R.
Q 11). What is the position of M with respect to L?
a) Third to the right b) M and L sit diagonally opposite to each other
c) Second to the right d) Second to the left
e) Fifth to the right

Q 12). Who sits exactly between Q and R?


a) T b) P c) K d) M e) S and K

www.ibpsguide.com | www.ibpsguide.in | mock.ibpsguide.in | www.sscexamguide.com


Copyright © 2016 IBPS Guide 3
yoursmahboob.wordpress.com

Q 13). Which of the following pairs represents the persons seated in the centre of the side. Who face each
other?
a) S, Q b) K, L c) M, P d) R, T e) T, Q

Q 14). Who amongst the following sit between R and K when counted in anti-clockwise direction from K?
a) No one sits between R and K as R and K are immediate neighbor of each other
b) S, P and L c) P and Q d) L and R e) M, S and T

Q 15 – 20) Nine people L, M, N, O, P, Q, R, S and T stay in a building not necessarily in the same order. The
building has nine floors and only one person stays on one floor. All of them own one each, and each car is of a
different car, i.e. grey, white, black, yellow, green, red, orange and pinknot necessarily in the same order. The
ground floor is numbered 1, the floor above it, number 2 and so on and the topmost floor is numbered 9. S owns
a black coloured car and stays on even numbered floor. L stays on even numbered floor below the floor on
which S stays. The one who own an orange coloured car stays on 4rth floor. P stays on the second floor and
owns the white coloured car. The one who owns a pink coloured car stays on the third floor. L does not own a
green coloured car. There are two floors between the floors on which the people owning the black and red
coloured cars stay. N owns a grey coloured car. There are three floors between the floors on which N and R
stays. O stays on a floor immediately above T‘s floor. There is one floor between the floors on which Q and R
stay. Q does not own the pink coloured car. The one who owns the blue car stays on the top–most floor. Q does
not stay on the ground floor.

Q 15) How many floors are there between the floor on which Q stays and the floor on which N stays?
a) Six b) Eight c) One d) None e) More than three

Q 16) Which of the following is true as per the given information?


a) M stays on a floor immediately below the floor on which S stays
b) R stays on a floor immediately above the floor on which L stays
c) Q stays on eight floor d) O owns the orange coloured car and stays on 4rth floor
e) None is true

Q 17) On which of the following floor does M stay?


a) 7th b) 6th c) 9th d) 1st e) None of these

Q 18) Who amongst the following stays on the top–most floor?


a) M b) O c) R d) S e) None of these

Q 19) L owns a car of which of the following colours?


a) Pink b) Blue c) Yellow d) Orange e) None of these

www.ibpsguide.com | www.ibpsguide.in | mock.ibpsguide.in | www.sscexamguide.com


Copyright © 2016 IBPS Guide 4
yoursmahboob.wordpress.com

Q 20) How many floors are there between the floor on which S stays and the floor on which red coloured car
stays?
a) 3 b) 2 c) 1 d) 4 e) More than 4

Directions (Q. 21 – 25): Read the following information carefully and answer the questions given below:
Eight students A, B, C, D, E, F, G and H in a school sports competition participated in a game in which they
were sitting around a circular ring. The seats of the ring are not directed towards the centre. All the eight
students are in four groups I, II, III and IV, (ie) two students in each group, but not necessarily in the same
order. These students are from different sport houses, viz Maharishi, Vyas, Aryabhatt, Vashistha,
Shakaracharya, Balmiki, Dhuruv and Dayanand.
• No two students of the same group are sitting adjacent to each other except those of group III. Students
from group IV are sitting opposite each other.
• D is neither in Dayanand nor in Aryabhatt house.
• The student from Dhurv house is sitting on the immediate right of the student from Dayanand house.
• C, who is Vashista house, is in group I. She is sitting in the immediate right of F, who is in group III.
• F is not in Aryabhatt house and she has also participated in other sports
• B from Vyas house is neither on group IV nor in group I or II.
• B is sitting opposite E. Only Balmiki participant A is sitting between Dhurv participant E and the
Shankarachary participant.
• Both the students of group II are sitting adjacent to students of group IV.
Q 21).Who is in Maharishi house?
a) G b) F c) H d) Can‘t say e) None of these

Q 22). ‘H‘ is in which house?


a) Dayanand b) Shankaracharya c) Maharishi d) Can‘t say e) None of these

Q 23).Who among the following students are in group IV?


a) A and B b) A and C c) G and H d) Data inadequate e) None of these

Q 24).The students from which house is sitting opposite the Vashistha house participant?
a) Dayanand b) Shankaracharya c) Dhruv d) Can‘t say e) None of these

Q 25).Which of the following statements is false?


a) The Balmiki representative is sitting opposite the Maharishi house representative
b) The Vyas house representative is sitting opposite the Dhruv house representative
c) H and G are sitting opposite each other
d) The two girl – participants are sitting opposite each other
e) None of these

www.ibpsguide.com | www.ibpsguide.in | mock.ibpsguide.in | www.sscexamguide.com


Copyright © 2016 IBPS Guide 5
yoursmahboob.wordpress.com

Q 26 – 32) There are ten friends A, B, C, D, E, F, G, H, I and J sitting around a rectangular table in such a way
that four of them sit at the corners, two each on the longer sides and one each on the smaller sides, but not
necessarily in the same order. Some of them are facing the centre while the rest are not facing the centre. Not
more than two friends sitting together face the same direction. A sit in the smaller sides and third to the right of
F. I is not an immediate neighbor of A, B or F but sits on the immediate right of C, who is not facing the centre.
Only two among four sitting on the corners face outward. H and D are sitting on the immediate left and third to
the left of J respectively. One of the four friends sitting on the corner is I .E sits on the immediate left of D and
is not an immediate neighbour of D. A and E face the same direction. D and G sit diagonally and face opposite
directions. Five of them face the same directions.
Q 26) Who among the following sits third to the right of E?
a) G b) A c) J d) H e) None of these

Q 27) What is the position of D with respect to A?


a) Second to the right b) Fourth to the right
c) Immediate right d) Can‘t be determined
e) None of these

Q 28) Which of the following groups sit on the corners of the rectangular table?
a) DIHJ b) HDGC c) HDGI d) JDIG e) None of these

Q 29) Four of the following five are alike in a certain way and hence form a group. Which is the one that does
not belong to that group?
a) FC b) AE c) IE d) HB e) DJ

Q 30) How many persons are not facing the centre?


a) Four b) Six c) Five d) Three e) None of these

Q 31) Who among the following sits opposite to H?


a) I b) B c) D d) E e) None of these

Q 32) E is related to C in same way as I is related B. following the same pattern H is related to which of the
following?
a) J b) F c) D d) None of these e) A

Q 33 – 37) Six picture cards P, Q, R, S, T and U are printed in six different coloured inks – Violet, majenta,
orange, black, yellow and brown and are arranged from left to right (not necessarily in the same order and the
colour as given). The pictures were of king, princess, queen, palace, joker and prince. The picture of palace was
in violet colour, but it was nor printed on card S. Card P which was bearing queen‘s picture printed in brown
ink, was at extreme right. The picture of princess was neither on card S nor card T and was not printed in either

www.ibpsguide.com | www.ibpsguide.in | mock.ibpsguide.in | www.sscexamguide.com


Copyright © 2016 IBPS Guide 6
yoursmahboob.wordpress.com

orange or yellow ink. Card R had picture of king printed in black ink and it was 5th from the right and next to
card Q having the picture of prince.

Q 33) If the princess‘s card is between the cards of the palace and prince, then at what number the joker‘s card
is placed from the left?
a) 1st b) 4th c) 5th d) 2nd

Q 34) Which of the following combination of card and colour is true for picture of princess?
a) T – yellow b) U – majenta c) Q – orange d) Data inadequate

Q 35) In which colour was the picture of joker printed?


a) Data inadequate b) Yellow c) Majenta d) Orange

Q 36) Picture of palace was printed in which of the following cards?


a) T b) U c) S d) Either S or T

Q 37) What was the colour of the card T?


a) Violet b) Majenta c) Orange d) None of these

Direction (38 – 44): L, N, P, R, M, Q, T and Y are the members of a committee sitting around a circular table
but not facing the centre. Each member has a different zodiac sign, viz Leo, Virgo, Libra, Cancer, Aries,
Gemini, Pisces and Scorpio, but not necessarily in the same order. T is third to the right of P. The one whose
sun-sign Leo is second to the left of the one whose sun-sign is Libra.
Y's sun-sign is Libra and is sitting exactly between P and L. The one whose sun-sign is Pisces sits second to the
right of N. The one whose sun-sign is Aries is second to the right of the person whose sun-sign is Gemini. P sits
third to the left of the person whose sun-sign is Virgo. Neither Q nor L is the immediate neighbour of N. Q is
fourth to the left of L. N's sun-sign is neither Cancer nor Ariel. The person whose sun-sign is Leo is sitting
second to the right of the person whose sign is Cancer. R‘s sun-sign is Leo and is not an immediate neighbor of
N.
Q 38. Which of the following is N‘s sun-sign?
a) Pisces b) Scorpio c) Gemini d) Can't be determined e) None of these

Q 39. Who sits third to the right of L?


a) P b) Y c) R d) Q e) None of these

Q 40. What is Y's position with respect to Q?


a) Third to the left b) Fourth to the left c) Second to the right d) Third to the right
e) 2nd to the left

www.ibpsguide.com | www.ibpsguide.in | mock.ibpsguide.in | www.sscexamguide.com


Copyright © 2016 IBPS Guide 7
yoursmahboob.wordpress.com

Q 41. How many persons are there between P and N?


a) None b) Two c) Three d) Four e) None of these

Q 42. What is the sun-sign of P?


a) Gemini b) Libra c) Leo d) Can't be determined e) None of these

Q 43. Which of the following combinations is true?


a) M-Gemini b) P-Cancer c) N-Scorpio d) None is true e)All are true

Q 44. How many persons sits between T and R?


a) one b) two c) three d) four e) None of these

Q 45 –50) Twelve people are sitting in two parallel rows containing six people each, in such a way that there is
an equal distance between adjacent persons. in row 1 – J, K, L, M, N and O are seated, all of them facing South.
In row 2 – U, V, W, X, Y and Z are seated, all of them facing North. Therefore, in a given seating arrangement
each member seated in a row faces another member of the other row. U sits third to the right of X. Neither U
nor X sits at the extreme ends.N faces X. O does not face U and O does not sit at any of the extreme ends. O is
not an immediate neighbor of N. V sits at one of the extreme end. Only two people sit between V and Y. Y does
not face O. Two persons sit between L and K. L is not an immediate neighbour of N. W does not face O. J is not
an immediate neighbour of L.
Q 45) Who amongst the following sit at extreme ends of the rows?
a) V, Y b) M, N c) J, L d) V, Z

Q 46) Who amongst the following faces U?


a) L b) N c) J d) M

Q 47) How many persons are seated between N and M?


a) One b) Two c) Three d) Four

Q 48) J is related to O in the same way as W is related to Z. To which of the following is Y related to, following
the same pattern?
a) V b) X c) W d) U

Q 49) Which of the following is true regarding Z?


a) Z sits second to the right of W b) Z is not an immediate neighbour of U
c) Z sits second to the right of X d) Z sits at one of the extreme ends of the line

Q 50) Among the following given options all of them belongs to the same particular group except:
a) J b) L c) W d) Z

www.ibpsguide.com | www.ibpsguide.in | mock.ibpsguide.in | www.sscexamguide.com


Copyright © 2016 IBPS Guide 8
yoursmahboob.wordpress.com

SEATING ARRANGEMENT (Answers with Detailed Explanation)

Q 1 – 5) The diagram drawn below shows the arrangement of those 8 people.

Q 1) b) Q 2) e) Q 3) c) Q 4) a) Q 5) c)

Q 6 – 10) Based on the given information, following circular arrangement is drawn:

Q 6) d) Q 7) b) Q 8) d) Q 9) a) Q 10) c)

Q 11– 14)

www.ibpsguide.com | www.ibpsguide.in | mock.ibpsguide.in | www.sscexamguide.com


Copyright © 2016 IBPS Guide 9
yoursmahboob.wordpress.com

11)d 12)b 13)e 14)c

Q 15 – 20) The tabulated is given below to show the arrangements – :

Floor Colour of the person Person


9 Blue M
8 Black S
7 Green Q
6 Yellow L
5 Red R
4 Orange O
3 Pink T
2 White P
1 Grey N
Q 15) e) Q 16) d) Q 17) c) Q 18) a)
Q 19) c) Q 20) b)

Q 21 – 25)

21)b 22)d 23)c 24)b 25)c


Q 26 – 32) Givenbelow is the figure showing the positions of ten friends around a rectangular table

Q 26) e) Q 27) b) Q 28) a) Q 29) d) Q 30) c) Q 31) b) Q 32) c)

www.ibpsguide.com | www.ibpsguide.in | mock.ibpsguide.in | www.sscexamguide.com


Copyright © 2016 IBPS Guide 10
yoursmahboob.wordpress.com

Q 33 – 37) Based on the information given in the question we get the following table:

Card Colour Picture Position


P Brown Queen 6
Q Orange/Yellow Prince
Q 33) a) The vacant positions are 1, 3, 4
R Black King 2
and 5. The S Yellow/Orange Joker given situation demands that
three T Violet Palace consecutive cards – 3, 4 and 5
be assigned U Majenta Princess to these. So, Joker will be at
no.1 from left.

Q 34) b)

Q 35) a) Yellow or Orange

Q 36) a)

Q 37) a)

Q 38 – 44) Based on the given information, the circular arrangement thus formed is drawn below:

38) B 39)E 40)D 41)C 42)A 43)C 44)C

Q 45 – 50) Based on the given information following figure depicting the positions is drawn:

www.ibpsguide.com | www.ibpsguide.in | mock.ibpsguide.in | www.sscexamguide.com


Copyright © 2016 IBPS Guide 11
yoursmahboob.wordpress.com

Q 45) c) Q 46) d) Q 47) b) Q 48) d) Q 49) c) Q 50) d)

2. PUZZLES

Direction (1 – 6):
In a dance club seven males and seven females participated in a dance programme. P, Q, R, S, T, U and V were
the male participants while C, D, E, F, G, H, and I were the female participants. All the fourteen persons were
divided into seven groups. Each group consisted of one male. After the end of the programme each group was
assigned a grade according to its performance. For the purpose of grading seven letters were used, ie X, Y, Z, A,
B, J and K indicating descending order. U‘s group got grade Y. Grade of Q‘s group was better than at least three
groups while grade of R‘s group was better than at least four. T‘s partner was G. E‘s partner was not S. F‘s
group got grade X. P, S and R were not partners of H, I and C respectively. Grade of one group was the same as
the name of member of that group. Grades of D‘s group and H‘s group were better than H‘s group and I‘s group
respectively. Grade of Q‘s group was better than R‘s. Grades of T‘s and P‘s groups were neither J nor K. G‘s
group didn‘t get grade B. E‘s group got the grade K.
Q 1) Which of the following is not correctly matched?

Q 2) Which of the following statements is/are true?


1. C‘s group got the grade J.
2. D‘s group got the grade K

a) Only 2 b) Either 1 or 2 c) Only 1 d) Can‘t say

www.ibpsguide.com | www.ibpsguide.in | mock.ibpsguide.in | www.sscexamguide.com


Copyright © 2016 IBPS Guide 12
yoursmahboob.wordpress.com

Q 4) Grade B was assigned to which person‘s group?


a) D b) F c) Can‘t say d) None of these

Q 5) The group which got the best grade consisted of


a) UI b) QF c) RH d) PD

Q 6) Who among the following was the partner of R?


a) H b) C c)I d) Can‘t say
Directions (Q. 7-12):
Eight candidates A, B, C, D, P, Q, R and S have applied for IBPS exam for different banks, viz BOB, BOI,
Dena Bank, UBI, OBC, Indian Bank, Corporation Bank and UCO Bank but not necessarily in the same order.
There are five male and three female candidates in this group. Each male and each female has applied from
his/her city, viz Chennai, Delhi, Mumbai, Kochi, Patna, Raipur, Gwalior and Agra. No male has applied from
Raipur and Kochi.
P has applied for Indian Bank from Agra. The one who has applied for Corporation Bank is neither from
Mumbai nor from Kochi. B has applied for OBC. His sister C has applied for Dena Bank from Chennai. The
one who belongs to Delhi has applied for BOB. The one who has applied from Mumbai is not a female. D has
applied for UBI and her friend has applied for BOB. Q is from Gwalior and has not applied either for
Corporation Bank or for UCO Bank. The one who has applied for UCO Bank has applied neither from Patna
nor from Mumbai. S has applied from Kochi. A does not apply from Patna.

Q 7). Which of the following groups is a group of female applicants?


a) B, D, C b) D, Q, S c) C, D, S d) P, Q, R e) None of these

Q 8). Who among the following has applied from Ranchi?


a) P b) D c) S d) C e) None of these

Q 9). Which of the following combinations is true?


a) A– UBI – Ranchi b) R – Corporation Bank – Mumbai
c) C – UCO Bank – Chennai d) B – OBC – Mumbai e) None of these

Q 10). Who among the following has applied for BOB?


a) A b) S c) D d) Q e) None of these
www.ibpsguide.com | www.ibpsguide.in | mock.ibpsguide.in | www.sscexamguide.com
Copyright © 2016 IBPS Guide 13
yoursmahboob.wordpress.com

Q 11). B belongs to which of the following cities?


a) Delhi b) Ranchi c) Mumbai d) Gwalior e) None of these

Q 12). Four of the five are alike in a certain way and hence form a group. Which is the one that does not belong
to that group?
a) P b) R c) A d) C e) Q

Direction Q (13 – 17): Study the following information carefully and answer the given questions.

Seven friends A, B, C, D, E, F and G study in Class XII in three different sections A, B and C. Not less than
two friends study in one section. All of them have a different favourite subject—Science, Political Science,
History, Mathematics, English, Biology and Commerce. Each of them has a favourite sport – Cricket, Hockey,
Football, Basketball, Tennis, Volleyball and Table Tennis.
E‘s favourite subject is Commerce and plays Table Tennis.
B and G study in the A section, B‘s favourite subject is History. Neither Tennis nor Volleyball is his favourite
game.
The one whose favourite game is Football has Political Science as his favourite subject and is in Section B only
with C.
A‘s favourite subject is Science. He plays Cricket and studies only with F.
C studies Mathematics and G studies Biology. F plays Basketball.
The one whose favourite subject is Mathematics doesn‘t play Volleyball.

Q 13). Who plays Tennis?


a) A b) C c) E d) F e) G

Q 14). Who among the following study in Section B?


a) CA b) FE c) CD d) ED e) AD

Q 15). How many friends study in Section A?


a) Four b) Three c) Two d) One e) None of these

Q 16). Who among the following plays Hockey?


a) A b) B c) C d) D e) E

Q 17). Which of the following pairs of games is not played by students of the same section?
a) Cricket and Basketball b) Hockey and Tennis
c) Tennis and Football d) Hockey and Volleyball
e) Table Tennis and Volleyball

www.ibpsguide.com | www.ibpsguide.in | mock.ibpsguide.in | www.sscexamguide.com


Copyright © 2016 IBPS Guide 14
yoursmahboob.wordpress.com

Direction (18 – 23):


Ashish, Kamlesh, Sandy, Raju and Sarita are five aspirants or Bank PO. In order to excel in these exams joined
the coaching institute Legion Classes. They joined the classes at different centres. These centres are located in
Noida, Varanasi, Patna, Mumbai and Bengaluru. No two centres of KY classes are in a single town. Owing to
the different problems they were confronting they joined different types of courses. Names of the courses are:
SSCJ, SIA, ACJ, SSK and BPO. Each person has a different surname. Surnames are: Sharma, Tiwari, Yadav,
Rao and Malhotra. Given below are the additional informations, answer the question that follows these
informations –:
1. One who joined SSCJ course is associated with Bengaluru.
2. Three persons among them, ieAshish Sharma, MsRao and the one who is associated with Noida centre, are
good at Non – Verbal Reasoning. But the other two, MrMalhotra and the one who has joined SSK are good at
verbal Reasoning.
3. One who has joined SSK is not Kamlesh.
4. Sarita is not associated with Noida. But she is good at Non – Verbal Reasoning.
5. MsRao and the person who is associated with Patna centre are unfamiliar with each other but good at Non –
Verbal Reasoning.
6. MsTiwari has joined the SSK course.

Q 18) Who is MrMalhotra?


a) Kamlesh b) Ashish c) Raju d) Can‘t say

Q 19) Whose surname is Rao?


a) One who has joined ACJ b) One who has joined SIA
c) One who has joined SSCJ d) Can‘t say

Q 20) One who has joined SSK courses at


a) Bengaluru b) Patna c) Noida d) Can‘t say

Q 21) Which of the following statements is correct?


a) Rao is the surname of Ashish b) Raju is associated with Noida centre?
c) MrYadav joined SSCJ d) None of these

Q 22) Which of the following statements is not necessarily correct?


a) Ashish has joined SIA b) Kamlesh is MrYadav
c) Raju is MrMalhotra d) Sandy is associated with Varanasi

Q 23) Who among the following is studying at Patna centre of KY Classes?

www.ibpsguide.com | www.ibpsguide.in | mock.ibpsguide.in | www.sscexamguide.com


Copyright © 2016 IBPS Guide 15
yoursmahboob.wordpress.com

a) Ashish b) Kamlesh c) Raju d) Sarita

Direction (24 – 31): There are seven friends gathering at a party. They were dressed in different colours of caps
and ties. The colours of ties were – White, Grey, Yellow, Violet, Brown, Orange and Pink. The colours of caps
were Blue, Black, Yellow, Green, Brown, Red and White. No one wore the same colour of cap and tie. P wore
Black colour cap but not Pink or Orange tie. Q wore Grey tie. S wore tie of the same colour as the cap of R.
Similarly, the colour of cap worn by T was same as the colour of tie worn by U. Neither the cap nor the tie worn
by R was brown. V wore Violet tie and White cap.

Q 24) Who wore Brown tie?


a) S b) T c) U d) Data inadequate

Q 25) Which of the following person – cap – tie combination may be true?
a) Q – Yellow – Grey b) U – Blue – Brown c) S – Brown – Yellow
d) P – Black – Orange

Q 26) Which of the given pair of persons wore the same colour of tie and cap respectively?
a) P, V b) T, U c) R, S d) R, U

Q 27) Which of the following person – tie – cap combinations is definitely false?
a) R – Pink – Yellow b) T – Orange – Brown
c) U – Brown – Blue d) None of these

Q 28) If Q wore Green cap, what was the colour of the cap worn by U?
a) Blue b) Red c) Either Blue or Red None of these

Q 29) What was the colour of cap worn by U?


a) Blue b) Red c) Green d) Data inadequate

Q 30) What was the colour of cap that P wore?


a) Black b) Green c) Brown d) None of these

Q 31) Whatcolour of tie T wore?


a) Orange b) Pink c) Either Orange or Pink d) None of these

Direction (32 – 34):


India is a multi – religion, multi – language and multi – cultural country where people belonging to different
religions join in celebrating the festivities together. The Indian Government declares such big occasions as

www.ibpsguide.com | www.ibpsguide.in | mock.ibpsguide.in | www.sscexamguide.com


Copyright © 2016 IBPS Guide 16
yoursmahboob.wordpress.com

public holidays to enable the citizens to enjoy and foster feelings of brotherhood. Five broad – minded persons
belonging to different religions were asked to give their preferences of four such festivals which they would like
to enjoy with like – minded brethren. Their options are –:
A. Holi, Dussehra, Diwali, Guru Nanak Birthday
B. Shivratri, Christmas, Onam, Eid
C. Holi, Shivratri, Christmas, Diwali
D. Holi, Dussehra, Guru Nanak Birthday, Eid
E. Christmas, Diwali, Onam, Guru Nanak Birthday

Q 32) Which pair celebrates Christmas and Onam but not Dussehra and Holi?
a) A and C b) A and E c) B and D d) B and E

Q 33) Who enjoys Holi and Eid but not Diwali and Chhristmas?
a) D b) E c) B d) C

Q 34) Which pair does not participate in Eid and Onam but joins in Holi?
a) D and E b) A and C c) C and D d) A and B

Direction (35 – 38):


There are six different companies namely – A, B, C, D, E and F, in which six friends – Jayesh, Gyanesh,
Raghav, Akshay, Madhav and Kamlesh are currently working. Each one wears company sponsored different
coloured shirt i.e. White, Black, Violet, Brown, Majenta and Red. The one wearing White coloured shirt works
in company D and the one wearing Black colour shirt works in company A. Kamlesh does not work in company
C or E. Jayesh wears Violet shirt and works in company B. Akshay does not work in company E and
Majentacoloured shirt is not sponsored by company C. Madhav works in company F and neither Akshay nor
Gyanesh works in company D. Company E does not sponsor Majenta or Brown coloured shirt and Raghav
works in company A.

Q 35) Which of the following company – person – shirt combination is true?


a) C – Akshay – Black b) D – Raghav – White c) E – Gyanesh – Red
d) C – Madhav – Brown

Q 36) Which of the following is true?


a) Company F sponsors Black coloured shirt b) Madhav wears Red coloured shirt
c) Akshay works in company E d) Red colour is sponsored by company E

Q 37) What is the colour of shirt that is sponsored by company C?


a) White b)Black c) It cannot be ascertained d) None of these

www.ibpsguide.com | www.ibpsguide.in | mock.ibpsguide.in | www.sscexamguide.com


Copyright © 2016 IBPS Guide 17
yoursmahboob.wordpress.com

Q 38) Which of the following sequence of companies represents Jayesh, Gyanesh, Raghav, Akshay, Madhav
and Kamlesh?
a) B, A, E, C, F, D b) B, E, A, C, F, D c) B, A, E, D, F, C d) B, E, D, F, C, A

Direction (39–43): Read the following information carefully and answer the questions given below it—
Six exams Maths, science, History, Economics, English and Hindi are to be scheduled starting from 2nd
March and ending on 8th march wit Sunday being an off day, not
necessarily in the same order. Each of the exam has different time duration: 40 mins, 50 mins, 60 mins, 75
mins, 90 mins and 100 mins, again not necessarily in the same order.
8th march is not sunday and an exam of 40 mins is scheduled on that day. Maths exam is for less than 60 mins
and is scheduled immediately before English exam. There are two exams scheduled between Hindi exam which
is for 100 mins and History exam which is for 60 mins. English exam is before Sunday and there are two days
between sunday and maths exam. Economics exam which is for 75 mins is not scheduled on 2nd march. The
exam schedules on saturday is of 100 mins.

Q 39. How many exams are scheduled before sunday?


(a) Two (b) One (c) Five (d) Three
(e) None of these

Q 40. Which of the following combinations of exam - Day - Time Duration is correct ?
(a) English - Wednesday - 75 mins (b) Maths - Thursday - 50 mins
(c) History - Thursday – 60 mins (d) Hindi - Tuesday - 100 mins
(e) None is correct

Q 41. What is the time duration of science exam?


(a) 90 mins (b) 75 mins (c) 50 mins (d) 40 mins
(e) None of these

Q 42. On which day is Economics exam scheduled?


(a) Monday (b) Saturday (c) Tuesday (d) Friday (e) Cannot be determined

Direction (44 – 50):


Rajeev, Virat, Ajay, Shailesh, Umesh and Pranjal are six friends who joined a youth club. They play different
games – Football, Cricket, Tennis, Basketball, Badminton and Squash. They belong to different states –
Rajasthan, Uttar Pradesh, Karnataka, Gujarat, Maharshtra and Himachal Pradesh. The oldest person in the
group plays Basketball and has Audi car. The youngest person in the group has BMW car and plays Squash.
One who belongs to Maharashtra has Lexus car and he is older than the person who belongs to Himachal
Pradesh and younger than the person who has Land rover car. One who has Aston martin car is older than the
person who has Cadillac car. Shailesh play cricket and Pranjal plays Squash. Ajay, who plays Tennis is older

www.ibpsguide.com | www.ibpsguide.in | mock.ibpsguide.in | www.sscexamguide.com


Copyright © 2016 IBPS Guide 18
yoursmahboob.wordpress.com

than Shailesh and Umesh. Umesh does not play Basketball. When they are arranged according to their age, Ajay
is between Virat and Shailesh. Virat plays football. For each of them the number of letters of their state‘s
nameis same as the number of letters of the game which they are associated with.

Q 44) Who is the owner of Aston Martin Car?


a) Virat b) Ajay c) Shailesh d) Data inadequate

Q 45) The person who plays cricket possesses which car?


a) Land rover b) Cadillac c) Land rover or Cadillac d) Data inadequate

Q 46) Who belongs to Maharshtra?


a) Umesh b) Shailesh c) Ajay d) Data inadequate

Q 47) Who is oldest among them?


a) Virat b) Rajeev c) Data inadequate d) Virat or Rajeev

Q 48) The person who is third from the top, when they are arranged according to decreasing order of their age,
belongs to which state?
a) Uttar Pradesh b) Gujarat c) Maharashtra d) None of these

Q 49) The person who possesses Cadillac car plays –


a) Tennis b) Football c) Cricket d) Data inadequate

Q 50) The person who played football is in possession of which car?


a) Aston Martin b) Lancer c) Either Aston Martin or Lancer d) None of these

PUZZLES (Answers with Detailed Explanation)


Direction (1 – 6) The table given below shows the grades assigned to the seven males and the female members
are – :

Q 1) c) Q 2) c) Q 3) a) Q 4) d) Q 5) b) Q 6) a)

www.ibpsguide.com | www.ibpsguide.in | mock.ibpsguide.in | www.sscexamguide.com


Copyright © 2016 IBPS Guide 19
yoursmahboob.wordpress.com

Direction (7 – 12): Based on the information given figure is drawn below – :

Q 7) c Q 8)b Q 9) d Q 10) a Q 11)c Q 12)d

Direction (13 – 17): Based on the information given figure is drawn below – :

Q 13)b Q 14) c) Q 15) b) Q 16) b Q 17) b

Direction (18 – 23): The tabulated data given below shows the students with courses they applied for in the
respective centres of KY Classes set on the basis of the information given in the questions –:

www.ibpsguide.com | www.ibpsguide.in | mock.ibpsguide.in | www.sscexamguide.com


Copyright © 2016 IBPS Guide 20
yoursmahboob.wordpress.com

Q 18) c) Q 19) c) Q 20) d) Q 21) d) Q 21) d) Q 23) a)

Direction (24 – 31): The tabulated data based on the given information is shown below:

Q 24) c) Q 25) b) Q 26) a) Q 27) d) Q 28) c) Q 29) d) Q 30) d) Q 31) c)

Direction (32 – 34):

Q 32) d) Q 33) a) Q 34) b)

Direction (35 – 38): Based on the given information in the question the tabulated data is given below:

Q 35) c) Q 36) d) Q 37) d) Q 38) b)

Direction (39 - 43):

www.ibpsguide.com | www.ibpsguide.in | mock.ibpsguide.in | www.sscexamguide.com


Copyright © 2016 IBPS Guide 21
yoursmahboob.wordpress.com

Q 39) e Q 40) b Q 41) d Q 42) a Q 43) d

Direction (44 – 50) Based on the given information we get –


The decreasing order of their age is – Rajeev >Virat> Ajay >Shailesh>Umesh>Pranjal
And the table with the whole information is given below –

Q 44) d) Q 45) c) Q 46) a) Q 47) b) Q 48) d) Q 49) d) Q 50) c)

3. CODING-DECODING
Directions (Q. Nos. 1-5) Study the following information carefully and answer the questions given below it.
Digits in the numbers are to be coded as follows

Digits / % 4 2 6 * 7 α # 5 9 @ © 3 8 1 $
Symbols
Letters/ W I R L U M V T C G S Z K O F Y
Codes

Conditions
(i) If the first unit in the group is an even digit and the last unit is a symbol, both these are to be coded as the
code for the symbol.
(ii) If the first unit in the group is an odd digit and the last unit is an even digit their codes are to be
interchanged.
(iii) If both the first and the last units in the group are symbols, both these are to be coded as ‗X‘.

Q 1. 9124α6
a) GFRIVL b) LFRIVG c) GFRIVG d) XFRIVX e) None of these

www.ibpsguide.com | www.ibpsguide.in | mock.ibpsguide.in | www.sscexamguide.com


Copyright © 2016 IBPS Guide 22
yoursmahboob.wordpress.com

Q 2. 4@312α
a) VSKFRV b) VSKFRI c) XSKFRX d) ISKFRV e) None of these

Q 3. @91#26
a) LGFYRS b) SGFYRS c) XGFYRX d) SGFTRL e) None of these

Q 4. 387#©9
a) KMOTZG b) KOMZTG c) GOMTZK d) KOMTZG e) None of these

Q 5. %4187*
a) WIFOMU b) UNIFOMW c) XIFOMX d) UIFOMU e) None of these

Directions (Q. Nos. 6-10) Study the following information carefully and answer the questions given below it.
Digits in the numbers are to be coded as follows

Letter B I J G A K E H D O F
Digit / 2 9 7 @ % 4 5 8 $ # 6
Symbol

Conditions:
i) If the first and the last letters are vowels, both are to be coded as £.
ii) If the first letter is a consonant and the last letter is a vowel, both are to be coded by the code for the vowel.
iii) If the first and the last letters are consonants, both are to be coded by the code for the first letter.

Q 6. OFDBJI
a) #6$279 b) #6$27£ c) £$627£ d) £6$27£ e) None of these

Q 7. BGJEAO
a) #@57#% b) #@%57# c) #@75%# d) 2@%57# e) None of these

Q 8. HBIAFD
a) 892%6$ b) 829%68 c) $29%6$ d) $92%68 e) None of these

Q 9. BKAEFJ
a) 24%567 b) 24%652 c) 24%562 d) 74%562 e) None of these

Q 10. AEKDIB
a) %54$92 b) %54$9% c) 2%54$2 d) %5$492 e) None of these

Directions (Q. Nos. 11-15) Study the following information carefully and answer the questions given
below it.
Digits in the numbers are to be coded as follows

Digits 9 6 8 2 7 5 1 3 4
www.ibpsguide.com | www.ibpsguide.in | mock.ibpsguide.in | www.sscexamguide.com
Copyright © 2016 IBPS Guide 23
yoursmahboob.wordpress.com

Letters / R @ T © U L B Q %
Symbols

Conditions
(i) If the first digit is odd and the last digit is even, the codes for the first and last digits are to be reversed.
(ii) If the first and the last digits are even, both are to be coded as *.
(iii) If the first and the last digits are odd, both are to be coded as $.

Q 11. 671254
a) @UB©L% b) $UB©L$ c) *UB©L* d) %UB©L© e) None of these

Q 12. 215349
a) RBLQ%© b) *BLQ%* c) $BLQ%$ d) ©BLQ%R e) None of these

Q 13. 591426
a) @RB%©L b) LRB%©@ c) @BR%©L d) $RB%©* e) None of these

Q 14. 813469
a) RBQ%@T b) TBQ%@R c) TBL%@R d) *BQ%@* e) None of these

Q 15. 794821
a) UR%T©B b) BR%T©U c) U%RT©B d) $R%T© $ e) None of these

Directions (Q. Nos. 16-20) Study the following information carefully and answer the questions given
below it.
Digits in the numbers are to be coded as follows
Digits 5 3 6 4 9 2 1 8 7
Codes Z P C H K V O R L

Conditions
(i) If the first as well as the last digits are even, both are to the coded by the code for the first digit.
(ii) If the first as well as the last digits are odd, both are to be coded by the code for the last digit.

Q 16. 397416
a) PKLHOP b) CKLHOC c) PVLHOC d) PKLHOC e) None of these

Q 17. 562183
a) PCVORP b) ZCVORZ c) PCVORZ d) ZCVORP e) None of these

Q 18. 734192
a) ZPHOKV b) LPCOKV c) LPHOVK d) LPHOKV e) None of these

Q 19. 627851
a) PULRZO b) CVLZRO c) CVLRZO d) CKLRZO e) None of these
www.ibpsguide.com | www.ibpsguide.in | mock.ibpsguide.in | www.sscexamguide.com
Copyright © 2016 IBPS Guide 24
yoursmahboob.wordpress.com

Q 20. 812354
a) RLVPZH b) HOVPZH c) ROVPZR d) HOVPRZ e) None of these

Directions (Q. Nos. 21-25) Study the following information carefully and answer the questions given
below it.
Digits in the numbers are to be coded as follows
Letters J A L S E Q G D K M B I N P R
Number 8 5 # 9 2 £ 6 % 3 7 * 4 @ $ 1
/ Symbol
Code

Conditions
(i) If the first letter is a vowel and the last a consonant, the codes for the first and the last are to be
interchanged.
(ii) If the first letter is a consonant and the last a vowel, both are to be coded as the code of the vowel.
(iii) If no vowel is present in the group of letters, the second and the fifth letters are to be coded as ©.

Q 21. KQAPJE
a) 3£5$82 b) 3£58$2 c) 2£5$82 d) 2£5$83 e) None of these

Q 22. EMANRB
a) *75@12 b) 275@1* c) ©75@2© d) *75@1* e) None of these

Q 23. JAQDKP
a) 85£%38 b) $5£%3$ c) $5£%38 d) $5£3%8 e) None of these

Q 24. QDBGRM
a) £%*617 b) £©*6©7 c) £%*167 d) % £*61© e) None of these

Q 25. IKQLMS
a) 43£#74 b) ©3£#7© c) 4£3#74 d) 93£#74 e) None of these

Directions (Q. Nos. 26-30) Study the following information carefully and answer the questions given
below it.
Digits in the numbers are to be coded as follows
Letter A I H R U J W E K D F P M T

Digit/Symbol © 7 3 # @ % 9 8 1 2 6 5 4 $

Conditions:
i) If the first letter is a consonant and the last letter is a vowel their codes are to be interchanged.
ii) If both the first and the last letters are consonants both are to be coded as the code for the first letter.
www.ibpsguide.com | www.ibpsguide.in | mock.ibpsguide.in | www.sscexamguide.com
Copyright © 2016 IBPS Guide 25
yoursmahboob.wordpress.com

iii) If both the first and the last letters are vowels both are to be coded as the last letter.

Q 26. EWMAPH
a) 89©435 b) 894©58 c) 894©53 d) 89©458 e) None of these

Q 27. DAIMWU
a) 2©749@ b) @©749@ c) @©7492 d) 2©7492 e) None of these

Q 28. MUPRKE
a) 8@5#18 b) 4@5#14 c) 4@5#18 d) 8@5#14 e) None of these

Q 29. AMTHDU
a) @$423@ b) @4$32@ c) @4$32© d) ©4$32@ e) None of these

Q 30. MARKET
a) 4©#184 b) 4©1#84 c) 4©#18$ d) 4#©148 e) None of these

Directions (Q. Nos. 31-35) Study the following information carefully and answer the questions given
below it.
Digits in the numbers are to be coded as follows

Letter H B N Z Y G P M A C X E D O U
Number © 8 1 % 5 ? 3 $ 4 7 9 β 6 2 #
/ Symbol
Code

Conditions:
(i) If both the first and last elements are vowels, the codes for the vowels are to be interchanged.
(ii) If the group of elements contains a single vowel, that vowel is to be coded as the code for the element
following it.
(iii) If the second element is a vowel and the fifth element is a consonant, the vowel is to be coded as the code
for the consonant.

Q 31. PXUNCM
a) $9#173 b) $91173 c) 39717$ d) 39117$ e) 39#17$

Q 32. DEHAZN
a) 6β©441 b) 6©%4%1 c) 11©4%6 d) 6β©4β7 e) 6%©4%1

Q 33. MHCYBG
a) $©758? b) ?©758$ c) $©?758 d) 758$©? e) ?c57$8

Q 34. OMPCZA
a) 2$37%4 b) 437$%2 c) 4$37%2 d) 2%37$4 e) 4$3722

www.ibpsguide.com | www.ibpsguide.in | mock.ibpsguide.in | www.sscexamguide.com


Copyright © 2016 IBPS Guide 26
yoursmahboob.wordpress.com

Q 35. OUBNYE
a) β58152 b) β#8152 c) 2#815β d) 25815β e) β581#2

Q 36. If the word ‗TABLECLOTH‘ is coded as ‗XEMRANRIXT‘, how can ‗HOTEL‘ be coded?
a) RIXAT b) TIXAR c) TAXIR d) RAXIT e) None of these

Q 37. If ‗SPEAKER‘ is coded as 25-22-11-7-17-11-24s, then how will you cod ‗DANGER‘ in the same code?
a) 11-7-20-16-11-24 b) 13-7-20-9-11-25 c) 10-7-20-13-10-24 d) 13-7-20-10-11-25 e) 10-7-19-14-11-24

Q 38. If ‗apple‘ is called ‗milk‘, ‗milk‘ is called ‗soap‘, ‗soap‘ is called ‗oil‘, ‗oil‘ is called ‗petrol‘ and ‗petrol‘
is called ‗apple‘, then which of the following will be used for washing clothes?
a) Petrol b) Milk c) Apple d) Soap e) Oil

Q 39. On another planet, the local terminology for ‗earth‘, ‗water‘, ‗light‘, ‗air‘ and ‗sky‘ are ‗sky‘, ‗light‘, ‗air‘,
‗water‘ and ‗earth‘, respectively. If someone is thirsty there, what would he drink?
a) Light b) Air c) Sky d) Water e) None of these

Q 40. If in a certain code language, ‗WORKING‘ is coded as ‗XQUONTN‘, then how is ‗GAMBLE‘ coded in
that same code language?
a) JDOCMF b) CLEMNK c) HCPFQK d) AELGMN e) None of these

Q 41. In a certain code, ‗BANKEXAM‘ is written as ‗LOBCNBYF‘ and ‗COVERS‘ is written as ‗WPDTSF‘.
How is ‗LATE‘ written in that same code?
a) FUBM b) UFMB c) BMFU d) BMDS e) None of these

Q 42. In a code language, ‗OPERATION‘ is written as ‗CXFBWYQCL‘, and ‗ORGANISATION‘ is written as


‗CBDWLQJWYQCL‘. How would ‗SEPARATION‘ be coded?
a) JFXWBWYQCL b) EJXEBYQCL c) QCLYWBFXJE d) JFQYWBCXQL e) None of these

Q 43. If ‗Chair‘ is known as ‗mug‘, ‗mug‘ is known as ‗glass‘, ‗glass‘ is known as ‗plate‘, ‗plate‘ is known as
‗spoon‘, then which utensil will be used for drinking water?
a) Mug b) Plate c) Glass d) Spoon e) None of these

Q 44. In a certain code language 'te da ka ni' means 'intelligence is in genes', 'se po lo ni' means, 'genes are not
responsible' and 'ba da fu te' means 'intelligence is through experience'. What does 'ka' stand for in that code
language?
a) genes b) through c) intelligence d) in e) responsible

Q 45. In a certain code BUILDER is written as JVCKSFE. How is SEALING written in that code?
a) BTFKHOJ b) JOHKBFT c) TFBKHOJ d) BFTKJOH e) None of these

Q 46. In a certain code PAGEDOWN is written as ' FHBQOXPE ', how is SANCTION written in that code?
a) DOBTMNHS b) DOBTOPJU c) TBODSHNM d) TBODMNHS e) None of these

www.ibpsguide.com | www.ibpsguide.in | mock.ibpsguide.in | www.sscexamguide.com


Copyright © 2016 IBPS Guide 27
yoursmahboob.wordpress.com

Q 47. If the animals which fly in the sky are called ‗hunters‘, those living in water are called ‗snakes‘, animals
who crawl are called ‗flying‘ and those which can walk are called ‗swimmers‘, then what will a lizard be
called?
a) Snakes b) Hunters c) Swimmers d) Flying e) None of these

Q 48. 'EJ' is related to 'JO' in the same way as 'QT' is related to


a) UY b) UX c) UZ d) VY e) None of these

Q 49. If a 'bus' is called 'bike', 'bike' is called 'cycle', 'cycle' is called 'boat', 'boat' is called 'aeroplane', 'aeroplane'
is called 'car' and car' is called 'scooter' which of the following can fly?
a) Boat b) Aeroplane c) Car d) Cycle e) None of these

Q 50. In a certain code ―BREADTH‖ is written as ―CQFZFSI‖. How is OCTOBER written in that code?
a) PBUNCDS b) PBUCNSD c) BPUNCSD d) PBUCNDS e) None of these

Answer:
1. b 2. a 3. d 4. d 5. c 6. d 7. c 8. b 9. c 10. a 11. c 12. d 13. a 14. b 15. d 16. d 17. c 18. d 19. c 20. c
21. c 22. a 23. e 24. b 25. d 26. c 27. c 28. d 29. b 30. a 31. d 32. e 33. a 34. c
35. a 36. b 37. c 38. e 39. a 40. c 41. c 42. a 43. b 44. d 45. c 46. a 47. d 48. d 49. c 50. C

3. Coding-Decoding (Answer with Explanation)

Q (1-5) 4. d)
1. b) 3 8 7 # © 9
9 1 2 4 α 6 ↓ ↓ ↓ ↓ ↓ ↓
↓ ↓ ↓ ↓ ↓ ↓ K O M T Z G
L F R I V G (no such condition is follow)
(Second (ii) condition follows)
5. c)
2. a) % 4 1 8 7 *
4 @ 3 1 2 α ↓ ↓ ↓ ↓ ↓ ↓
↓ ↓ ↓ ↓ ↓ ↓ X I F O M X
V S K F R V (Third (iii) condition follows)
(First (i) condition follows)
Q (6-10)
3. d) 6. d) Condition (i) applies
O F D B J I
@ 9 1 $ 2 6 ↓ ↓ ↓ ↓ ↓ ↓
↓ ↓ ↓ ↓ ↓ ↓ £ 6 $ 2 7 £
S G F T R L
(no such condition is follow) 7. c) Condition (ii) applies
B G J E A O
www.ibpsguide.com | www.ibpsguide.in | mock.ibpsguide.in | www.sscexamguide.com
Copyright © 2016 IBPS Guide 28
yoursmahboob.wordpress.com

↓ ↓ ↓ ↓ ↓ ↓ 7 9 4 8 2 1
# @ 7 5 % # ↓ ↓ ↓ ↓ ↓ ↓
$ R % T © $
8. b) Condition (iii) applies (Condition (iii) is applicable)
H B I A F D
↓ ↓ ↓ ↓ ↓ ↓ Q (16-20)
8 2 9 % 6 8 16. d)
3 9 7 4 1 6
↓ ↓ ↓ ↓ ↓ ↓
9. c) Condition (iii) applies P K L H O C
B K A E F J (no condition follows)
↓ ↓ ↓ ↓ ↓ ↓
2 4 % 5 6 2 17. c)
5 6 2 1 8 3
10. a) No condition follows ↓ ↓ ↓ ↓ ↓ ↓
A E K D I B P C V O R P
↓ ↓ ↓ ↓ ↓ ↓ (condition number (ii) follows)
% 5 4 $ 9 2
18. d)
Q (11-15) 7 3 4 1 9 2
11. c) ↓ ↓ ↓ ↓ ↓ ↓
6 7 1 2 5 4 L P H O K V
↓ ↓ ↓ ↓ ↓ ↓ (no condition follows)
* U B © L *
(condition (ii) is applicable) 19.c)
6 2 7 8 5 1
12. d) ↓ ↓ ↓ ↓ ↓ ↓
2 1 5 3 4 9 C V L R Z O
↓ ↓ ↓ ↓ ↓ ↓ (no condition follows)
© B L Q % R
(none of the conditions is applicable) 20. c)
8 1 2 3 5 4
13. a) ↓ ↓ ↓ ↓ ↓ ↓
5 9 1 4 2 6 R O V P Z R
↓ ↓ ↓ ↓ ↓ ↓ (condition number (i) follows)
@ R B % © L
(condition (i) is applicable) Q (21-25)
21. c)
14. b) KQAPJE - 2£5$82
8 1 3 4 6 9 Condition ii follows
↓ ↓ ↓ ↓ ↓ ↓
T B Q % @ R 22. a)
(none of the conditions is applicable) EMANRB - *75@12
Condition ii follows
15. d)
www.ibpsguide.com | www.ibpsguide.in | mock.ibpsguide.in | www.sscexamguide.com
Copyright © 2016 IBPS Guide 29
yoursmahboob.wordpress.com

23. e)
JAQDKP - 85£%3$ 34. c)
No condition follows OMPCZA - 4$37%2
Condition (i) follows.
24. b)
QDBGRM - £©*6©7 35. a)
Condition iii follows OUBNYE - β58152
Conditions (i) and (iii) follow
25. d)
IKQLMS - 93£#74 36. b)
Condition I follows According to question,
T-X
Q (26-30) A-E
26. c) B-M
EWMAPH - 894©53 L-R
No condition follows E-A
C-N
27. c) L-R
DAIMWU - @©7492 O-I
Condition (i) follows T-X
H-T
28. d) Similarly,
MUPRKE - 8@5#14 H-T
Condition (i) follows O-I
T-X
29. b) E-A
AMTHDU - @4$32@ L-R
Condition (iii) follows
37. c) As
30. a) S →+ 6 = 25
MARKET - 4©#184 P →+ 6 = 22
Condition (ii) follows E →+ 6 = 11
A →+ 6 = 7
Q (31-35) K →+ 6 = 17
31. d) E →+ 6 = 11
PXUNCM - 39117$ R →+ 6 = 24
Conditions (ii) follows. Similarly,
D → + 6 = 10
32. e) A →+ 6 = 7
DEHAZN - 6%©4%1 N →+ 6 = 20
Conditions (iii) follows. G →+ 6 = 13
R →+ 6 = 24
33. a) E →+ 6 = 11
MHCYBG - $©758?
No condition follows.
www.ibpsguide.com | www.ibpsguide.in | mock.ibpsguide.in | www.sscexamguide.com
Copyright © 2016 IBPS Guide 30
yoursmahboob.wordpress.com

38. e) Soap is used for washing clothes and here A →W


soap is called oil. N →L
I →Q
39.a) Water quenches thirst and here water is called S→J
as light. A →W
T →Y
40. c) As I →Q
W →+1 = X O →C
O → +2 = Q N →L
R →+3 = U
K →+4 = O Similarly,
I →+5 = N S→J
N →+6 = T E→F
G →+7 = N P→X
Similarly, A→W
G →+1= H R→B
A → +2 = C A→W
M → +3 = P T→Y
E → +6 = K I→Q
L → +5 = Q O→C
B → +4 = F N→L

41. c) If BANK = KNAB = K +1 = L; N +1 = O; A 43.b) Glass is used for drinking water and here
+1 B; B +1 =C glass is called as plate.
EXAM = MAXE = M +1 = N; A +1 = B; X +1 = Y;
E +1 = F 44. d) te da ka ni → intelligence is in genes ...(1)
And COV = VOC = V +1 =W; O +1 = P; C +1 =D se po lo ni → genes are not responsible ...(2)
ERS = SRE = S +1 = T; R +1 = S; E +1 = F ba da fu te → intelligence is through experience
Then, LA = AL = A +1 = B; L +1 = M ...(3)
TE = ET = E +1= F; T +1 = U From (1) and (2), ni → genes
From (1) and (3), da and te → intelligence and is
42. a) \ ka → in
O →C
P →X 45.c) BUI = IUB = I+1 = J; U +1 = V; B +1 = C
E →F L -1 = K
R →B DER = RED = R +1 = S; E +1 =F; D +1 = E
A →W Similarly,
T →Y SEA = AES = A +1 = B; E +1 = F; S +1 = T
I →Q L -1 = K
O →C ING = GNI = G +1 = H; N +1 = O; I +1 = J
N →L
And 46.a) PAGE = EGAP +1 for all =FHBQ
O →C DOWN = NWOD +1 for all = OXPE
R →B Similarly,
G →D SANC = CNAS +1 for all = DOBT
www.ibpsguide.com | www.ibpsguide.in | mock.ibpsguide.in | www.sscexamguide.com
Copyright © 2016 IBPS Guide 31
yoursmahboob.wordpress.com

TION = NOIT +1 for all = MNHS E +1 = F


A -1 = Z
47.d) Lizard → flying D +1 = F
T -1 = S
48.d) Each corresponding letter moves five places H +1 = I
forward in the alphabet. Similarly,
O +1 = P
49.c) 'Aeroplane' can fly and 'aeroplane' is called C -1 = B
'car'. T +1 = U
O -1 = N
50. c) As B +1 = C
B +1 = C E -1 = D
R -1 = Q R +1 = S

4. INPUT - OUTPUT

Direction (Q. 1-5): Study the given information and answer the following questions:

When a word and number arrangement machine is given an input line of words and numbers, it arranges them
following a particular rule. The following is an illustration of input and its rearrangement. (All the numbers are
two-digit numbers.)
Input : rocking 18 available 50 reach 89 gave 33 process 81 23 goal.
Step I: gave rocking 18 available 50 reach 33 process 81 23 goal 89
Step II: gave goal rocking 18 available 50 reach 33 process 23 81 89
Step III: gave goal reach rocking 18 available 33 process 23 50 81 89
Step IV: gave goal reach process rocking 18 available 23 33 50 81 89
Step V: gave goal reach process rocking available 18 23 33 50 81 89
Step V is the last step of the above arrangement as the intended arrangement is obtained. As per the rules
followed in the above steps, find out the appropriate steps for the given input.
Input: 23 technology 52 number 94 extra 31 playing 77 66 fabricate torcher

Q 1. If in the last step all the words get rearranged in alphabetical order, which of the following words would
retain its original position?
a) fabricate b) extra c) playing d) number e) None of these

Q 2. Which step number would be the following output?


extra number playing torcher 23 technology 31 fabricate 52 66 77 94
a) Step III b) Step IV c) Step V d) There will be no such step. e) None of these

Q 3. Which of the following would be the last step of the rearrangement?


www.ibpsguide.com | www.ibpsguide.in | mock.ibpsguide.in | www.sscexamguide.com
Copyright © 2016 IBPS Guide 32
yoursmahboob.wordpress.com

a) IV b) V c) VI d) VII e) None of these

Q 4. In Step IV, if ‗extra‘ is related to ‗94‘ and ‗number‘ is related to ‗77‘ then ‗23‘ is related to which of the
following, if the same pattern is followed?
a) 52 b) 66 c) fabricate d) 31 e) None of these

Q 5. Which of the following would be at the 7th position from the left in Step III?
a) 31 b) fabricate c) 52 d) playing e) None of these

Directions (Q. 6-10): Study the following information and answer the given questions:
A word and number arrangement machine when given an input line of words and numbers rearranges them
following a particular rule in each step. The following is an illustration of input and rearrangement.
Input: sanction payroll revenue 44 passenger otherwise 80 demonstrating information shipping 60
Step I: payroll sanction revenue 44 passenger otherwise 80 demonstrating information shipping 60
Step II : payroll sanction passenger revenue 44 otherwise 80 demonstrating information shipping 60
Step III: payroll sanction passenger information revenue 44 otherwise 80 demonstrating shipping 60
Step IV: payroll sanction passenger information demonstrating revenue 44 otherwise 80 shipping 60
Step V: payroll sanction passenger information demonstrating otherwise 80 revenue 44 shipping 60
And step V is the last step of the above input, as the desired arrangement is obtained. As per the rules followed in
the above steps, find out in each of the following questions the appropriate step for the given input.
Input: ―indignity quartz 58 premier 42 customer imagination salary dynamic 15 viewer 36‖

Q 6. If in Step IV ‗salary‘ is related to ‗36‘ and ‗customer‘ is related to ‗viewer‘, in the same way, ‗indignity‘ is
related to which of the following?
a) premier b) 42 c) 15 d) dynamic e) None of these

Q 7. How many steps would be required to get the final output?


a) Six b) Seven c) Five d) Four e) None of these

Q 8. Which step number is the following output? ―salary customer indignity imagination dynamic 42 premier 15
quartz 58 viewer 36‖
a) Step V b) Step IV c) Step VI d) Step III e) There is no such step

Q 9. Which of the following represents the position of ‗imagination‘ in the third step?
a) 6th from the left b) 9th from the left c) 9th from the right
d) 6th from the right e) None of these

Q 10. Which word/number would be at the 5th place from the left in Step IV?
a) dynamic b) 15 c) quartz d) premier e) None of these

Directions (Q. 11-15): Study the given information and answer the following questions.

A word and number arrangement machine when given an input line of words and numbers rearranges them
following a particular rule. The following is an illustration of input and its rearrangement:
www.ibpsguide.com | www.ibpsguide.in | mock.ibpsguide.in | www.sscexamguide.com
Copyright © 2016 IBPS Guide 33
yoursmahboob.wordpress.com

Input: can now 18 16 27 all done 36 insert 49


Step I: 16 18 can now 27 all done 36 insert 49
Step II: 16 18 all can now 27 done 36 insert 49
Step III: 16 18 all can 27 36 now done insert 49
Step IV: 16 18 all can 27 36 done insert now 49
Step V: 16 18 all can 27 36 done insert 49 now
Step V is the last step of the rearrangement. As per the rules followed in the above steps, find out in each of the
following questions the appropriate step for the following input.
Input: 57 19 professor male 28 correct 36 38 47 female doctor 51 study

Q 11. What is the position of ‗male‘ in the final step?


a) 7th from the left b) 2nd from the right c) 8th from the left
d) 5th from the right e) None of these

Q 12. Which step number is the following output?


19 28 correct doctor 36 38 female male 47 51 57 professor study
a) Step V b) Step VI c) Step VII d) There is no such step. e) None of these

Q 13. Which element is third to the right of ‗female‘ in Step V?


a) 38 b) professor c) study d) 51 e) None of these

Q 14. How many steps will be required to complete the arrangement of the given input?
a) Five b) Six c) Seven d) Eight e) None of these

Q 15. Which of the following is the third element from the left end of Step III?
a) 38 b) 57 c) correct d) doctor e) None of these

Direction (Q. 16-20): Study the given information and answer the following questions:

A word and number arrangement machine when given a particular input, rearranges it following a particular rule.
The following is the illustration of the input and the steps of rearrangement.
Input: you access 40 energy 29 nothing on day 5 18
Step I: access you 40 energy 29 nothing on day 5 18
Step II: access 40 you energy 29 nothing on day 5 18
Step III: access 40 energy you 29 nothing on day 5 18
Step IV: access 40 energy 5 you 29 nothing on day 18
Step V: access 40 energy 5 on you 29 nothing day 18
Step VI: access 40 energy 5 on day you 29 nothing 18
Step VII: access 40 energy 5 on 29 day you nothing 18
Step VIII: access 40 energy 5 on 29 day 18 you nothing
Step IX: access 40 energy 5 on 29 day 18 nothing you
And Step IX is the last step.

www.ibpsguide.com | www.ibpsguide.in | mock.ibpsguide.in | www.sscexamguide.com


Copyright © 2016 IBPS Guide 34
yoursmahboob.wordpress.com

Q 16. If the following is the II step of an input what will be V th step?


Step II: Again 93 she 40 wins 13 Olympic 33 the 3
a) again 93 she 3 the 33 Olympic 13 wins 40 b) again 93 Olympic she 40 wins 13 33 the 3
c) again 93 Olympic 3 she 40 the wins 13 33 d) again 93 Olympic 3 she 40 the 13 wins 33
e) None of these

Q 17. Which of the following is the last step for the Input
‗every 9 four ideal 22 3 under cost 5‘?
a) cost every 9 four 5 ideal 22 under 3 b) every 22 ideal 3 under 9 cost 5 four
c) every 22 under 3 ideal 9 cost 5 four d) every 22 ideal 3 under 5 cost 9 four
e) None of these

Q 18. Which step will be the last step for the Input ‗election 17 fees open 41 27 delay 15‘?
a) IV b) V c) VI d) VII e) None of these

Q 19. Which word/number will be at 4th from the left in step V for the given input in above question 3?
a) 41 b) delay c) open d) 15 e) None of these

Q 20. Which word/number will be 3rd to the right of ―41‖ in step IV for the given input in Q. 3?
a) open b) delay c) 15 d) 17 e) None of these

Direction (Q. 21-25): Study the given information and answer the following questions:
When a word and number arrangement machine is given an input line of words and numbers, it arranges them
following a particular rule. The following is an illustration of an input and rearrangement:

Input: 45 native charge 33 48 dark total freeze 62 88 98 gold office 21


Step I: 45 native charge 33 48 dark freeze 62 88 98 gold office total 21
Step II: 45 native charge 48 dark freeze 62 88 98 gold total 21 office 33
Step III: charge 48 dark freeze 62 88 98 gold total 21 office 33 native 45
Step IV: charge dark freeze 62 88 98 total 21 office 33 native 45 gold 48
Step V: charge dark 88 98 total 21 office 33 native 45 gold 48 freeze 62
Step VI: charge 98 total 21 office 33 native 45 gold 48 freeze 62 dark 88
Step VII: total 21 office 33 native 45 gold 48 freeze 62 dark 88 charge 98
Step VII is the last step of the above arrangement as the intended arrangement is obtained. As per the rules
followed in the given steps, find out the appropriate steps for the given input.
Input: 35 quite head clear 50 65 98 slow giant 71 82 19 oliver music

Q 21. Which of the following is Step III of the given input?


a) quite 82 head 50 giant 35 clear 19 65 98 slow 71 oliver music
b) head clear 65 98 giant 71 82 music slow 19 quite 35 oliver 50
c) head clear 98 giant 71 82 slow 19 quite 35 oliver 50 music 65
d) head clear 98 65 giant 71 82 music slow 19 quite 35 oliver 50
e) None of these

Q 22. How many steps will be required to complete the given input?
www.ibpsguide.com | www.ibpsguide.in | mock.ibpsguide.in | www.sscexamguide.com
Copyright © 2016 IBPS Guide 35
yoursmahboob.wordpress.com

a) Five b) Six c) Seven d) Eight e) None of these

Q 23. What is the position of ‗music‘ from the left end in the final step?
a) Sixth b) Seventh c) Eighth d) Fifth e) Ninth

Q 24. Which element is fourth to the right of ‗giant‘ in Step V?


a) clear b) 19 c) quite d) 98 e) 35

Q 25. Which of the following is the fourth element from the left end of Step VI?
a) clear b) music c) 71 d) oliver e) None of these

Directions (Q. 26-30): Study the following information toanswer the given questions.
A word and number arrangement machine when given an input line of words and numbers rearranges them
following a particular rule. The following is an illustration of input and rearrangement. (All the numbers are two-
digit numbers.)
Input: yellow jovial 48 cross truth 20 connect staff that 78
Step I: 20 truth yellow jovial 48 cross connect staff that 78
Step II: 20 truth 48 jovial yellow cross connect staff that 78
Step III: 20 truth 48 jovial 78 that yellow cross connect staff
Step IV: 20 truth 48 jovial 78 that yellow staff cross connect
Step V: 20 truth 48 jovial 78 that yellow staff connect cross
Step V is the last step of the rearrangement. As per the rules followed in the above steps, find out in each of the
following questions the appropriate step for the following input.
Input: union magnet 89 chalk 21 absorb liverpool black honest 11 everyday

Q 26. Which step number will be the following output?


11 honest 21 chalk 89 magnet union absorb liverpool black everyday
a) Step II b) Step III c) Step IV d) There will be no such step. e) None of these

Q 27. How many steps will be required to get the final output?
a) Five b) Six c) Seven d) Four e) None of these

Q 28. Which word/number would be the fifth from the right in Step V?
a) magnet b) union c) 89 d) liverpool e) None of these

Q 29. What is the position of ‗black‘ in Step IV?


a) 9th from the left b) 11th from the left c) 2nd from the right
d) 4th from the right e) None of these

Q 30. If in a certain way ‗honest‘ is related to ‗absorb‘ and ‗chalk‘ is related to ‗black‘ then ‗magnet‘ would be
related to which of the following in the last step?
a) 21 b) union c) everyday d) liverpool e) None of these

Directions (Q. 31-35): Study the following information carefully to answer the given questions.

www.ibpsguide.com | www.ibpsguide.in | mock.ibpsguide.in | www.sscexamguide.com


Copyright © 2016 IBPS Guide 36
yoursmahboob.wordpress.com

A word and number arrangement machine when given an input line of words and numbers rearranges them
following a particular rule in each step. The following is an illustration of the input and its rearrangement.
Input: song 72 excel owl 28 11 power 81 40 cut
Step I: excel song 72 owl 28 power 81 40 cut 11
Step II: excel owl song 72 power 81 40 cut 11 28
Step III: excel owl cut song 72 power 81 11 28 40
Step IV: excel owl cut power song 81 11 28 40 72
Step V: excel owl cut power song 11 28 40 72 81
Step V is the last step of the above input. As per the rules followed in the above steps, find out in each of the
following questions, the appropriate step for the given input below and answer the questions based on it.
Input: collection 53 vitamin 49 33 article 45 application 23 ice 69 encourage best

Q 31. What will be the position of ‗69‘ in Step IV?


a) 9th from the left b) Fifth from the right c) Extreme left d) Extreme right
e) None of these

Q 32. Which step would be the following output? ‗application article encourage collection 53 vitamin 49 ice 69
best 23 33 45‘
a) IV b) V c) VI d) III e) VII

Q 33. How many steps will be required to complete the arrangement of the above input?
a) Four b) Five c) Six d) Seven e) None of these

Q 34. Which of the following steps would be the last step but one?
a) V b) VI c) VII d) IV e) None of these

Q 35. Which word/number would be at the 9th position from the right end in Step V?
a) collection b) vitamin c) best d) 53 e) 49

Directions (Q. 36-40): Study the following information to answer the given questions.
A word and number arrangement machine when given an input line of words and numbers rearranges them
following a particular rule. The following is an illustration of an input and rearrangement.
Input: machine autograph 8 4 2 personality 9 on 6 venn 11 sender 7 Educator
Step I: autograph 9 machine 8 4 2 personality on 6 venn 11 sender 7 Educator
Step II: autograph 9 Educator 8 machine 4 2 personality on 6 venn 11 sender 7
Step III: autograph 9 Educator 8 machine 7 4 2 personality on 6 venn 11 sender
Step IV: autograph 9 Educator 8 machine 7 on 2 4 personality 6 venn 11 sender
Step V: autograph 9 Educator 8 machine 7 on 2 personality 11 4 6 venn sender
Step VI: autograph 9 Educator 8 machine 7 on 2 personality 11 sender 6 4 venn
Step VII: autograph 9 Educator 8 machine 7 on 2 personality 11 sender 6 venn 4
Step VII is the last step. As per rules followed in above steps, find out in each of the following questions the
appropriate step for the input given below.
Input: inflexible on 7 6 factory dollar 5 8 10 agree projects 2 9 immovable

www.ibpsguide.com | www.ibpsguide.in | mock.ibpsguide.in | www.sscexamguide.com


Copyright © 2016 IBPS Guide 37
yoursmahboob.wordpress.com

Q 36. Which of the following will be the step IV of the rearrangement?


a) agree 5 dollar 6 factory 7 inflexible on 8 10 projects 2 9 immovable
b) agree 5 dollar 6 factory 7 immovable 9 inflexible on 8 10 projects 2
c) agree 5 dollar 6 factory 7 immovable 9 inflexible 10 on 8 projects 2
d) agree 5 dollar 6 inflexible on 7 factory 8 10 projects 2 9 immovable
e) None of these

Q 37. Which of the following will be the last step of the rearrangement?
a) IV b) VI c) VII d) V e) None of these

Q 38. In step IV, if, in a certain way, ‗immovable‘ is related to ‗factory‘ and ‗9‘ is related to ‗7‘, which of the
following would ‗8‘ be related to, following the same pattern?
a) inflexible b) 9 c) on d) 6 e) None of these

Q 39. Which of the following is second to the right of the one that is 7th from the right end in step IV?
a) inflexible b) 8 c) 9 d) on e) None of these

Q 40. What will be the position of ‗factory‘ in step V?


a) Fifth from the right end b) Ninth from the right end c) Sixth from the left end
d) Sixth from the right end e) None of these

Directions (Q. 41 - 45): Study the following information carefully and answer the given questions.
A word and number arrangement machine when given an input line of words and numbers rearranges them
following a particular rule in each step. The following is an illustration of input and rearrangement.
Input: new 11 bold 22 carve hundred 32 29 45 houses it 38
Step I: 11 22 new bold carve hundred 32 29 45 houses it 38
Step II: it new 11 22 bold carve hundred 32 29 45 houses 38
Step III: 29 32 it new 11 22 bold carve hundred 45 houses 38
Step IV: bold carve 29 32 it new 11 22 hundred 45 houses 38
Step V: 38 45 bold carve 29 32 it new 11 22 hundred houses
Step VI: houses hundred 38 45 bold carve 29 32 it new 11 22
Step VI is the last step of the above input, as the desired arrangement is obtained.
As per the rules followed above find the appropriate step for the given input
Input: ice money 21 13 good 18 12 qualify 35 eligible 41 browse candidates 10

Q 41. Which step will be the last but one?


a) IX b) VI c) V d) VII e) None of these

Q 42. Which of the following represents the position of ‗ice‘ in Step VI?
a) Third from the left b) Fifth from the right c) Sixth from the right
d) Fourth from the left e) None of these

Q 43. Which word/number would be at the 5th position from the right in Step V?
a) ice b) qualify c) 10 d) 12 e) money

www.ibpsguide.com | www.ibpsguide.in | mock.ibpsguide.in | www.sscexamguide.com


Copyright © 2016 IBPS Guide 38
yoursmahboob.wordpress.com

Q 44. How many elements (words or numbers) are there between ‗21‘ and ‗12‘ in Step VII?
a) Eight b) Five c) Three d) Six e) None of these

Q 45. How many steps will be required to complete the arrangement?


a) VI b) VII c) VIII d) X e) IX

Directions (Q. 46–50): Study the following information carefully and answer the given questions.
A word and number arrangement machine, when given an input line of words and numbers, rearranges them
following a particular rule. The following is an illustration of the Input and its rearrangement.
Input: treat 96 23 unable 48 brown 37 own 62 art
Step I: 23 treat 96 unable 48 brown 37 own 62 art
Step II: 23 37 treat 96 unable 48 brown own 62 art
Step III: 23 37 96 treat unable 48 brown own 62 art
Step IV: 23 37 96 62 treat unable 48 brown own art
Step V: 23 37 96 62 48 treat unable brown own art
Step VI: 23 37 96 62 48 unable treat brown own art
Step VII: 23 37 96 62 48 unable own treat brown art
Step VIII: 23 37 96 62 48 unable own art treat brown
Step IX: 23 37 96 62 48 unable own art brown treat
Step IX is the last step of the rearrangement.
As per the rules followed in the above steps, find out in each of the following questions the appropriate step for
the input given below.
Input: ball 24 indicate 17 51 42 success hold 68 33 easy

Q 46. Which of the following would be the second-last step of the rearrangement ?
a) 17 33 51 42 68 24 easy indicate hold success ball
b) 17 33 51 24 42 68 indicate easy ball hold success
c) 17 33 51 68 42 24 indicate easy ball success hold
d) 17 33 51 68 42 24 easy indicate ball success hold
e) None of these

Q 47. Which of the following would be the last step of the arrangement?
a) VII b) VIII c) IX d) X e) None of these

Q 48. In step IV, which of the following numbers/words would be at 7th position from the left?
a) 24 b) ball c) 42 d) indicate e) None of these

Q 49. Which step number would be the following output?


17 33 51 68 42 24 indicate ball success hold easy
a) VI b) VII c) V d) VIII e) There will be no such step.

Q 50. In step VI of the rearrangement, if ‗68‘ is related to ‗success‘ in a certain way, which of the following
would ‗42‘ be related to, following the same pattern?
a) indicate b) 51 c) success d) hold e) None of these
www.ibpsguide.com | www.ibpsguide.in | mock.ibpsguide.in | www.sscexamguide.com
Copyright © 2016 IBPS Guide 39
yoursmahboob.wordpress.com

INPUT-OUTPUT (Answers with Detailed Explanations)

Q (1-5):
The words are rearranged in increasing order of their length and in case of a tie, they are arranged according to
the dictionary, from left to right. Numbers are rearranged in descending order from right to left. Each step
arranges a word and a number.
Input: 23 technology 52 number 94 extra 31 playing 77 66 fabricate torcher
Step I. extra 23 technology 52 number 31 playing 77 66 fabricate torcher 94
Step II. extra number 23 technology 52 31 playing 66 fabricate torcher 77 94
Step III. extra number playing 23 technology 52 31 fabricate torcher 66 77 94
Step IV. extra number playing torcher 23 technology 31 fabricate 52 66 77 94
Step V. extra number playing torcher fabricate 23 technology 31 52 66 77 94
Step VI. extra number playing torcher fabricate technology 23 31 52 66 77 94

1. d; fabricate number technology playing


2. b
3. c
4. c
5. a

Q (6-10):
The machine rearranges the words that are not followed by number according to the number of alphabets in each
word in ascending order. The words which are followed by numbers are placed thereafter in alphabetical order
along with the numbers.
Input: indignity quartz 58 premier 42 customer imagination salary dynamic 15 viewer 36
Step I. salary indignity quartz 58 premier 42 customer imagination dynamic 15 viewer 36
Step II. salary customer indignity quartz 58 premier 42 imagination dynamic 15 viewer 36
Step III. salary customer indignity imagination quartz 58 premier 42 dynamic 15 viewer 36
Step IV. salary customer indignity imagination dynamic 15 quartz 58 premier 42 viewer 36
Step V. salary customer indignity imagination dynamic 15 premier 42 quartz 58 viewer 36

6. b
7. c
8. e
9. c
10. a

Q (11-15):

The machine rearranges the numbers and words in such a manner that the numbers are arranged in each
alternate step in ascending order while words are also arranged in each alternate step in alphabetical order. In each
alternate step two numbers and two words are arranged.
Input: 57 19 professor male 28 correct 36 38 47 female doctor 51 study
www.ibpsguide.com | www.ibpsguide.in | mock.ibpsguide.in | www.sscexamguide.com
Copyright © 2016 IBPS Guide 40
yoursmahboob.wordpress.com

Step I: 19 28 57 professor male correct 36 38 47 female doctor 51 study


Step II: 19 28 correct doctor 57 professor male 36 38 47 female 51 study
Step III: 19 28 correct doctor 36 38 57 professor male 47 female 51 study
Step V: 19 28 correct doctor 36 38 female male 47 51 professor study 57
Step IV: 19 28 correct doctor 36 38 female male 57 professor 47 51 study
Step V: 19 28 correct doctor 36 38 female male 47 51 57 professor study
Step VI: 19 28 correct doctor 36 38 female male 47 51 professor study 57

11. a
12. a
13. d
14. b
15. c

Q (16-20):
Word arrangement machine first arranges words having first letter vowel in alphabetical order, after that words
having first letter consonant will be arranged in alphabetical order. Alternately the numbers are chosen such that -
greatest, lowest, 2nd greatest, 2nd lowest and so on.

16. c
17. b
18. c
19. d
20. b

16. Step II: again 93 she 40 wins 13 olympic 33 the 3


Step III: again 93 olympic she 40 wins 13 33 the 3
Step IV: again 93 olympic 3 she 40 wins 13 33 the
Step V: again 93 olympic 3 she 40 the wins 13 33

17. b

18. Input : election 17 fees open 41 27 delay 15


Step I: election 41 17 fees open 27 delay 15
Step II: election 41 open 17 fees 27 delay 15
Step III: election 41 open 15 17 fees 27 delay
Step IV: election 41 open 15 delay 17 fees 27
Step V: election 41 open 15 delay 27 17 fees
Step VI: election 41 open 15 delay 27 fees 17

19. d
20. b

Q (21-25):

www.ibpsguide.com | www.ibpsguide.in | mock.ibpsguide.in | www.sscexamguide.com


Copyright © 2016 IBPS Guide 41
yoursmahboob.wordpress.com

Words are arranged in reverse alphabetical order and the numbers are arranged in ascending order. One word and
one number is arranged in each step. The arrangement is done from right to left.
Input: 35 quite head clear 50 65 98 slow giant 71 82 19 oliver music
Step I. 35 quite head clear 50 65 98 giant 71 82 oliver music slow 19
Step II. head clear 50 65 98 giant 71 82 oliver music slow 19 quite 35
Step III. head clear 65 98 giant 71 82 music slow 19 quite 35 oliver 50
Step IV. head clear 98 giant 71 82 slow 19 quite 35 oliver 50 music 65
Step V. clear 98 giant 82 slow 19 quite 35 oliver 50 music 65 head 71
Step VI. clear 98 slow 19 quite 35 oliver 50 music 65 head 71 giant 82
Step VII. slow 19 quite 35 oliver 50 music 65 head 71 giant 82 clear 98

21. b
22. c
23. b
24. c
25. e; 19

Q (26-30):
It first arranges one number and one word in each step till all the numbers are arranged in ascending order. The
words that are arranged together with numbers are those words which are preceding the numbers in inputs. Rest
of the words are arranged in reverse alphabetical order.

Input: union magnet 89 chalk 21 absorb liverpool black honest 11 everyday


Step I. 11 honest union magnet 89 chalk 21 absorb liverpool black everyday
Step II. 11 honest 21 chalk union magnet 89 absorb liverpool black everyday
Step III. 11 honest 21 chalk 89 magnet union absorb liverpool black everyday
Step IV. 11 honest 21 chalk 89 magnet union liverpool absorb black everyday
Step V. 11 honest 21 chalk 89 magnet union liverpool everyday absorb black
Step VI. 11 honest 21 chalk 89 magnet union liverpool everyday black absorb

26. b
27. b
28. b
29. c
30. c

Q (31-35):
The machine rearranges one word and one number in each step. The words starting with a vowel are arranged
first in alphabetical order from the left. When this is done, the words starting with a consonant are arranged
in alphabetical order. The numbers are arranged in ascending order from the right end.
Input: collection 53 vitamin 49 33 article 45 application 23 ice 69 encourage best
Step I. application collection 53 vitamin 49 33 article 45 ice 69 encourage best 23
Step II. application article collection 53 vitamin 49 45 ice 69 encourage best 23 33
Step III. application article encourage collection 53 vitamin 49 ice 69 best 23 33 45
Step IV. application article encourage ice collection 53 vitamin 69 best 23 33 45 49
www.ibpsguide.com | www.ibpsguide.in | mock.ibpsguide.in | www.sscexamguide.com
Copyright © 2016 IBPS Guide 42
yoursmahboob.wordpress.com

Step V. application article encourage ice best collection vitamin 69 23 33 45 49 53


Step VI. application article encourage ice best collection vitamin 23 33 45 49 53 69

31. e; 8th from the left or 6th from the right


32. d
33. c
34. a
35. c

Q (36–40):
The machine rearranges one word and one number in each step. The word that comes first in alphabetical order is
placed first and is followed by the number equal to the total number of alphabets in the word.
Input: inflexible on 7 6 factory dollar 5 8 10 agree projects 2 9 immovable
Step I: agree 5 inflexible on 7 6 factory dollar 8 10 projects 2 9 immovable
Step II: agree 5 dollar 6 inflexible on 7 factory 8 10 projects 2 9 immovable
Step III: agree 5 dollar 6 factory 7 inflexible on 8 10 projects 2 9 immovable
Step IV: agree 5 dollar 6 factory 7 immovable 9 inflexible on 8 10 projects 2
Step V: agree 5 dollar 6 factory 7 immovable 9 inflexible 10 on 8 projects 2
Step VI: agree 5 dollar 6 factory 7 immovable 9 inflexible 10 on 2 8 projects
Step VII: agree 5 dollar 6 factory 7 immovable 9 inflexible 10 on 2 projects 8

36. b
37. c
38. a
39. d
40. e

Q (41-45)
The machine arranges words and numbers in the following manner:
Step I. The first two numbers are arranged in ascending order from the left.
Step II. The first two words are arranged according to the number of letters present in the word.
This process follows in each alternate step until all the numbers and words are arranged.

Input: ice money 21 13 good 18 12 qualify 35 eligible 41 browse candidates 10


Step I: 10 12 ice money 21 13 good 18 qualify 35 eligible 41 browse candidates
Step II: ice good 10 12 money 21 13 18 qualify 35 eligible 41 browse candidates
Step III: 13 18 ice good 10 12 money 21 qualify 35 eligible 41 browse candidates
Step IV: money browse 13 18 ice good 10 12 21 qualify 35 eligible 41 candidates
Step V: 21 35 money browse 13 18 ice good 10 12 qualify eligible 41 candidates
Step VI: qualify eligible 21 35 money browse 13 18 ice good 10 12 41 candidates
Step VII: 41 qualify eligible 21 35 money browse 13 18 ice good 10 12 candidates
Step VIII: candidates 41 qualify eligible 21 35 money browse 13 18 ice good 10 12

41. d
42. c
www.ibpsguide.com | www.ibpsguide.in | mock.ibpsguide.in | www.sscexamguide.com
Copyright © 2016 IBPS Guide 43
yoursmahboob.wordpress.com

43. d
44. a
45. c

Q (46–50):
The arrangement machine rearranges one word/ number in each step. It rearranges odd numbers first in ascending
order and then even numbers in descending order. It rearranges words starting with vowels in descending order
and finally words starting with consonants in ascending order.
Input: ball 24 indicate 17 51 42 success hold 68 33 easy
Step I: 17 ball 24 indicates 51 42 success hold 68 33 easy
Step II: 17 33 ball 24 indicate 51 42 success hold 68 easy
Step III: 17 33 51 ball 24 indicate 42 success hold 68 easy
Step IV: 17 33 51 68 ball 24 indicate 42 success hold easy
Step V: 17 33 51 68 42 ball 24 indicate success hold easy
Step VI: 17 33 51 68 42 24 ball indicate success hold easy
Step VII: 17 33 51 68 42 24 indicate ball success hold easy
Step VIII: 17 33 51 68 42 24 indicate easy ball success hold
Step IX: 17 33 51 68 42 24 indicate easy ball hold success

46. c
47. c
48. d; Step IV: 17 33 51 68 ball 24 indicate 42 success hold easy
49. b
50. d; Step VI: 17 33 51 68 42 24 ball indicate success hold easy

5. ALPHANUMERIC

Directions (1-5): These questions are based on the following letter / number/ symbol arrangement. Study it
carefully and answer the questions that follow

RDAK5VI2MJEN97UZV1W3H4FY8P6TG

Q 1. Which element will be eighth to the left of seventeenth from the left end?
a) M b) J c) 8 d) 5 e) None of these

Q 2. How many such numbers are there in the above arrangement each of which is immediately preceded by a
consonant but not immediately followed by a vowel?
a) None b) One c) Two d) Three e) More than three

Q 3. Four of the following are alike in a certain way based on their positions in the above arrangement and so
form a group. Which is the one that does not belong to that group?
a) E9J b) Z1U c) HF3 d) Y48 e) V25

Q 4. Which element will be sixth to the right of nineteenth from the right end?
www.ibpsguide.com | www.ibpsguide.in | mock.ibpsguide.in | www.sscexamguide.com
Copyright © 2016 IBPS Guide 44
yoursmahboob.wordpress.com

a) 5 b) Z c) V d) 1 e) None of these

Q 5. Which element will be tenth to the right of fourteenth from the left end?
a) F b) Y c) 8 d) P e) None of these

Directions (6-10): These questions are based on the following letter / number/ symbol arrangement. Study it
carefully and answer the questions that follow
24856&1 3 @ 6 4 5 2 # 9 7 1©3 €

Q 6. How many pairs of numbers are there in the series highlighted in bold in the above arrangement each of
which has as many numbers between them (in both forward and backward directions) as they have between them
in the numerical series?
a) One b) Two c) Three d) Four e) Five

Q 7. Which of the following digit / symbol is second to the right of the tenth from the left end?
a) @ b) 4 c) 3 d) 5 e) 2

Q 8. If all the symbols are dropped from the above arrangement, which of the following will be the twelfth from
the right end of the above arrangement?
a) 2 b) 5 c) 3 d) 7 e) None of these

Q 9. How many symbols are there in the above arrangement, each of which is immediately followed by a perfect
square? (One is also a perfect square)
a) One b) Two c) Three d) Four e) Five

Q 10. How many perfect squares are there in the above arrangement, each of which is immediately preceded by
an even number? (One is also a perfect square)
a) None b) One c) Two d) Three e) More than three

Directions (11-15): These questions are based on the following letter / number/ symbol arrangement. Study
it carefully and answer the questions that follow

B2@#K8Q7%UTI*V9$MDP14F©AE3

Q 11. If all the numbers in the above arrangement are summed up and then divided by 2, then what will be the
result?
a) 9 b) 16 c) 18 d) 20 e) None of these

Q 12. How many such vowels are there in the above arrangement each of which is immediately preceded by a
symbol and immediately followed by a letter?
a) One b) Two c) Three d) None e) None of these

Q 13. What should come next in the following series based on the above arrangement?
2@K 7%T V9M ?
a) M D 1 b) 1 4 © c) F © E d) P 1 F e) None of these
www.ibpsguide.com | www.ibpsguide.in | mock.ibpsguide.in | www.sscexamguide.com
Copyright © 2016 IBPS Guide 45
yoursmahboob.wordpress.com

Q 14. If all the symbols in the above arrangement are dropped, which of the following will be the tenth from the
left end?
a) M b) I c) V d) 9 e) None of these

Q 15. Which of the following is the seventh to the left of the eleventh from the right end of above arrangement?
a) % b) D c) $ d) 7 e) None of these

Directions (16-20): These questions are based on the following letter / number/ symbol arrangement. Study it
carefully and answer the questions that follow

Z 3*5AB1$GD94UE!C6Q&2#IJY

Q 16. Which of the following is fifth to the right of the nineteenth from the left end of the above arrangement?
a) # b) Z c) Y d) 5 e) None of these

Q 17. How many such consonants are there in the above arrangement each of which is immediately preceded by a
symbol and immediately followed by a letter?
a) None b) One c) Two d) Three e) None of these

Q 18. How many such symbols are there in the above arrangement each of which is immediately preceded by a
letter and immediately followed by a number?
a) None b) Two c) Three d) One e) None of these

Q 19. How many such numbers are there in the above arrangement each of which is immediately followed by a
vowel?
a) One b) Two c) Three d) Four e) None of these

Q 20. If all the symbols are dropped from the above arrangement, which of the following will be the tenth from
the right end?
a) 4 b) C c) E d) 9 e) None of these

Directions (21-25): These questions are based on the following letter / number/ symbol arrangement. Study it
carefully and answer the questions that follow:

5DGE«79$F16R%LIAJ3B#4@KP8UM2

Q 21. How many such numbers are there in the above arrangement each of which is immediately followed by a
consonant but not immediately preceded by a vowel?
a) None b) One c) Two d) Three e) More than three

Q 22. Four of the following five are alike in a certain way on the basis of their positions in the above arrangement
and so form a group. Which is the one that does not belong to the group?
a) $9F b) R6% c) 8PU d) #B4 e) 3BJ

www.ibpsguide.com | www.ibpsguide.in | mock.ibpsguide.in | www.sscexamguide.com


Copyright © 2016 IBPS Guide 46
yoursmahboob.wordpress.com

Q 23. If all the consonants are removed from the above arrangement which element will be fifth to the left of
seventh from the right?
a) 6 b) $ c) % d) 1 e) None of these

Q 24. If all the symbols are removed from the above arrangement which element will be third to the left of
thirteenth from the left?
a) L b) R c) 6 d) % e) None of these

Q 25. Which element will be fifth to the right of ninth from the right end if all the numbers are removed from the
above arrangement?
a) K b) @ c) P d) # e) None of these

Directions (Q. 26-30): In a certain instruction system the different computation processes are written as follows:
(i) a $ b «c means c is subtracted from the product of a and b.
(ii) a @ b # c means a is multiplied by the sum of b and c.
(iii) a l b e c means c is subtracted from b and the resultant is added to a.
(iv) a © b % c means b is divided by c and the resultant is added to square of a.
In each of the following questions, a set of instruction sequence is given. You are required to find out the
outcome which should come in place of the question mark (?) in each of the given sets of sequence.

Q 26. 17 $ 4 « 8 = t
7 © t % 15 = ?
a) 11 b) 53 c) 13 d) 51 e) None of these

Q 27. 13 @ 4 # 3 = p
p $ 5 « 55 = ?
a) 45 b) 75 c) 340 d) 400 e) None of these

Q 28. b $ 15 « 18 = 42
b © 36 % 9 = ?
a) 12 b) 16 c) 20 d) 18 e) None of these

Q 29. 5 © 49 % 7 = a
a l 87 e 29 = ?
a) 118 b) 108 c) 98 d) 80 e) None of these

Q 30. m l 78 e 56 = 50
m @ 7 # 13 = ?
a) 560 b) 56 c) 280 d) Cannot be determined e) None of these

Directions (31-35): These questions are based on the following letter / number/ symbol arrangement. Study it
carefully and answer the questions that follow

LF3#RN8A@Y4M©WP6HU9IK2E
www.ibpsguide.com | www.ibpsguide.in | mock.ibpsguide.in | www.sscexamguide.com
Copyright © 2016 IBPS Guide 47
yoursmahboob.wordpress.com

Q 31. If all numbers are dropped from the above arrangement, which of the following will be the fourteenth from
the left end?
a) I b) P c) W d) U e) None of these

Q 32. How many such numbers are there in the above arrangement each of which is immediately preceded by a
vowel?
a) None b) One c) Two d) Three e) None of these

Q 33. How many such consonants are there in the above arrangement each of which is immediately preceded by a
symbol and immediately followed by a letter?
a) None b) One c) Two d) Three e) None of these

Q 34. If it is possible to make a meaningful word with the fifth, sixth, eighth, twelfth, twentieth and twenty-third
letter of the above arrangement, which of the following will be the fourth letter from the left?
a) E b) I c) N d) R e) Cannot be determined

Q 35. Which of the following is sixth to the right of the nineteenth from the right end of the above arrangement?
a) Y b) M c) 4 d) @ e) None of these

Directions (36-40): These questions are based on the following letter / number/ symbol arrangement. Study it
carefully and answer the questions that follow

B#AR58E%MF4J1U@H2©9TI6*W3P#K7$YS
Q 36. If all the numbers in the above arrangement are dropped, which of the following will be the eleventh from
the right end?
a) U b) T c) F d) H e) None of these

Q 37. Four of the following are alike in a certain way based on their positions in the above arrangement and so
form a group. Which is the one that does not belong to that group?
a) 1 @ 4 b) © T H c) W P 6 d) # 7 3 e) 9 2 I

Q 38. Which of the following is the twelfth to the left of the twentieth from the left end of the above
arrangement?
a) % b) W c) $ d) J e) None of these

Q 39. Which of the following is the fifteenth to the left of the seventh from the right end of the above
arrangement?
a) J b) 4 c) F d) 1 e) M

Q 40. How many such numbers are there in the above arrangement each of which is immediately preceded by a
consonant and also immediately followed by a symbol?
a) None b) One c) Two d) Three e) More than three

www.ibpsguide.com | www.ibpsguide.in | mock.ibpsguide.in | www.sscexamguide.com


Copyright © 2016 IBPS Guide 48
yoursmahboob.wordpress.com

Directions (41-45): These questions are based on the following letter / number/ symbol arrangement. Study it
carefully and answer the questions that follow:

W1R%4JE#7MTUI9BH3A$9FQ5DG6USP

Q 41. Four of the following are alike in a certain way based on their positions in the above arrangement and so
form a group. Which is the one that does not belong to that group?
a) RW4 b) 5FG c) 9QA d) 3B$ e) 7ET

Q 42. Which element will be eighth to the left of twentieth from the right end?
a) W b) R c) % d) 4 e) None of these

Q 43. Which element will be seventh to the right of eighteenth from the right end?
a) A b) E c) $ d) # e) None of these

Q 44. If the order of the last 15 elements is reversed which of the following will be fifth to the right of twelfth
from the left end?
a) U b) $ c) 3 d) 6 e) None of these

Q 45. If the positions of E and A are interchanged and similarly the positions of R and U are interchanged then
how many symbols will be there each of which is either preceded or followed by a vowel?
a) None b) One c) Two d) Three e) More than three

Directions (46-50): These questions are based on the following letter / number/ symbol arrangement. Study it
carefully and answer the questions that follow

!9BQ=$25RJ#L3@YME68*+DF4%H7&

Q 46. How many such numbers are there in the above arrangement each of which is immediately followed by a
number but not immediately preceded by a symbol?
a) One b) Two c) Three d) Four e) None

Q 47. Four of the following are alike in a certain way based on their positions in the above arrangement and so
form a group. Which is the one that does not belong to that group?
a) =5$ b) #@L c) %7H d) 8D* e) !Q9

Q 48. How many such numbers are there in the above arrangement each of which is immediately preceded by a
symbol but not immediately followed by a letter?
a) None ' b) One c) Two d) Three e) More than three

Q 49. Which element will be thirteenth to the right of eleventh from the left end?
a) F b) % c) H d) 4 e) D

Q 50.1f all the symbols are dropped from the above arrangement which of the following will be the sixth from the
right end?
www.ibpsguide.com | www.ibpsguide.in | mock.ibpsguide.in | www.sscexamguide.com
Copyright © 2016 IBPS Guide 49
yoursmahboob.wordpress.com

a) R b) 8 c) 5 d) 6 e) None of these

ALPHANUMERIC (Answers with Explanation)

1. a) Eighth to the left of seventeenth from the left end = 17 – 8 = 9th from left end = M

2. e)
K5V
N97
V1W
W3H
H 4 F
Y 8 P
P 6 T

3. d)
E + 2 =9 ; E – 1 =J
Z+2=1;Z–1=U
H+2=F;H–1=3
Y–2=4;Y+1=8
V + 2 =2 ; V – 1 = 5
So Y 4 8 does not belong to that group

4. c) Sixth to the right of nineteenth from the right end = 19 – 6 = 13th from right end = V

5. b)
Tenth to the right of fourteenth from the left end = 10 + 14 = 24th from left end = Y

6.d) Required pairs


Hence, such pairs are 2 to 5, 2 to 6, 5 to 6, and 6 to 8.

7. d) Second digit/symbol to the right of tenth from the left end means 12th digit symbol from the left end means
5.

8. b) After dropping all the symbols, new arrangement is as,


2 48 5 6 1 3 6 4 5 2 9 7 1 3
Hence, Twelfth from right end → 5

9. b) There are two such symbols i.e.,& 1 and#9.

10. c) There are two such perfect square i.e., 24 and 64.
11. e) 2 + 8 + 7 + 9 +1+ 4 + 3/2= 34/2 = 17

12. b) %UT, ©AE

www.ibpsguide.com | www.ibpsguide.in | mock.ibpsguide.in | www.sscexamguide.com


Copyright © 2016 IBPS Guide 50
yoursmahboob.wordpress.com

13. b) 2→+1=@→+2=K→+3=7
7→+1=%→+2=T→+3=V
V→+1=9→+2=M→+3=1
1→+1=4→+2=©

14. c) New arrangement:


B2K8Q7UTIV9MDP14FAE3
Hence, tenth letter from the left is V.

15. a) Seventh to the left of eleventh from the right end = (11 + 7 =) 18th from the right end, ie %.

16. c) Fifth to the right of nineteenth from the left = (19 + 5=) 24th from the left, ie Y.

17. b) $GD

18. d) Q & 2

19. b) 5A, 4U

20. a) After rearrangement,


Z 3 5 A B 1 G D 9 4 U E C 6 Q 2 I J Y.
Hence tenth from the right end is 4.

21. c) 1 6 R and J 3 B

22. e) $9F this symbols and numbers are arranged in preceded by and followed by manner
So ―3BJ‖ is not arranged in the above manner

23. d) New arrangement


5E«79$16%IA3#4@8U2
Fifth to the left of seventh from the right = 5 + 7 = 12th from right = 1

24. b)
5DGE79F16RLIAJ3B4KP8UM2
Third to the left of thirteenth from the left = 13 – 3 = 10th from the left = R

25. a)
DGE«$FR%LIAJB#@KPUM
Fifth to the right of ninth from the right end = 9 – 5 = 4th from right end = K

26. b) 17 $ 4 « 8
(17 × 4) - 8 [Use instruction (iii)]
= 60
Hence t = 60
Now, we have
www.ibpsguide.com | www.ibpsguide.in | mock.ibpsguide.in | www.sscexamguide.com
Copyright © 2016 IBPS Guide 51
yoursmahboob.wordpress.com

7 © 60 % 15
(60 ÷ 15) + 72 [Use instruction (ii)]
= 53
? = 53

27. d) 13 @ 4 # 3
13 × (4 + 3) [Use instruction (i)]
= 91
Hence p = 91
Now, we have
91 $ 5 « 55
(91 × 5) - 55 [Use instruction (iii)]
= 400
?= 400

28. c) b $ 15 « 18 = 42
(b × 15) - 18 = 42 [Use instruction (iii)]
(b × 15) = 42 + 18
b=4
Now, we have
4 © 36 % 9
(36 ÷9) + 42 [Use instruction (ii)]
= 20
? = 20

29. e) 5 © 49 % 7
(49 ÷ 7) + 52 [Use instruction (ii)]
= 32
Hence a = 32
Now, we have
32 l 87 e 29
(87 - 29) + 32 [Use instruction (iv)]
= 58 + 32 = 90
?= 90

30. a) m l 78 e 56 = 50
(78 - 56) + m = 50 [Use instruction (iv)]
m = 50 - 22 = 28
Now, we have
28 @ 7 # 13
28 × (7 + 13) [Use instruction (i)]
= 560
? = 560

31. d) L F # R N A @ Y M © W P H U I K E
www.ibpsguide.com | www.ibpsguide.in | mock.ibpsguide.in | www.sscexamguide.com
Copyright © 2016 IBPS Guide 52
yoursmahboob.wordpress.com

32. b) U9

33. c) #RN, ©WP

34. e) Fifth letter → R


Sixth letter → N
Eighth letter → A
Twelfth letter → M
Twentieth letter → I
Twenty-third letter → E

35. c) Sixth to the right of the 19th from the right end, that is (19 – 6 =) 13th from the right, ie 4.

36. e) B # A R E % M F J U @ H © T I * W P # K $ Y S
11th from right end is ©
37.e) All values are
1+2=@;1–2=4
©+2=T;©-2=H
W+2=P;W–2=6
#+2=7;#-2=3
9–2=2;9+2=I
So 9 2 I is not belong to that group.

38. a) Twelfth to the left of the twentieth from the left end = 22-12 = 10th from left end = %

39.b) Fifteenth to the left of the seventh from the right end = 15 + 7 = 22nd from right end = 4

40. c) H 2 © and K 7 $

41. c)
R–2=W;R+2=4
5–2=F;5+2=G
9+2=Q;9-2=A
3–2=B;3+2=$
7–2=E;7+2=T
So 9QA is not belong to that group

42. e)
Eighth to the left of twentieth from the right end = 8 + 20 = 28th from right end = 1

43. c) Seventh to the right of eighteenth from the right end = 18 – 7 = 11th from right end = $

44. a) New arrangement is


W1R%4JE#7MTUI9PSU6GD5QF9$A3HB
www.ibpsguide.com | www.ibpsguide.in | mock.ibpsguide.in | www.sscexamguide.com
Copyright © 2016 IBPS Guide 53
yoursmahboob.wordpress.com

Fifth to the right of twelfth from the left end = 5 + 12 = 17th from left end = U

45. d) New arrangement


W1U%4JA#7MTRI9BH3E$9FQ5DG6RSP
―U %‖, ―A #‖ and ―E $‖

46. a) ―E 6 8‖

47. c)
=+3=5;=+1=$
#+3=@;#+1=L
% + 2 =7 ; % + 1 = H
8+3=D;8+1=*
!+3=Q;!+1=9
So ―%7H‖ is not belong to that group

48. b) ― $ 2 5 ―

49. d) Thirteenth to the right of eleventh from the left end = 13 + 11 = 24th from left end = 4

50. b) New arrangement


9BQ=25RJL3YME68DF4H7
Sixth from right end = 8

6. DIRECTION-RANKING

Direction
1). Rahane walks 10 metres in front and 10 metres to the right. Then every time turning to his left, he walks 5, 15
and 15 metres respectively. How far is he now from his starting point?
a) 10 metres b) 15 metres c) 5 metres d) 12 metres e) 7 metres

2). From her home, Jayanthi wishes to go to temple. From home, she goes towards North and then turns left and
then turns right, and finally she turns left and reaches temple. In which direction her temple is situated with
respect to her home?
a) North-East b) North-West c) South-East d) South-West e) None of these

3). If all the directions are rotated, i.e., if East is changed to North and North to West and so on, then what will
come in place of North-West ?
a) East-west b) East-north c) North-east d) South-west e) None of these

4). A is to the East of X, which is to the North of Z. If P is to the South of Z, then P is in which direction with
respect to A?
a) South-East b) North c) South d) Cannot be determined e) None of these
www.ibpsguide.com | www.ibpsguide.in | mock.ibpsguide.in | www.sscexamguide.com
Copyright © 2016 IBPS Guide 54
yoursmahboob.wordpress.com

5). From her house Archana walked 50 m towards North. Archana turned right and walked 30 m. Again Archana
went ten m to south How far Archana from her house?
a) 10 km b) 8 km c) 3 km d) 5 km e) 4 km

Directions (Q. 6-7): Study the following information carefully and answer the given questions.
Orange is 50m towards the west of Red. Blue is 20m towards the north of Red. Green is 30m towards the east of
Blue. White is 20m towards the south of Green.

6). If a person walks 20 m towards the north from Orange, takes a right turn and continues to walk, which of the
following points would he reach the first?
a) Green b) Red c) White d) Blue e) None of these

7). Which of the following points are in a straight line?


a) Orange, White, Blue b) Blue, White, Green c) Green, Blue, Orange d) Red, Green, Orange
e) Orange, Red, White

8). Amir is to the East of Kalam, Giri is to the South of Kalam and Sam is to the West of Giri. Amir is in which
direction with respect to Sam?
a) South-East b) North-East c) North-West d) South-West e) None of these

9). Four players Dhoni, Rohit, Kohli and Dhawan are standing a play filed in such a way that Rohit is to East of
Dhoni, Kohli is to the South of Dhoni and Dhawan is to the North of Dhoni. In which direction of Rohit is
Dhawan Standing?
a) North – East b) South c) South-East d) North-West e) North

10). Prakash's house is to the right of Vikram's house at a distance of 20 metres in the same row facing north.
Mohan's house is in the North- East direction of Vikram's house at a distance of 25 metres. Determine that
Prakash's house is in which direction with respect of Mohan's house?
a) East b) South c) North-East d) West e) None of these

11). Five persons Milan, Milap, Milun, Mirza and Misal are standing in a row. Milap is between Milan and Milun
and Mirza is between Milun and Misal. If the distance of Milun from Milap is equal to the distance of Mirza from
Milun, what is the relation between the distances of Milan to Milap and Milap to Misal?
a) There is no relation in Milan, Milap and Milap, Misal
b) Both are equal
c) Milan, Milap is larger than Milap, Misal
d) Milan, Milap is smaller than Milap, Misal
e) None of these

www.ibpsguide.com | www.ibpsguide.in | mock.ibpsguide.in | www.sscexamguide.com


Copyright © 2016 IBPS Guide 55
yoursmahboob.wordpress.com

12). From his house, Gokul went 15 km to the North. Then he turned West and covered 10 km. Then, he turned
South and covered 5 km. Finally, turning to East, he covered 10 km. In which direction is he from his house?
a) West b) East c) North d) South e) None of these

13). Arun goes 300 m to North and then turning to East he goes 400 m. Again he turns to his right and goes 200
m. After this he turns to his right and goes 400 m. How far is he from his starting point?
a) 0 m b) 100 m c) 250 m d) 400 m e) 200 m

Directions (Q. 14-16): Study the following information carefully and answer the given questions.
Seven States Goa, Bihar, Kerala, Manipur, Karnataka, Assam and Rajasthan are situated as follows:
Karnataka is 2 km to the west of Bihar.
Assam is 2 km to the north of Goa.
Kerala is 1 km to the west of Goa.
Manipur is 2 km to the south of Rajasthan.
Rajasthan is 2 km to the east of Kerala.
Manipur is exactly in the middle of Bihar and Karnataka.

14). Goa is in the middle of


a) Karnataka and Kerala b) Karnataka and Rajasthan c) Assam and Rajasthan d) Rajasthan and Kerala
e) Bihar and Assam

15). Which two states are the farthest from one another?
a) Manipur and Kerala b) Assam and Karnataka c) Assam and Bihar d) Rajasthan and Karnataka
e) Assam and Kerala

16). How far is Karnataka from Assam (in km)?


a) 9 B) √30 c) 8 d) √18 e) 4

17). Prasath is travelling by Cycle. He travels 20 km and then turns right and travels 10 km, then he takes left turn
and travels 5 km and finally he takes right turn. If now, he is facing West, from which direction did he started?
a) North West b) West c) North d) East e) South

18). Tanav and Tanay are standing at a distance of 20 km from each other on a straight East-West road. Tanav
and Tanay start walking simultaneously, eastwards and westwards respectively, and both cover a distance of 5
km. Then Tanav turns to his left and walks 10 km. ‗Tanay‘ turns to his right and walks 10 km and at the same
speed. Then both turn to their left and cover a distance of 5 km at the same speed. What will be the distance
between them?
a) 10 km b) 5 km c) 20 km d) 25 km e) 15 km
www.ibpsguide.com | www.ibpsguide.in | mock.ibpsguide.in | www.sscexamguide.com
Copyright © 2016 IBPS Guide 56
yoursmahboob.wordpress.com

19). One day, Balaji left home and walked 10 km southwards, turned right and cycled 5 km and turned right and
cycled 10 km and turned left and cycled 10 km. How many kilometres will he have to walk to reach his home
straight?
a) 22 km b) 5 km c) 30 km d) 25 km e) 15 km

20). The Bank is in the East of the Temple while my house is in the South of the Temple. The Airport is in the
North of the post office. If the distance of the Airport from the post-office is equal to the distance of my house
from the Temple, in which direction is the Airport with respect to my house?
a) East b) North c) North-east d) North West e) South-west

21). Of the five fruits Apple, Banana, Grapes, Pears and Mango situated close to each other, Apple is to west of
Banana, Grapes is to the south of Apple, Mango is to the north of Banana, and Pears is to the east of Mango.
Then, Grapes is in which direction with respect to Pears?
a) North-West b) South-East c) South-West d) Data Inadequate e) None of these

22). Nitish walked 30 km towards east and took a right turn and walked 40 km. He again took a right turn and
walked 50 km. Towards which direction is he from his starting point?
a) South-East b) West c) South d) South-West e) None of these

23). Raman walks 20 m North. Then, he turns right and walks 30 m. Then he turns right and walks 35 m. Then he
turns left and walks 15 m. Then he again turns left and walks 15 m. In which direction and how many metres
away is he from his original position?
a) 25 metres West b) 20 metres East c) 15 metres West d) 20 metres West e) 45 metres East

24). Krishna walked from A to B in North, 100 m. Then, she turned to the left and walked 30 m. Again, she
turned to left and walked 140 m. Now, how far is she from the starting point?
a) 20 m b) 50 m c) 10 m d) 40 m e) None of these

25). Murali walks 10 km towards North. From there he walks 6 km towards South. Then, he walks 3 km towards
East. How far and in which direction is he with reference to his starting point?
a) 3 km North -West b) 9 km South – West c) 6 km North - East d) 5 km North - East
e) Cannot be determined

Answers:
1. c 2. b 3. d 4. e 5. d 6. b 7. e 8. b 9. d 10. b 11. d 12. c 13. b 14. d 15. c 16. e 17. c 18. a 19. e 20.c 21. c
22. d 23. e 24. b 25. d

DIRECTION (Explanations)

www.ibpsguide.com | www.ibpsguide.in | mock.ibpsguide.in | www.sscexamguide.com


Copyright © 2016 IBPS Guide 57
yoursmahboob.wordpress.com

The movements of Rahane are as shown in Diagram


(A to B, B to C, C to D, D to E and E to F)
Since FE = AB + CD, so F lies in line with A.
Rahane's distance from the starting point A
= AF = (DE – BC)
= (15 – 10) m
= 5 m.

It is clear from the diagram that temple is in North-west direction with respect to home.

South-west

www.ibpsguide.com | www.ibpsguide.in | mock.ibpsguide.in | www.sscexamguide.com


Copyright © 2016 IBPS Guide 58
yoursmahboob.wordpress.com

South West

J R = √42 + 32 = √16 + 9 = √25 = 5 km

6. Red
7. Orange, Red, White

www.ibpsguide.com | www.ibpsguide.in | mock.ibpsguide.in | www.sscexamguide.com


Copyright © 2016 IBPS Guide 59
yoursmahboob.wordpress.com

Amir is to the North-East of Sam

Hence, Rohit is in North - West direction of Dhawan

Prakash's house is in South direction with respect of Mohan's house

The position of all the five persons is as follows:


Hence Milan, Milap is smaller than Milap, Misal

www.ibpsguide.com | www.ibpsguide.in | mock.ibpsguide.in | www.sscexamguide.com


Copyright © 2016 IBPS Guide 60
yoursmahboob.wordpress.com

The movements of Gokul are as shown in diagram.


(P to Q, Q to R, R to S, S to T)
Clearly, his final position is T which is to the North of his house P.

Distance from starting point = MO – NO = 300 – 200 = 100 m

14). Clearly, Goa is in the middle of Rajasthan and Kerala


www.ibpsguide.com | www.ibpsguide.in | mock.ibpsguide.in | www.sscexamguide.com
Copyright © 2016 IBPS Guide 61
yoursmahboob.wordpress.com

15). Clearly, Assam and Bihar are the farthest from one another.
16). Required distance = Assam, Karnataka = Assam, Goa + Goa, Karnataka = Assam, Goa + Rajasthan, Manipur
= (2 + 2) KM = 4 km

North

10 km

Here, Balaji starts from home at P, moves 10 km southwards up to Q, turns right and moves 10 km up to R, turns
right again and moves 10 km up to S and finally turns left and moves 10 km up to T.
Thus, his distance from initial position P = PT
= PS + ST
= QR + ST = (5 + 10) km = 15 km.

www.ibpsguide.com | www.ibpsguide.in | mock.ibpsguide.in | www.sscexamguide.com


Copyright © 2016 IBPS Guide 62
yoursmahboob.wordpress.com

The positions of Temple, house, Bank and Airport are as follows:


Hence the Air port is in the North-east of my house.

Hence, Grapes is to the South-West with respect to Pears

South – West

www.ibpsguide.com | www.ibpsguide.in | mock.ibpsguide.in | www.sscexamguide.com


Copyright © 2016 IBPS Guide 63
yoursmahboob.wordpress.com

The movements of Raman from M to R are as shown in figure.


Since OP = MN + QR, so R lies in line with M.
Raman‘s distance from original position M = MR
= (MS + SR) = (NO + PQ) = (30 + 15) m = 45m.
Also, R lies to the east of M.

Required distance (AE) = Square Root of 302 + 402 = 50m

Clearly, Murali moves from P 10 km north-wards up to Q, then moves 6 km southwards up to R, turns towards
East and walks 3 km up to S.
Then, PR = (PQ – QR) = (10 – 6) = 4 km; RS = 3 km.

www.ibpsguide.com | www.ibpsguide.in | mock.ibpsguide.in | www.sscexamguide.com


Copyright © 2016 IBPS Guide 64
yoursmahboob.wordpress.com

So, Murali's distance from starting point P


=PS = √𝑃𝑅 2 + 𝑅𝑆 2 = √42 + 32 = 5km.
Also, S is to the North-east of P.

RANKING

1). Among Dhanvi, Dhanya, Dhithi, Dhriti and Dhuthi, Dhriti is older than Dhithi but not as old as Dhuthi;
Dhanya is older than only Dhanvi. Who among them is the youngest?
a) Dhanvi b) Dhanya c) Dhithi d) Cannot be determined e) None of these

2). In a row of thirty-seven boys facing South, Arafath is eighth to the right of Mishra who is fourteenth to the left
of David. How many boys are there between David and Arafath in the row?
a) 4 b) 6 c) 8 d) Data inadequate e) None of these

3). Among Kaushika, Kaveri, Kavini, Kavita and Kavya each having a different weight, Kavita is heavier than
Kaushika and Kavya and Kaveri is lighter than Kavini. Who among them is the heaviest?
a) Kavita b) Kaveri c) Kavini d) Data inadequate e) None of these

4). Priya is 18th from the left end and Saranya is 11th from the right end of a row of 40 children. How many
children are there between Priya and Saranya in the row?
a) 10 b) 9 c) 12 d) 11 e) None of these

5). Among Suresh, Dinesh, Ganesh, Ramesh and Lokesh each having a different height, Ganesh is taller than
Suresh and Lokesh but shorter than Ramesh and Dinesh. Suresh is not the shortest. Who among them is the
tallest?
a) Ramesh b) Dinesh c) Suresh d) Suresh or Lokesh e) Data inadequate

6). Nisha is senior to Divya but not to Radha. Latha is junior to Richa. No one is senior to Najima. Who is most
junior?
a) Divya b) Nisha c) Latha d) Data inadequate e) None of these

7). In a queue of children, Hameed is fifth from the left and Aruna is sixth from the right. When they interchange
their places among themselves, Hameed becomes thirteenth from the left. Now what will be Aruna's position
from the right?
a) 4th b) 14th c) 8th d) 15th e) None of these

8). Five students took part in a race. Ramesh finished before Gokul but behind Teja. Lokesh finished before Balaji
but behind Gokul. Who won the race?
a) Ramesh b) Teja c) Gokul d) Lokesh e) None of these

www.ibpsguide.com | www.ibpsguide.in | mock.ibpsguide.in | www.sscexamguide.com


Copyright © 2016 IBPS Guide 65
yoursmahboob.wordpress.com

9). Among Badal, Badri, Balan, Balbir and Baldev each one has secured different marks in an examination. Balbir
secured more marks than Badal and Baldev. Badri secured less marks than Balan. Who among them secured third
highest marks?
a) Badri b) Balbir c) Badal d) Baldev e) Data inadequate

10). Zaheer is 16th from the left end in the row of boys and Munaf is 18th from the right end. Rahaman is 11th
from Zaheer towards the right end and 3rd from Munaf towards the right end. How many boys are there in the
row?
a) Data inadequate b) 42 c) 40 d) 48 e) None of these

11). Among Taral, Taran, Tarik, Tariq and Tarit each having a different weight, Taran is heavier than only Tarik.
Taral is heavier than Taran and Tarit but not as heavy as Tariq. Who is the third heaviest among them?
a) Taral b) Taran c) Tariq d) Tarit e) None of these

12). In a row of thirty five children, Prem is fifteenth from the right end and there are ten children between Prem
and Udaya. What is Udaya's position from the left end of the row?
a) 15th b) 5th c) 30th d) Data inadequate e) None of these

13). Aravind is older than Gandhi. Praveen is younger than Gandhi and Karan. Karan is not as old as Aravind.
Who among Aravind, Karan, Gandhi and Praveen is the oldest?
a) Aravind b) Gandhi c) Aravind or Gandhi d) Data inadequate e) None of these
14. In a row of children facing North, Ganesh is twelfth from the left end. Karthik who is twenty-second from the
right end is fourth to the right of Ganesh. Total how many children are there in the row?
a) 35 b) 36 c) 37 d) 34 e) None of these

15). Ashok is 16th from the top and twelfth from the bottom in merit in the class. How many students are there in
the class?
a) 29 b) 28 c) 27 d) Cannot be determined e) None of these

16). Among five friends Naresh is taller than Natesh but not Namish. Namit is taller than Namish but not Nakul.
If they stand in increasing order of their heights, who will be first in line ?
a) Nakul b) Namish c) Natesh d) Data inadequate e) None of these

17). In a row of girls, Soundarya is 15th from the left and Hema is 18th from the right. If they inter-change their
places, Hema becomes 15th from the left. How many girls are there in the row?
a) 33 b) 48 c) 47 d) Data inadequate e) None of these

www.ibpsguide.com | www.ibpsguide.in | mock.ibpsguide.in | www.sscexamguide.com


Copyright © 2016 IBPS Guide 66
yoursmahboob.wordpress.com

18). Five women Nakula, Najma, Nalina, Nalini and Nalika read a newspaper. The one who reads first gives it to
Nalina. The one who reads last had taken it from Nakula. Nalika was not the first or the last to read. There were
two readers between Najma and Nakula. Najma passed the newspaper to whom?
a) Nakula b) Nalina c) Nalini d) Nalika e) None of these

19). In a row of 40 students facing North, Prakash is 6th to the left of Gopinath. If Gopinath is 30th from the left
end of the row, how far is Prakash from the right end of the row?
a) 17th b) 16th c) 15th d) 26th e) None of these

20). Among Hriday, Hridik, Hriman, Hrithik and Hritish, Hriday is taller than Hridik but shorter than Hriman.
Hridik is taller than only Hritish. Hriman is not the tallest. Who among them will be in the middle if they stand in
the order of their heights?
a) Hriday b) Hriman c) Hrithik d) Cannot be determined e) None of these

21). Among Rajan, Rajas, Rajat and Rajak, Rajan is older than only Rajak. Rajas is older than Rajat. Who among
them is the oldest?
a) Rajas b) Rajat c) Rajas or Rajat d) Data inadequate e) None of these

22). In a class of forty students, Jalal's rank from the top is twelfth. Dhoni is eight ranks below Jalal. What is
Dhoni's rank from the bottom?
a) 20th b) 21st c) 22nd d) 19th e) None of these

23). In a class of 20 students, Anjali‘s rank is 15th from the top. Varun is 4 ranks above Anjali. What is Varun‘s
rank from the bottom?
a) 10th b) 11th c) 9th d) 12th e) None of these

24). In a class of 10 girls and 20 boys, Deepika's rank is '4' among the girls and '18' in the class. What is Deepika's
rank among the boys in the class?
a) Cannot be determined b) 16 c) 14 d) 15 e) None of these

25). In a row of forty boys facing North, Yogesh is twelfth from the left end and Sankar is eighteenth from the
right end. How many boys are between Yogesh and Sankar in the row?
a) 10 b) 11 c) 12 d) Cannot be determined e) None of these

Answer:
1. a 2. e 3. d 4. c 5. e 6. d 7. b 8. b 9. e 10. e 11. d 12. d 13. a 14. c 15. c 16. c 17. d 18. b 19. a 20. a 21. a 22. b
23. a 24. d 25. a

www.ibpsguide.com | www.ibpsguide.in | mock.ibpsguide.in | www.sscexamguide.com


Copyright © 2016 IBPS Guide 67
yoursmahboob.wordpress.com

RANKING (Explanations)
1).a) Dhuthi > Dhriti > Dhithi > Dhanya > Dhanvi
Dhanvi among them is the youngest

2). e)
David 1 2 3 4 5 Arafath 1 2 3 4 5 6 7 Mishra
There are five boys between David and Arafath.

3).d) Kavita > Kaushika, Kavya and Kaveri < Kavini


The heaviest among them can‘t be determined.

4).c) 17 + * # + 10
* - Priya # - Saranya
Number of boys between Priya and Saranya
= 40 – 18 – 11 + 1 = 12

5). e) Ganesh > Suresh, Lokesh


Ganesh < Ramesh, Dinesh
Suresh > Lokesh
Ramesh, Dinesh > Ganesh > Suresh > Lokesh
So, either Ramesh or Dinesh is the tallest.

6).d) Nisha > Divya


Radha > Nisha
Richa > Latha
Najima is the senior most
But no other data is there to find who is the junior most.

7). b) Clearly 6th position from right (Aruna) is 13th position from left. That means there are 13 + 5 = 18 children
in the row. Hence 5th position from left will be 18 – 5 + 1 = 14th from right (Aruna‘s new position).

Previous by Hameed – 5th


Aruna – 13th
1 2 3 4 5 6 7 8 9 10 11 12 13 14 15 16 17 18
Present Aruna – 5th
Hameed – 13th

8). b) Ramesh > Gokul


Teja > Ramesh

www.ibpsguide.com | www.ibpsguide.in | mock.ibpsguide.in | www.sscexamguide.com


Copyright © 2016 IBPS Guide 68
yoursmahboob.wordpress.com

Lokesh > Balaji


Gokul > Lokesh
Hence, Teja > Ramesh > Gokul > Lokesh > Balaji
Hence, Teja won the race.

9).e) Correct order can‘t be determined.

10).e) Total number of boys = 41.

11).d) Ascending order of weight : Tarik > Taran > Tarit > Taral > Tariq
So, third heaviest is Tarit.

12).d) Since Udaya can be to the right of Prem


35 ---------------------------------- 15th---------------- 10 -------------- 4th
Prem – 15th
Udaya – 4th
or to the left of Prem
9---------- 10th -------------------- 15th ----------- 14
Udaya – 10th
Prem – 15th
Udaya‘s position can‘t be determined.

13).a) Gandhi < Aravind; Praveen < Gandhi, Karan; Karan < Aravind
Aravind > Karan / Gandhi > Praveen

14).c) Total number of children in a row


= 12 + 4 + 22 – 1 = 37

15).c) Total number of students in the class


= 16 + 12 - 1 =27

16).c) Namish > Naresh > Natesh


Nakul > Namit > Namish
Natesh < Naresh < Namish < Namit < Nakul

17).d) In order to solve this question, we must know the position of either Soundarya or Hema from both the
ends.

18).b) From the information given in the question, the newspaper was read in the following order

www.ibpsguide.com | www.ibpsguide.in | mock.ibpsguide.in | www.sscexamguide.com


Copyright © 2016 IBPS Guide 69
yoursmahboob.wordpress.com

Najma, Nalina, Nalika, Nakula, Nalini.


Hence Najma passed the newspaper to Nalina.

19).a) 23 + * + + + + + + # + 10
* - Prakash # - Gopinath
Prakash's position from the right end of the row
= 40 – 23 = 17th

20).a) Hrithik > Hriman > Hriday > Hridik > Hritish
A will be in the middle if they stand in the order of the height.

21).a) Rajan > Rajak, Rajas > Rajat


Rajas > Rajat > Rajan > Rajak (Rajan is older than only Rajak)
Hence, Rajas is the eldest.

22).b) Total students = 40


12 + + + + + + + 20
Jalal Dhoni
Dhoni's rank from bottom
= 40 – (12 + 8) + 1
= 21st

23).a) Varun‘s rank from the top = 15 – 4 = 11th.


Varun‘s rank from the bottom
= 20 + 1 – 11 = 10th

24).d) 3 Girls are ahead of Deepika.


14 Boys are ahead of Deepika.
Deepika's rank is 15th among the boys in the class.

25).a) Yogesh = 12th from left


Sankar = 18th from right = (40 – 18 + 1) = 23rd from left
No. of boys between Yogesh and Sankar = 23 – 12 – 1 = 10

www.ibpsguide.com | www.ibpsguide.in | mock.ibpsguide.in | www.sscexamguide.com


Copyright © 2016 IBPS Guide 70
yoursmahboob.wordpress.com

7. EQUALITY-INEQUALITY

Q 1 – 5) As per the given information below, answer the questions that follows –
 P @ Q means P is smaller than Q
 P % Q means P is greater than Q
 P ¥ Q means P is neither greater than nor less than Q
 P $ Q means P is either smaller than or equal to Q
 P # Q means P is either greater than or equal to Q
Now in each of the following questions assuming the given statements to be true, choose amongst the correct
options for the questions given below:

Q 1). A@B, B¥Q, Q$H, H@R, R¥S


Conclusion 1 –A@S Conclusion2 – Q%S
a) If only conclusion 1 is true
b) If only conclusion 2 is true
c) If either conclusion 1 or conclusion 2 is true
d) None of these

Q 2). M#N, N¥J, J%B, B¥K, K#H


Conclusion 1 – M#N Conclusion 2 – J%H
a) If only conclusion 1 is true
b) If only conclusion 2 is true
c) If either conclusion 1 or conclusion 2 is true
d) If both conclusion 1 &conclusion 2 are true

Q 3). U¥V, V$X, X¥Y,Y%Z, Z #A


Conclusion 1 – U@Y Conclusion 2 - U¥Y
a) If only conclusion 1 is true
b) If only conclusion 2 is true
c) If either conclusion 1 or conclusion 2 is true
d) If both conclusion 1 and conclusion 2 are true

Q 4). C@D, D%E, E$F, F#G, G¥H


Conclusion 1 –C$F Conclusion 2 – D@F
a) If only conclusion 1 is true
b) If only conclusion 2 is true
c) If either conclusion 1 or conclusion 2 is true
d) Neither conclusion 1 nor conclusion2 follows

Q 5). G%K, K$H, H#N, N¥T, T@B


Conclusion 1 – H%N Conclusion 2 - H¥N
a) If only conclusion 1 is true
b) If only conclusion 2 is true
c) If both conclusion 1 and conclusion 2 are true
d) None of these
www.ibpsguide.com | www.ibpsguide.in | mock.ibpsguide.in | www.sscexamguide.com
Copyright © 2016 IBPS Guide 71
yoursmahboob.wordpress.com

Q Directions (6 – 10): In these questions, the relationship between different elements is shown in the
statements. Give answer:
a) If only conclusion I follows
b) If only conclusion II follows
c) If either conclusion I or II follows
d) If neither conclusion I nor II follows
e) If both conclusion I and IIfollow
Q 6). Statements: A = B ≠ E > T
Conclusions: I. A ≥ T II. A < T

Q 7). Statements: S > O ≥ F ≤ T


Conclusions: I. O > T II. O ≤ T

Q 8). Statements: C = A ≠ P
Conclusions: I. P > C II. P < C

Q 9). Statements: D ≥ A ≥ Y, S = Y
Conclusions: I. D > S II. D = Y

Q 10). Statements: I > D > E ≠ A = L


Conclusions: I. I ≥ L II. I < L

Q 11 – 15) Answer the question on the basis of the statements given below :
 U@V means U is less than V
 U$V means U is either less than or equal to V
 U#V means U is greater than V
 U%V means U is either greater than or equal to V
 U©V means U is neither greater than nor less than V

Q 11). T#R, R%H, H@F


Conclusion 1 – H@T Conclusion 2 – F©T
a) If only conclusion 1 is true
b) If only conclusion 2 is true
c) If either conclusion 1 or conclusion 2 is true
d) None of these

Q 12) L$K, K@M, J%M


Conclusion 1 – L@M Conclusion 2 – K@J
a) If only conclusion 1 is true
b) If only conclusion 2 is true
c) If both conclusion 1 and conclusion 2 is true
d) None of these

Q 13). F©D, D#V, V@P


www.ibpsguide.com | www.ibpsguide.in | mock.ibpsguide.in | www.sscexamguide.com
Copyright © 2016 IBPS Guide 72
yoursmahboob.wordpress.com

Conclusion 1 – F©P Conclusion 2 –V@F


a) If only conclusion 1 is true
b) If either conclusion 1 or conclusion 2 is true
c) If both conclusion 1 and conclusion 2 is true
d) None of these

Q 14). E$W, W@Q, Q%H


Conclusion 1 – E$Q Conclusion 2 – E©H
a) If only conclusion 1 is true
b) If only conclusion 2 is true
c) If both conclusion 1 or conclusion 2 is true
d) None of these

Q 15). J©T, T#W, W%R


Conclusion 1 – J#R Conclusion 2 – T%R
a) If only conclusion 2 is true
b) If either concusion 1 or conclusion 2 is true
c) If only conclusion 1 is true
d) None of these
Directions(16 – 21) Answer the question on the basis of the statements given below :
 X%Y means X is smaller than Y
 X©Y means X is either smaller than or equal to Y
 X*Y means X is neither greater than nor smaller than Y
 X@Y means X is greater than Y
 X$Y means X is either greater than or equal to Y

Q 16). M@D, D©W, W*B


Conclusion 1 – B$D Conclusion 2 – M@B
a) If either conclusion 1 or conclusion 2 is true
b) If both conclusion 1 and conclusion 2 is true
c) If only conclusion 1 is true
d) None of these

Q 17). H*B, B$A, A%R


Conclusion 1 – R@H Conclusion 2 – A©H
a) If either conclusion 1 or conclusion 2 is true
b) If both conclusion 1 and conclusion 2 is true
c) If only conclusion 1 is true
d) If only conclusion 2 is true

Q 18). B$D, D@J, J%M


Conclusion 1 – J%B Conclusion 2 – M@D
a) If neither conclusion 1 nor conclusion 2 is true
b) If only conclusion 2 true
c) If only conclusion 1 is true
www.ibpsguide.com | www.ibpsguide.in | mock.ibpsguide.in | www.sscexamguide.com
Copyright © 2016 IBPS Guide 73
yoursmahboob.wordpress.com

d) None of these

Q 19). R@M, M*K, K©T


Conclusion 1 – T@M Conclusion 2 – T*M
a) If neither conclusion 1 nor conclusion 2 is true
b) If only conclusion 2 is true
c) If either conclusion 1 or conclusion 2 is true
d) None of these

Q 20). J$N, N%D, D@F


Conclusion 1 – F%N Conclusion 2 – J@D
a) If only conclusion 1 is true
b) If only conclusion 2 is true
c) If neither conclusion 1 and conclusion 2 is true
d) None of these

Q 21). K%N, N*J, J©F


Conclusion 1 – F$N Conclusion 2 – J@K
a) If only conclusion 1 is true
b) If only conclusion 2 is true
c) If both conclusion 1 and conclusion 2 is true
d) None of these

Q 22). Z ≥ U = T ; T < W ; R ≥ W ; Q = R
Conclusion 1 – Z ≥ Q Conclusion 2 – Z ≤ W
a) If only conclusion 1 is true
b) If only conclusion 2 is true
c) If either conclusion 1 or conclusion 2 is true
d) None of these

Q 23). Z ≥ U = T ; T < W ; R ≥ W ; Q = R
Conclusion 1 – T < Q Conclusion 2 – W > Q
a) If only conclusion 1 is true
b) If only conclusion 2 is true
c) If either conclusion 1 or conclusion 2 is true
d) None of these

Directions (24 – 30): Answer the questions as per the information given below :
 U©V means U is not smaller than V
 U*V means U is not greater than V
 U@V means U is neither greater than nor smaller than V
 U$V means U is neither smaller than nor equal to V
 U%V means U is neither greater than nor equal to V

Q 24). K©L, L%O, O@M, M*N


www.ibpsguide.com | www.ibpsguide.in | mock.ibpsguide.in | www.sscexamguide.com
Copyright © 2016 IBPS Guide 74
yoursmahboob.wordpress.com

Conclusions : 1 – N©O 2 – M$L 3 – K*N 4 – L@N


a) Only 1 is true
b) Only 2 is true
c) Both 1 & 2 are true
d) None of these

Q 25). A*B, B$C, C%D, D©E


Conclusions : 1 – D$A 2 – B$D 3 – E%C 4 – A@E
a) Only 1 is true
b) Either 1 or 2 is true
c) Neither of the conclusions is true
d) None of these

Q 26). F$P, P@R, R©S, S%T


Conclusions : 1 –R%F 2 – S*P 3 – P©T 4 – S%F
a) Only 1, 2 & 3 are true
b) Only 1 & 2 are true
c) Only 3 & 4 are true
d) Only 1, 2 & 4 are true

Q 27). G%H, H*I, I$J, J@K


Conclusions : 1 – G%I 2 – G%J 3 – K$I 4 – H*J
a) Only 1 is true
b) Only 2 is true
c) Only 1, 2 & 3 are true
d) Only either 1 or 2 and 3 are true

Q 28). V@W, W%X, X*Y, Y$Z


Conclusions : 1 – Z$X 2 – Y©V 3 – W%Y 4 – Y@W
a) Only 1 & 3 are true
b) Only 2 is true
c) None is true
d) Only 3 is true

Q 29) R%S, S@T, T©U, U$V


Conclusions : 1 – R@V 2 – S©U 3 – R%T 4 – T$V
a) Only 1 & 2 are true
b) Only 2 is true
c) None is true
d) Only 2, 3 & 4 are true

Q 30). P©S, S@T, T$U, U*V


Conclusions : 1 – P$U 2 – U%S 3 – V$T 4 – T*V
a) Only 1 & 3 are true
b) Only 4 is true
www.ibpsguide.com | www.ibpsguide.in | mock.ibpsguide.in | www.sscexamguide.com
Copyright © 2016 IBPS Guide 75
yoursmahboob.wordpress.com

c) Only 1 and 2 and either 3 or 4 is true


d) None of these

Direction (31 – 38): Answer the Questions as per the information given below :
 J@K means J is neither smaller than nor equal to K
 J#K means J is not greater than K
 J$K means J is not smaller than K
 J%K means J is neither greater than nor smaller than K
 J&K means J is neither greater than nor equal to K

Q 31). D&T, R#T, R$M


Conclusion 1 – M&T Conclusion 2 – M%T
a) If only conclusion 1 is true‘
b) If only conclusion 2 is true
c) If either conclusion 1 or conclusion 2 is true
d) None of these

Q 32). E%H, H$M, M#Q


Conclusion 1 – H$Q Conclusion 2 – E$M
a) If only conclusion 1 is true
b) If only conclusion 2 is true
c) If either conclusion 1 or conclusion 2 is true
d) If neither conclusion 1 nor conclusion 2 is true

Q 33). F$D, H#M, M%D


Conclusion 1 – F$H Conclusion 2 – F@H
a) If only conclusion 1 is true
b) If only conclusion 2 is true
c) If either conclusion 1 or conclusion 2 is true
d) None of these

Q 34). G#H, H$K, K@M


Conclusion 1 – M#G Conclusion 2 – G&M
a) If only conclusion 2 is true
b) If only conclusion 1 is true
c) If either conclusion 1 or conclusion 2 is true
d) If neitherconclusion 1 nor conclusion 2 is true

Q 35). R&M, M#L, L$Q


Conclusion 1 – M%Q Conclusion 2 – M@Q
a) If conclusion 1 is true
b) If conclusion 2 is true
c) If either conclusion 1 or conclusion 2 is true
d) If neither conclusion 1 nor conclusion 2 is true

www.ibpsguide.com | www.ibpsguide.in | mock.ibpsguide.in | www.sscexamguide.com


Copyright © 2016 IBPS Guide 76
yoursmahboob.wordpress.com

Q 36). F#R, Q$R, Q&M


Conclusion 1 – F#Q Conclusion 2 – R&M
a) If only conclusion 1 is true
b) If only conclusion 2 is true
c) If either conclusion 1 or conclusion 2 is true
d) If both conclusion 1 and conclusion 2 are true

Q 37). S#A, S@T, L&T


Conclusion 1 – L&A Conclusion 2 – S@L
a) If only conclusion 1 is true
b) If only conclusion 2 is true
c) If both conclusion 1 and conclusion 2 are true
d) If neither conclusion 1 nor conclusion 2 is true

Q 38). G$J, J@K, K%N


Conclusion 1 – G@N Conclusion 2 – G%N
a) If both conclusion 1 and conclusion 2 are true
b) If only conclusion 2 is true
c) If only conclusion 1 is true
d) If neither conclusion 1 nor conclusion 2 is true

Directions (39 – 40): In the given Questions the following statements are given as per the given questions, find
out which of the conclusions given are true according to the statements given :
Q 39) Statements: R=K, T≤K , T≥M, M<W
Conclusions : 1 – W<K 2 – M ≤R 3 – K≤R
a) Only 1 is true
b) Only 2 is true
c) Only 1 and 2 are true
d) Only 2 and 3 are true

Q 40). Statements :B>N, N≥R, H≤R


Conclusions : 1 – B>R 2 – H≤N 3 – H<N
a) Only 1 and 2 are true
b) Only 2 and 3 are true
c) Only 1 and 3 are true
d) All 1, 2 and 3 are true

Directions(41 – 45): In these questions, relationship between different elements is shown in the statements. The
statements are followed by two conclusions. Give answers –
Q 41). Statements :J>A=K<D<E>F
Conclusion 1 – F<K Conclusion 2 – J>D
a) If only conclusion 1 is true
b) If only conclusion 2 is true
c) If either conclusion or conclusion 2 is true
d) If neither conclusion 1 nor conclusion 2 is true
www.ibpsguide.com | www.ibpsguide.in | mock.ibpsguide.in | www.sscexamguide.com
Copyright © 2016 IBPS Guide 77
yoursmahboob.wordpress.com

e) If both conclusions 1 and 2 are true

Q 42). Statements :Q=R>T>X>Y; K<T


Conclusion 1 –K<Q Conclusion2 – Y<T
a) If only Conclusion I is true
b) If only Conclusion II is true
c) If either Conclusion I or II is true
d) If neither Conclusion I nor II is true
e) If both Conclusions I and II are true

Q 43). Statements :T<J>S>P; T>F; P>G


Conclusion 1 – G>J Conclusion 2 – J>F
a) If only Conclusion I is true
b) If only Conclusion II is true
c) If either conclusion or conclusion 2 is true
d) If neither Conclusion I nor II is true
e) If both Conclusions I and II are true

Q 44). Statements :U=V<X>Y; W>X<Z


Conclusion 1 – W>U Conclusion 2 – Z>Y
a) If only Conclusion I is true
b) If only Conclusion II is true
c) If either Conclusion I or II is true
d) If neither Conclusion I nor II is true
e) If both Conclusions I and II are true

Q 45). Statements :A > B = C; D < C > E


Conclusion 1 – D < AConclusion 2 – E < A
a) If only Conclusion I is true
b) If only Conclusion II is true
c) If either conclusion or conclusion 2 is true
d) If neither conclusion or conclusion 2 is true
e) If both Conclusions I and II are true

Directions(46 – 50):In the following questions, thesymbols @, #, %, $ and * are used with the followingmeaning
as illustrated below.
 ‘S @ T’ means ‘S is not smaller than T
 ‘S # T’ means ‘S is neither smaller than nor equal to T’
 ‘S % T’ means ‘S is neither smaller than nor greater than T’
 ‘S $ T’ means ‘S is not greater than T’
 ‘S * T’ means ‘S is neither greater than nor equal to T’

Q 46). Statements: L#M, N@O, M*P, O%L


Conclusion 1 – N*P Conclusion 2 – N@P
a) If only conclusion 2 is true
www.ibpsguide.com | www.ibpsguide.in | mock.ibpsguide.in | www.sscexamguide.com
Copyright © 2016 IBPS Guide 78
yoursmahboob.wordpress.com

b) If either conclusion or conclusion 2 is true


c) If both Conclusions I and II are true
d) If only conclusion 1 is true
e) If neither conclusion 1 nor conclusion 2 is true

Q 47). Statements: U*V, Y#Z, X$Y, W%X, V*W


Conclusion 1 – U#Z Conclusion 2 – V*Y
a) If only conclusion 1 is true
b) If either conclusion or conclusion 2 is true
c) If only conclusion 2 is true
d) If both conclusions 1 and conclusion 2 are true
e) If neither conclusion 1 nor conclusion 2 is true

Q 48). Statements: C%D, E$K, K*C, K@L


Conclusion 1 – C*D Conclusion 2 – E@L
a) If neither conclusion 1 nor conclusion 2 is true
b) If both Conclusions I and II are true
c) If only conclusion 1 is true
d) If either conclusion or conclusion 2 is true
e) If only conclusion 2 is true

Q 49). Statements :A@U, U%V, V#T, K$M


Conclusion 1 :A#T Conclusion 2 – A%K
a) If only conclusion 1 is true
b) If either conclusion 1 or conclusion 2 is true
c) If both conclusion 1 and conclusion 2 are true
d) If neither conclusion 1 nor conclusion 2 is true
e) None of these

Q 50). Statements :N#A, I%T, T@N, L*A


Conclusion 1 :I$L Conclusion 2 : I#L
a) If only conclusion 2 follows
b) If neither conclusion 1 nor conclusion 2 follows
c) None of these
d) If conclusion 2 follows
e) If either conclusion 1 or conclusion 2 follows.

www.ibpsguide.com | www.ibpsguide.in | mock.ibpsguide.in | www.sscexamguide.com


Copyright © 2016 IBPS Guide 79
yoursmahboob.wordpress.com

EQUALITY -INEQUALITY (Answers with Explanation)

Directions (1 – 5):
Q 1). a). Since between A & S one entity is < and another entity is > so no regular pattern hence no
conclusion follows.

Q 2). d). Since between M & N only one entity is there i.e. ≥ hence M≥N holds true so 1 follows. Between J
& H there are three entities – >, =, ≥. Here > is the strong entity so J>H.

Q 3). c) Since between U and Y there are two entitites – = and ≤ so both at first U<Y and U=Y does not
agree. Secondly in both conclusion there are both equal attributes U and Y while ≤ is the strong entity in
between them and in one conclusion its given < while in other its given = . henceanwereill be either or.

Q 4). d) Since between C and F both < and > is there so no regular pattern is there hence conclusion 1 does
not follow. Between D and F the same thing no regular pattern follows hence 2 also does not follow. So
neither follows.

Q 5). d) Since between H and N there is only one entity ≥ so both at first H>N and H=N does not agree.
Secondly in both conclusion there are both equal attributes U and Y while ≥ is that one strong entity in
between them and in one conclusion its given > while in other its given =. Hence either of them follows. So
according to option for either or is not given so none of these will be the answer.

Directions(6 – 10) :
Q 6). c)
‗B ≠ E‘ means that B > E or B < E. So a > T or A < T or A = T.
Hence, either I or II follows

Q 7). c)
O ≥ F ≥ T, as > and < are there between O and T, the relationship between cannot be compared.
Hence O > T or O ≤ T

Q 8). c)
As C = A ≠ P, C > P or C < P
So either I or II follows

Q 9). c)
D ≥ A ≥ Y = S; Either D > Y (or S) or D = Y (or S)

Q 10). c)
I > L or I < L or I = L
Hence, either I or II follows.

Directions (11 – 15):

www.ibpsguide.com | www.ibpsguide.in | mock.ibpsguide.in | www.sscexamguide.com


Copyright © 2016 IBPS Guide 80
yoursmahboob.wordpress.com

Q 11). a) Between H and T there are two entities ≤ and < so there is a regular pattern while < is the strong
entity, so conclusion 1 follows but between F and T there is no regular pattern due to < and >, so conclusion 2
does not follows.

Q 12). c) Between L and M there are two entities ≤ and < while < is the strong entity so conclusion 1 follows.
Between K and J there are two entities < and ≤ while < being the strong entity so conclusion 2 also follows.

Q 13). d) Between F and P there are three entities – =, >, <, so no regular pattern is there hence conclusion 1
does not follow. While between V and F there are two entities < and = thereby following a regular pattern and
as < is the strong entity, hence conclusion 2 follows. But in option this option is not given so none of these
will be the answer.

Q 14). d) Between E and Q there are two entities ≤ and <, so regular pattern follows with < being the strong
entity so E<Q is the answer but in conclusion 1 its given E≤Q , so 1 does not follow. Now between E and H
there are three different entities ≤, < and ≥, which shows no regular pattern. Hence conclusion 2 does not
follows.

Q 15). c) J and R have three entities between them all being different – =,> and ≥ with > being the strong
entity, so conclusion 1 follows. While between T and R there are two entities > and ≥ with > being the strong
entity so T>R agree but no given in the conclusion 2 so it does not follow.

Directions(16 – 21):
Q 16). c) In between B and D there are two entities ≥ and = thereby following a regular pattern with ≥ being
the stronger entity so conclusion 1 follows. Now between M and B, the two entities being > and ≤ which
contradicts the regularity of the pattern so conclusion 2 does not follow.

Q 17). d) Between R and H the two entities > and ≤ are there which implies the irregularity in the pattern so
conclusion 1 does not follow while between A and H there are two entities ≤ and = which shows a regular
pattern with ≤ the stronger entity, so conclusion 2 agrees.

Q 18). c) Between J and B two entities < and ≤ are there with < being the stronger entity so, conclusion 1
follows. While between M and D there is no regular pattern due to two different sided entities like > and < so
conclusion 2 does not follows.

Q 19). c) Between T and M, there are two entities ≥ and = with ≥ being the stronger one so T≥ M holds true,
but both conclusions 1) M<T and 2) M = T turns out to be wrong and the variables being M and T are the
same so it fulfills the conditions of either or hence c is the option.

Q 20). c) In both conclusion there are two different sided entities between F and N, J and Dso neither of the
conclusion follows, hence c is the correct option.

Q 21). c) Between F and N there is a regular pattern due to one sided entity with ≥being the stronger one so
conclusion 1 follows. Similarly between J and K there is a regular pattern with > being the strong entity so
conclusion 2 also follows, hence option c is the correct answer.

www.ibpsguide.com | www.ibpsguide.in | mock.ibpsguide.in | www.sscexamguide.com


Copyright © 2016 IBPS Guide 81
yoursmahboob.wordpress.com

Q 22). d) Between Z and Q there are different sided entities like ≥ and < so no regular pattern, hence
conclusion 1 does not follow. While between J and W same is the case so no regular pattern, hence conclusion
2 also does not follows, hence d is the correct option.

Q 23). a) Between T and Q there are three entities <, ≤ and = are there which shows one sided entities so
regular pattern is there with < being the stronger entity and which is given in conclusion 1 so 1 follows. In
conclusion 2 between W and Q there are two entities as per the statement given and those are ≥ and = with ≥
the stronger one but this is not given in conclusion 2 hence 2 does not follow. So option a is the correct
answer.

Directions(24 – 30) :
Q 24). c) Between N and O there is a regular pattern with one sided entity and the stronger one is ≥ which
makes conclusion 2 follows. Then in conclusion 2 between M and L there are one sided entities like ≥ and >
with > as the stronger entity which is given conclusion 2 hence it follows. In conclusion 3 between K and N
there are two different sided entities like ≥, and < due to which conclusion 3 does not follow. In conclusion 4
between L and N there are only one sided entities like < and ≤ with < being the stronger entity but its not
given in the conclusion 4 so 4 does not follow. Hence c is the correct option.

Q 25). c) Since in all the conclusion between the given variables D,A; B, D; E, C; A, E; none of them have
one sided entities and so with no regular pattern, hence neither of the conclusions are true. So option c is the
correct option.

Q 26). d) In conclusion 1, 2 and 4 R, F; S, P; S, F forms a regular pattern with one side entity and in each the
strong entities are <, ≤ and < which are given in the conclusion also so all these three conclusions 1, 2 and 4
follows. While in conclusion 3 between P and T there are two different sided entities like ≥ and < which does
not give it a regular pattern so it does not agree, so option d is the correct answer

Q 27). c).

Q 28). c) In conclusion 3 between W and Y there are two one sided entities < and ≤. In conclusion 1, 2 and 4,
they all have two or more than two different sided entities like ≤ and >etc so they don‘t follow a regular
pattern, hence conclusion 1,2 and 4 does not follows so correct option is d.

Q 29). d) In conclusion 1 between R and V there are two different sided entities < and > so no reguar pattern
and hence conclusion 1 does not follow. In conclusion 2, 3 and 4 there are one sided entities in each case
showing a regular pattern, so 2,3 and 4 follows, hence d is the correct option.

Q 30). e)

Directions(31 – 38):
Q 31). c) In both conclusion 1 and 2 between M and T the existing entity being the stronger and also
following the regular pattern is ≤, so M≤T should be there but its not given in conclusion 1 and 2, but the
variables being the same M and T with the combining entity in both conclusions giving out to be ≤, thereby
satisfying the conditions for either or, option c is the correct answer.

www.ibpsguide.com | www.ibpsguide.in | mock.ibpsguide.in | www.sscexamguide.com


Copyright © 2016 IBPS Guide 82
yoursmahboob.wordpress.com

Q 32). b) In conclusion 1 between H and Q there are two different sided entities ≥ and ≤ due to which there is
no regular pattern, so conclusion 1 does not follows. In conclusion 2 between E and M, only 1 sided entity is
there with ≥ the stronger 1 which is given in conclusion, hence conclusion 2 follows and option b is the
correct answer.

Q 33). a) In conclusion 1 between F and H there is only one sided entity with ≥ being the stronger one so
conclusion 1 follows. The same is not the case in conclusion 2 where it is given F > M which does not agrees
as per the statement given, hence a is the correct option.

Q 34). d)

Q 35). d) Since in both the conclusions between M and Q, there are two different sided entities, so no regular
pattern, hence neither of the two conclusion follows, so option d is correct.

Q 36). d) In conclusion 1 between F and Q there is only one one sided entity which is ≤ with it being the
stronger one so conclusion 1 follows. In conclusion 2 between R and M there are two one sided entities with <
being the stronger one which is given in the conclusion, so conclusion 2 also follows. Hence correct option is
d, both follows.

Q 37). c) In conclusion 1 between L and A there are two one sided entities making it a regular pattern, with <
being the stronger entity so 1 follows. In conclusion 2 between S and L there is only one one sided entity and
that is > which is given in conclusion 2 so 2 also follows, hence c is correct – both follows.

Q 38). d)

Q 39). b) In conclusion 1 between W and K, there are two different sided entities, so it does not follows,
while in conclusion 3 between K and R only entity = exists which is not mention in 3 so it does not follows. In
conclusion 2 between M and R there are two one sided entities ≤ and = in which ≤ is the strong one which is
mentioned in conclusion 2 so it follows. Hence option b is correct.

Q 40). a) In conclusion 2 between B and R there are two one sided entity > and ≥ are there with > being the
stronger one, so 1 follows. In conclusion 2 between H and N only one one sided entity is given which is ≤
which is given in the conclusion 2 so it follows. In conclusion 3 same variables as that of conclusion 2 but the
entity mention in it is different so 3 does not follows. Hence option a is correct.

Directions(41 – 45):
Q 41). d) in both the conclusions there are different sided entities so no regular pattern, so neither of the
conclusion follows. Hence option d is correct.

Q 42). e) In conclusion 1 between K and Q there are two one sided entities < and = are given with < being the
strong one so conclusion 1 follows. While in conclusion 2 between Y and T only one one sided entity ie< is
given so conclusion 2 also follows. Hence option e is correct.

www.ibpsguide.com | www.ibpsguide.in | mock.ibpsguide.in | www.sscexamguide.com


Copyright © 2016 IBPS Guide 83
yoursmahboob.wordpress.com

Q 43). b) In conclusion 1 between G and J only one one sided entity and that is < is there which is not given
in the conclusion so 1 does not follows. In conclusion 2 between J and F only one one sided entity ie> is there
which is mention in the conclusion 2 so 2 follows. Hence option b is vorrect.

Q 44). e) In conclusion 1 between W and U there are two one sided entity with >bein the stronger one which
is given in conclusion 1 so 1 follows. In conclusion 2 between Z and Y a regular pattern is followed where >
being the stronger entity is given in the conclusion 2 so 2 also follows. Hence option e is correct.

Q 45). e) In both the conclusion 1 and 2 there is one single sided entity between both D and A and E and A
and that is < which agrees with the statements also, so both conclusions follows. Hence e is correct.

Directions(46 – 50) :
Q 46). b) Since both conclusions 1 and 2 does not follow and the second condition of having the same
variables also gets satisfied, we can see the entities present in either of them if we exclude 1 from another ie if
we exclude < then we get ≥ and vice versa so 3rd condition also gets satisfied. Hence there is either or in it,
according to which option b is correct.

Q 47). c) For conclusion 1, from the statement we get that between U and Z, there are two sided entities due
to which the pattern is not regular, so 1 does not follow. For conclusion 2 from the statement given, we get
that there are three one sided entities <, = and ≤, with < being the stronger one. Hence conclusion 2 follows.
So correct option is c.

Q 48). a) From the given statements, conclusion 1 does not follow because there is only one entity given I
between the two which is = and this entity is not mentioned in the conclusion, while in conclusion 2 we get
from the statement that between E and L there are two different sided entities, so 2 does not follow.

Q 49). a) from the statement between A and T, there are three one sided entities ≥, = and > with > being the
stronger one which is given in conclusion 1, so 1 follows. But in conclusion 2 from the statement there is no
relation given between A and K so 2 does not follow, hence correct option is a.
Q 50). d)

8. SYLLOGISM

Directions: (1-25) In each question below, there All pens are phones.
are three or four statements followed by four Conclusions:
conclusions numbered I, II, III and IV. You I. Some phones are watches.
have to take the given statements to be true II. Some pens are gold.
even if they seem to be at variance with III. Some gold are watches.
commonly known facts and then decide which IV. No pen is gold.
of the given conclusions logically follow(s) from
the given statements. a) Only either II or IV and III follow
b) Only either II or IV and I follow
Q 1. Statements: c) Only either I or II and IV follow
All watches are gold. d) None follows
Some phones are gold. e) All follow
www.ibpsguide.com | www.ibpsguide.in | mock.ibpsguide.in | www.sscexamguide.com
Copyright © 2016 IBPS Guide 84
yoursmahboob.wordpress.com

c) Only III and IV follow


Q 2. Statements: d) Only II and III follow
All tigers are rabbits. e) None of these
No rabbit is a mouse.
All mouse are fruits. Q 5. Statements:
Conclusions: Some tapes are radios.
I. No lion is a mouse. Some radios are keys.
II. Some fruits are mouse. All tigers are keys.
III. Some rabbits are tigers. Conclusions:
IV. Some tigers are mouse. I. Some tapes are tigers.
II. Some tapes are keys.
a) All follows III. No water is tape.
b) Only either I or II and both III and IV follow IV. All keys are tigers.
c) Only either I or IV and both II and III follow
d) Only either I or IV and II follow a) None follows
e) None of these b) Both II and III follow
c) Only either II or III follows
Q 3. Statements: d) Only either I or III follows
Some hills are trains. e) Only either I or IV follows
Some trains are roads.
Some roads are codes. Q 6. Statements:
Conclusions: Some buses are cars.
I. Some codes are roads. All cars are boxes.
II. Some trains are hills. Some roads are boxes.
III. Some roads are hills. Conclusions:
IV. Some codes are trains. I. Some boxes are buses.
II. Some roads are buses.
a) All follow III. Some roads are cars.
b) Only I and II follow IV. Some boxes are roads.
c) Only III and IV follow
d) Only I and IV follow a) None follows
e) None follows b) Only I and II follow
c) Only III and IV follow
Q 4. Statements: d) Only I and IV follow
All branches are trees. e) All follows
All trees are leafs.
All leafs are wires. Q 7. Statements:
Conclusions: All bats are cells.
I. All wires are branches. Some cells are rabbits.
II. All trees are wires. All rabbits are carrots.
III. Some wires are leafs. Conclusions:
IV. Some leafs are branches. I. Some rabbits are bats.
II. Some carrots are cells.
a) All follow III. Some carrots are bats.
b) Only II, III and IV follow IV. Some carrots are rabbits.
www.ibpsguide.com | www.ibpsguide.in | mock.ibpsguide.in | www.sscexamguide.com
Copyright © 2016 IBPS Guide 85
yoursmahboob.wordpress.com

II. Some sheets are cycles.


a) Only I and III follow III. Some sheets are files.
b) Only I and II follow IV. Some buses are files.
c) Only II and III follow
d) Only III and IV follow a) None of the above
e) None of these b) I and II only
c) II and III only
Q 8. Statements: d) III and IV only
All cards are lines. e) All of the above
No lines are tools.
All tools are doors. Q 11. Statements:
Conclusions: Some bikes are buildings.
I. No cards are tools. All buildings are bottles.
II. No cards are doors. Some bottles are trucks.
III. Some doors are lines. Conclusions:
IV. Some tools are cards. I. Some bikes are trucks.
II. Some bikes are bottles.
a) Only I follow III. Some bottles are buildings.
b) Only I and II follow IV. No truck is a bike.
c) Only II and III follow
d) Only III and IV follow a) None of the above
e) None of these b) II and III only follow
c) Either only I or II and III and IV follows
Q 9. Statements: d) Either only I or IV and II and III follows
Some wires are leafs. e) All of the above
No wire is a boat.
All boats are cars. Q 12. Statements:
Conclusions: No cards are lines.
I. Some wires are cars. All copies are cards.
II. Some cars are boats. All cards are telephones.
III. Some cars are wires. No radios are telephones.
IV. Some leafs are wires. Conclusions:
I. Some lines are not copies.
a) All of the above II. Some copies are neither cards nor telephones.
b) II and IV only follow III. Some telephones are neither copies nor cards.
c) I and III only follow IV. Some copies are not radios.
d) II and III only follow
e) None of these a) Only I follow
b) Only IV follows
Q 10. Statements: c) Only I and IV follow
Some buses are cycles. d) All follows
All cycles are files. e) None of these
All files are sheets.
Conclusions: Q 13. Statements:
I. Some sheets are buses. Some dogs bark.
www.ibpsguide.com | www.ibpsguide.in | mock.ibpsguide.in | www.sscexamguide.com
Copyright © 2016 IBPS Guide 86
yoursmahboob.wordpress.com

All dogs bite. a) None follows


All bats bite. b) All follows
Those animals who bark are not dangerous. c) Only either I or II and III follow
Conclusions: d) Only I and III follow
I. Even those dogs who do not bark bite. e) Only I, II and III follow
II. Those dogs who do not bark, do not necessarily
bite. Q 16. Statements:
III. All those dogs which are of white colour, bite. Some umbrellas are sticks.
IV. Some dogs are bats. Some sticks are balls.
Some balls are bats.
a) Only II and III follow All bats are guns.
b) Only II follows Conclusions:
c) Only III follows I. Some balls are umbrellas.
d) Only I and III follow II. Some guns are bats.
e) None of these III. Some sticks are guns.
IV. Some balls are guns.
Q 14. Statements:
Some soldiers are famous. a) Only I, II and IV follow
Some soldiers are intelligent. b) Only II and IV follow
All soldiers are honest. c) Only II, III and IV follow
All honest and famous persons are kind-hearted. d) Only I, II and III follow
Conclusions: e) None of these
I. Some soldiers are either famous or intelligent.
II. Some soldiers are neither famous nor Q 17. Statements:
intelligent. Some cards are notebooks.
III. All intelligent persons are kind hearted. Some notebooks are dictionaries.
IV. Some intelligent persons are not kind-hearted. Some dictionaries are files.
All files are envelopes.
a) All follow Conclusions:
b) Only I and II follow I. Some envelopes are notebooks.
c) Only I, II and III follow II. Some files are books.
d) Only either I or II follows III. Some books are dictionaries.
e) None of these IV. No book is an envelope.

Q 15. Statements: a) None follows


Some bags are cats. b) Only I, II and III follow
All cats are dogs. c) Only II and III follow
No dog is a cycle. d) Only III and IV follow
All cycles are bulls. e) None of these
Conclusions:
I. No dog is a bull. Q 18. Statements:
II. Some dogs are bulls. Some keys are letters.
III. Some dogs are bags. All letters are locks.
IV. Some bulls are bags. All locks are numbers.
No number is a coin.
www.ibpsguide.com | www.ibpsguide.in | mock.ibpsguide.in | www.sscexamguide.com
Copyright © 2016 IBPS Guide 87
yoursmahboob.wordpress.com

Conclusions:
I. Some keys are numbers. Q 21. Statements:
II. All letters are numbers. All cells are bins.
III. Some locks are keys. All bins are petals.
IV. No coin is a letter. No petal is root.
All roots are leafs.
a) Only I and II follow Conclusions:
b) Only I, II and III follow I. No cell is bin.
c) Only II and III follow II. No jungle is root.
d) Only II, III and IV follow III. All cells are petals.
e) All follow IV. All leafs are roots.

Q 19. Statements: a) Only I and II follow


Some floppies are CDs. b) Only III follow
Some CDs are keyboards. c) Only I, II and III follow
Some keyboards are computers. d) All follows
Some computers are monitors. e) None of these
Conclusions:
I. Some monitors are floppies. Q 22. Statements:
II. No floppy is a monitor. All poles are fans.
III. Some computers are CDs. All fans are stands.
IV. Some keyboards are floppies. Some stands are lines.
Some lines are boxes.
a) Only I follow Conclusions:
b) Only I, III and IV follow I. Some boxes are poles.
c) Only II follows II. Some fans are boxes.
d) Only either I or II follows III. Some lines are poles.
e) None of these IV. Some lines are fans.

Q 20. Statements: a) None follows


All cards are pins. b) Only I and II follow
Some pins are tablets. c) Only II and IV follow
All tablets are needles. d) Only III and IV follow
Some needles are threads. e) Only I, II and IV follow
Conclusions:
I. Some needles are pins. Q 23. Statements:
II. Some pins are cards. Some scales are weights.
III. Some threads are needles. All weights are metals.
IV. Some needles are tablets. Some metals are rings.
All rings are bands.
a) None follows Conclusions:
b) Only I and II follow I. Some bands are scales.
c) Only I, II and III follow II. Some weights are bands.
d) Only II, III and IV follow III. Some rings are scales.
e) All follow IV. Some metals are scales.
www.ibpsguide.com | www.ibpsguide.in | mock.ibpsguide.in | www.sscexamguide.com
Copyright © 2016 IBPS Guide 88
yoursmahboob.wordpress.com

even if they seem to be at variance with


a) Only I and III follow commonly known facts and then decide which
b) Only I and II follow of the given conclusions logically not follow(s)
c) Only II and III follow from the given statements.
d) Only II and IV follow
e) None of these Q 26. Statements:
Some schools are colleges.
Q 24. Statements: Some colleges are hostels.
Some tools are seeds. No hostel is office.
Some seeds are cycles. All offices are institutes.
Some cycles are tubes. Conclusions:
Some tubes are rains. I. No hostel is institute.
Conclusions: II. Some hostels are schools.
I. Some tubes are seeds. III. Some hostels are institutes.
II. Some cycles are tools. IV. Some offices are colleges.
III. No seed is tube.
IV. Some rains are cycles. a) Only I not follow
b) Only II & IV not follows, either I or III not
a) Only I follows follow
b) Only either I or III follows c) Only IV not follows
c) Only I and II follow d) Only either I or III not follows
d) Only either I or III and IV follow e) None of these
e) None of these
Q 27. Statements:
Q 25. Statements: Some pins are needles.
All stones are pearls. Some threads are needles.
Some pearls are shells. All needles are nails.
Some shells are boxes. All nails are hammers.
No box is container. Conclusions:
Conclusions: I. Some pins are hammers.
I. Some stones are shells. II. Some threads are nails.
II. No pearl is container. III. Some pins are threads.
III. No shell is container. IV. No pin is thread.
IV. Some pearls are containers.
a) Only I, II and either III or IV not follow
a) Only II follows b) Only III & IV not follow
b) Only II & III follow c) Only I & II not follow
c) Only either II or IV follows d) All not follow
d) Only III follows e) only II follows, Either III or IV follows
e) None follows
Q 28. Statements:
Directions: (26 – 35) In each question below, Some bikes are rooms.
there are three or four statements followed by No room is sofa.
four conclusions numbered I, II, III and IV. All sofas are tables.
You have to take the given statements to be true Some tables are desks.
www.ibpsguide.com | www.ibpsguide.in | mock.ibpsguide.in | www.sscexamguide.com
Copyright © 2016 IBPS Guide 89
yoursmahboob.wordpress.com

Conclusions:
I. Some sofas are desks. Q 31. Statements:
II. No room is table. Some doors are walls.
III. Some bikes are tables. All walls are floors.
IV. No desk is room. All floors are rooms.
Some rooms are codes.
a) None follows Conclusions:
b) Only I follow I. All walls are rooms.
c) Only either II or III follows II. Some rooms are doors.
d) Only III and IV follow III. Some rooms are walls.
e) All follow IV. Some floors are doors.

Q 29. Statements: a) None follows


Some rings are letters. b) Only I & II not follow
All letters are bangles. c) Only II & III not follow
All bracelets are bangles. d) Only II, III & IV not follow
Some bangles are pendants. e) All follows
Conclusions:
I. Some rings are bangles. Q 32. Statements:
II. Some letters are pendants. Some spoons are forks.
III. Some bracelets are rings. Some forks are bowls.
IV. No pendant is ring. All bowls are plates.
Some plates are utensils.
a) All not follows Conclusions:
b) Only I not follow I. Some utensils are forks.
c) Only II, III & IV not follow II. Some plates are forks.
d) Only IV not follows III. Some plates are spoons.
e) None of these IV. Some utensils are spoons.

Q 30. Statements: a) Only I not follow


Some jeeps are tapes. b) Only II not follows
All tapes are buses. c) Only I & III not follow
Some boats are jeeps. d) Only I, III & IV not follow
Some scooters are buses. e) None of these
Conclusions:
I. Some scooters are tapes. Q 33. Statements:
II. Some boats are buses. All bikes are tables.
III. Some jeeps are scooters. All tables are desks.
IV. All buses are tapes. Some desks are branches.
Some desks are sofas.
a) All not follows Conclusions:
b) Only IV not follows I. Some branches are sofas.
c) Only II and IV not follow II. Some sofas are tables.
d) Only III not follows III. Some branches are tables.
e) None of these IV. No bike is bench.
www.ibpsguide.com | www.ibpsguide.in | mock.ibpsguide.in | www.sscexamguide.com
Copyright © 2016 IBPS Guide 90
yoursmahboob.wordpress.com

You have to take the given statements to be true


a) All not follows even if they seem to be at variance with
b) Only I & II not follow commonly known facts and then decide which
c) Only II & III not follow of the given conclusions logically follow(s) from
d) Only I, II & III not follow the given statements.
e) None of these
Q 36. Statements :
Q 34. Statements: All erasers are lines
Some sweets are chocolates. Some lines are pencils
Some chocolates are mints. All pencils are red
Some mints are food. No red is ink
Some food is diet. Conclusions :
Conclusions: I. There is a possibility that some lines are not red
I. No sweets are diet. II. Some pencils are not lines
II. No food is chocolates. III. No pencil is ink
III. Some sweets are diet. IV. Some lines are not ink
IV. Some sweets are food.
a) Only I and II follows
a) All not follows b) Only II and IV follow
b) Either I or III not follows c) Only I, II and IV follows
c) Only III & IV not follow d) Only I, III and IV follows
d) Only II & IV and either I or III not follows e) None of these
e) None of these
Q 37. Statements :
Q 35. Statements: Some sheets are files
Some doctors are lawyers. Some cats are files
All teachers are lawyers. Some cats are whites
Some engineers are lawyers. Conclusions :
All engineers are businessmen. I. Some cats being sheets is a possibility
Conclusions: II. Some sheets not being files is a possibility
I. Some teachers are doctors. III. Some whites are files
II. Some businessmen are lawyers. IV. Some sheets are whites
III. Some businessmen are teachers.
IV. Some lawyers are teachers. a) Only I and II follows
b) Only II and III follow
a) All not follows c) Only I and IV follow
b) Only II not follows d) Only II, III and IV follow
c) Only I and III not follow e) None of these
d) Only II and IV not follow
e) None of these Q 38. Statements :
Some gases are liquids
All liquids are water
Directions: (36 – 50)In each question below, Some keys are whites
there are three or four statements followed by Conclusions :
four conclusions numbered I, II, III and IV. I. All gases being water is a possibility
www.ibpsguide.com | www.ibpsguide.in | mock.ibpsguide.in | www.sscexamguide.com
Copyright © 2016 IBPS Guide 91
yoursmahboob.wordpress.com

II. All such gases that are not water can be liquids. Q 41. Statements :
III. Some liquids are whites No table are bikes
IV. All whites are keys is a possibility All bikes are black
Some blacks are notes
a) None follows Conclusions :
b) Only I follow I. All back being take is a possibility
c) Only II follows II. Some bikes being table is a possibility
d) Only III follows III. Some bikes are notes
e) Only I and IV follows. IV. No note is a table

Q 39. Statements : a) None follows


Some professors are clerks b) Only I follow
Some teachers are professors c) Only II follows
Some lecturers are teachers d) Only III follows
Conclusions : e) Only I and II follows.
I. Some lecturers as well as teachers are professors
is the possibility Q 42. Statements :
II. Some teachers who are lecturers are also Some cats are sheets
professors No sheet is a file
III. Some clerks are teachers All files are animals
IV. No lecturer is a clerk Conclusions :
I. All those animals if they are sheets are also cats
a) Only I follow II. Some files that are not sheets being cats is a
b) Only II follows possibility.
c) Only III follows III. Some animals are not files
d) Only I and II follows. IV. Some animals are cats
e) None follows
a) None follows
Q 40. Statements : b) All follow
All engineers are doctors c) Only II and III follow
Some doctors are compounders d) Only IV follow
Some compounders are staffs e) None of these
Conclusions :
I. All staffs are doctors Q 43. Statements :
II. All engineers being compounders is a Some stones are diamonds
possibility All stones are black
III. All compounders being doctors is a possibility No black is precious
IV. No engineer is a staff. Conclusions :
I. All precious being stones is a possibility
a) None follows II. All diamonds being black is a possibility.
b) All follow III. Some diamonds are precious
c) Only II and III follow IV. No diamond is black
d) Only IV follow
e) None of these a) None follows
b) Only II follows
www.ibpsguide.com | www.ibpsguide.in | mock.ibpsguide.in | www.sscexamguide.com
Copyright © 2016 IBPS Guide 92
yoursmahboob.wordpress.com

c) Only I follow III. No updates are a mail.


d) Only III follows IV. Some groups are chats
e) Only I and II follows.
a) Only I and II follows
Q 44. Statements : b) Only II and III follow
Some roses are beautiful c) Only I, II and III follows
All leafs are roses d) Only II, III and IV follow
All buds are leafs e) None of these
Conclusions :
I. All roses being leafs is a possibility Q 47. Statements :
II. Some leafs are beautiful No stone is red
III. All roses being buds is a possibility No red is flower
IV. All buds are beautiful Some red are blue
Some blue are orange
a) All follow Conclusions :
b) None follows I. Some leafs are not red is a possibility
c) Only I and III follow II. All oranges are red
d) Only II and IV follow III. Some roses being leafs is a possibility.
e) None of these IV. No orange is flower

Q 45. Statements : a) Only II and III follow


All engineers are doctors b) Only I and IV follow
Some doctors are talented c) Only II, III and IV follow
Some talented are lawyers d) Only I and III follow
Conclusions : e) None of these
I. All engineers being talented is a possibility
II. Some doctors are lawyers Q 48. Statements :
III. All talented being doctors is a possibility Some tables are bikes
IV. No doctor is a lawyer No furniture is table
Some furniture is brown
a) All follows Conclusions :
b) Only either I or II and both III and IV I. There is a possibility that some bikes are not
follows furniture
c) Only either II or IV and both I and III II. Some bikes are not tables are a possibility.
follow III. Some brown is not table
d) Only either I or IV follows IV. No brown is a bike
e) None of these
a) Only I, II and III follows
Q 46. Statements : b) Only II and III follow
Some groups are mails c) Only I and IV follow
Some mails are chats d) Only II, III and IV follow
All updates are chats e) None of these
Conclusions :
I. All mails being updates is a possibility Q 49. Statements :
II. Some updates being group is a possibility Some gold are yellow
www.ibpsguide.com | www.ibpsguide.in | mock.ibpsguide.in | www.sscexamguide.com
Copyright © 2016 IBPS Guide 93
yoursmahboob.wordpress.com

All silvers are gold


All metals are silvers
Conclusions :
I. All gold being silvers is a possibility
II. There is a possibility that all metals are gold.
III. Some silvers are yellow
IV. Some metals being yellow is a possibility

a) Only I and IV follow


b) Only II and IV follow
c) Only either I or II follows
d) Only either II or IV and III follow
e) None of these.

Q 50. Statements :
Some professors are students
Some teachers are professors
Some lecturers are teachers
Conclusions :
I. All those teachers who are lecturers are also
professors.
II. No lecturer is student
III. Some student is teacher
IV. All teachers as well as all professors being
lecturers is a possibility

a) Only I and III follow


b) Either II or IV follows
c) None follows
d) Only IV follows
e) None of these

www.ibpsguide.com | www.ibpsguide.in | mock.ibpsguide.in | www.sscexamguide.com


Copyright © 2016 IBPS Guide 94
yoursmahboob.wordpress.com

Answers with Detailed Explanation:

Answer- 1. a
Answer- 2. e

Answer - 3. b

www.ibpsguide.com | www.ibpsguide.in | mock.ibpsguide.in | www.sscexamguide.com


Copyright © 2016 IBPS Guide 95
yoursmahboob.wordpress.com

Answer - 4. b
Answer - 5. a

Answer - 6. d

www.ibpsguide.com | www.ibpsguide.in | mock.ibpsguide.in | www.sscexamguide.com


Copyright © 2016 IBPS Guide 96
yoursmahboob.wordpress.com

Answer - 7. e
Answer - 8. a

Answer - 9. b

www.ibpsguide.com | www.ibpsguide.in | mock.ibpsguide.in | www.sscexamguide.com


Copyright © 2016 IBPS Guide 97
yoursmahboob.wordpress.com

Answer - 10. e
Answer - 11. d

Answer - 12. c

www.ibpsguide.com | www.ibpsguide.in | mock.ibpsguide.in | www.sscexamguide.com


Copyright © 2016 IBPS Guide 98
yoursmahboob.wordpress.com

Answer - 13. d
Answer - 14. e

www.ibpsguide.com | www.ibpsguide.in | mock.ibpsguide.in | www.sscexamguide.com


Copyright © 2016 IBPS Guide 99
yoursmahboob.wordpress.com

Answer - 15. c
Answer - 16. b

Answer - 17. a

www.ibpsguide.com | www.ibpsguide.in | mock.ibpsguide.in | www.sscexamguide.com


Copyright © 2016 IBPS Guide 100
yoursmahboob.wordpress.com

Answer - 18. e
Answer - 19. d

Answer - 20. e

www.ibpsguide.com | www.ibpsguide.in | mock.ibpsguide.in | www.sscexamguide.com


Copyright © 2016 IBPS Guide 101
yoursmahboob.wordpress.com

Answer - 21. b
Answer - 22. a

Answer - 23. e

www.ibpsguide.com | www.ibpsguide.in | mock.ibpsguide.in | www.sscexamguide.com


Copyright © 2016 IBPS Guide 102
yoursmahboob.wordpress.com

Answer - 24. b
Answer - 25. c

Answer - 26. b

www.ibpsguide.com | www.ibpsguide.in | mock.ibpsguide.in | www.sscexamguide.com


Copyright © 2016 IBPS Guide 103
yoursmahboob.wordpress.com

Answer - 27. e
Answer - 28. a

Answer - 29. c

www.ibpsguide.com | www.ibpsguide.in | mock.ibpsguide.in | www.sscexamguide.com


Copyright © 2016 IBPS Guide 104
yoursmahboob.wordpress.com

Answer - 30. a
Answer - 31. e

Answer - 32. d

www.ibpsguide.com | www.ibpsguide.in | mock.ibpsguide.in | www.sscexamguide.com


Copyright © 2016 IBPS Guide 105
yoursmahboob.wordpress.com

Answer - 33. a
Answer - 34. d

Answer - 35. c

www.ibpsguide.com | www.ibpsguide.in | mock.ibpsguide.in | www.sscexamguide.com


Copyright © 2016 IBPS Guide 106
yoursmahboob.wordpress.com

Answer - 36. d
Answer - 37. a

Answer - 38. e
Answer - 39. a

www.ibpsguide.com | www.ibpsguide.in | mock.ibpsguide.in | www.sscexamguide.com


Copyright © 2016 IBPS Guide 107
yoursmahboob.wordpress.com

Answer - 40. c
Answer - 41. a

Answer - 42. c

www.ibpsguide.com | www.ibpsguide.in | mock.ibpsguide.in | www.sscexamguide.com


Copyright © 2016 IBPS Guide 108
yoursmahboob.wordpress.com

Answer - 43. e
Answer - 44. c

Answer - 45. c

www.ibpsguide.com | www.ibpsguide.in | mock.ibpsguide.in | www.sscexamguide.com


Copyright © 2016 IBPS Guide 109
yoursmahboob.wordpress.com

Answer - 46. a
Answer - 47. d

Answer - 48. a

www.ibpsguide.com | www.ibpsguide.in | mock.ibpsguide.in | www.sscexamguide.com


Copyright © 2016 IBPS Guide 110
yoursmahboob.wordpress.com

Answer - 49. e
Answer - 50. d

9. BLOOD RELATIONS

Q 1 – 5).Study the following information carefully and answer the questions given below :
There is a family of seven members Shivika, Raman, Deepika, Anuj, Vijay, Jayesh, and Priti. They all
appeared in an IQ test to check their intelligence. There are three females in the family and two married
couples. Priti is the most intelligent member in the family. Raman is father of Vijay and he is more intelligent
than his son. Deepika has son and one daughter and she is more intelligent than her husband. The father of
Raman is more intelligent than Raman. Vijay, the grandson of Jayesh, is the least intelligent in the family.
Jayesh is the second most intelligent in the family. The mother of Raman is less intelligent than Raman. The
grandmother of Priti has two sons and one of them is Anuj, who is more intelligent than his brother but less
intelligent than his sister – in – law. Shivika is not more intelligent than her sons. The father of Priti is
married to Deepika.
Q 1) Who among the following is wife of Raman?
a) Jayesh b) Deepika c) Anuj d) Priti

Q 2) Who among the following is the third least intelligent in the family?
a) Shivika b) Raman c) Deepika d) Anuj

Q 3) How is Priti related to Anuj?


a) Niece b) Nephew c) Son d) Can‘t be determined

www.ibpsguide.com | www.ibpsguide.in | mock.ibpsguide.in | www.sscexamguide.com


Copyright © 2016 IBPS Guide 111
yoursmahboob.wordpress.com

Q 4) The one who is the most intelligent is not a


a) Daughter b) Granddaughter c) Wife d) Sister

Q 5) How is Shivika related to Deepika?


a) Mother – in – law b) Mother c) Father d) Daughter

Q 6) Ravikishan said, ―The girl is the wife of the grandson of my mother‖. How is Ramakrishna related to the
girl?
a) Father b) Cannot be determined c) Grandfather d) Daughter – in – law

Q ( 7 – 11) In a family of eight persons – C, D, E, F, G, H, I & J – there are four males and four females.
There are three married couples and two persons are unmarried. Each of them reads a different newspaper, viz
Wall Street Journal, Washington Journals, Financial Times, Economic Times, Business Standard, Indian
Express, The Hindu & Deccan herald. No couples reads both the versions of same newspaper. In the family of
two generations, each male member except J has two brothers and one sister. I is the mother – in – law of E
who reads Washington Journals. D who reads Financial Times, is the daughter – in – law of J. G who reads
Business Standard is the unmarried brother of H, who does not read The Hindu. No female reads Economic
Times or Indian Express. F is the brother – in – law of E but he does not read The Hindu, Deccan Herald or
Indian Express. C does not read Indian Express wall street journal and the hindu. J has no son – in – law. H is
D‘s sister – in – law. And does not read wall street journal.

Q 7) Who among the following reads Economic Times?


a) Can‘t say b) F c) C d) E

Q 8) F reads which of the following magazines?


a) Can‘t say b) Wall Street Journal c) Business Standard d) Deccan Herald

Q 9) Which of the following pairs of persons does not represent the couples?
1–J&I 2–C&E 3–F&D
a) Can‘t say b) Only 1 c) Only 2 d) All of the above pairs of persons represent the couples

Q 10) How many sons does J have?


a)Two b)Three c)One d)Can‘t say

Q 11)How is C related with H?


a)Father b) Brother c) Husband d) Can‘t say

Q (12 – 15) There are 7 members in a family – Prajakta, Ayush, Rakesh, Shyam, Anjali, Raveena& Ravi.
There are only 3 females in the family – Prajakta, Anjali &Raveena. The family consists of three couples, four
pairs of brother and sister, 3 brothers and one pair of sisters. Raveena is not the sister of Prajakta or Ayush.
Shyam is not married to either Raveena or Anjali. Raveena‘s husband is neither Ayush nor Shyam. Rakesh is
the brother of both Ayush and Prajakta. Ayush is a brother of Ravi. Ravi is unmarried.
Q 12) Who is Shyam‘s wife?
a) Either Prajakta or Anjali b) Anjali c) Prajakta d) Raveena

www.ibpsguide.com | www.ibpsguide.in | mock.ibpsguide.in | www.sscexamguide.com


Copyright © 2016 IBPS Guide 112
yoursmahboob.wordpress.com

Q 13) Which of the following is not a pair of brother and sister?


a) Prajakta&Ayush b) Prajakta&Rakesh
c) Anjali &Rakesh d) Anjali &Shyam

Q 14) Which of the following is a pair of husband and wife?


a) Rakesh&Raveena b) Shyam& Anjali
c) Rakesh& Anjali d)Ayush&Prajakta

Q 15) Which of the following statements is false?


a) Rakesh is Raveena‘s husband
b) Prajakta is Raveena‘s sister – in – law
c) Rakesh has the same relation with Anjali as Shyam has with Raveena
d) Prajakta is not Shyam‘s sister.

Q (16 – 20) In a family of six persons – L, M, N, O, P & Q – there are three males and three females. There
are two married couples and two persons are unmarried. Each one of them watch different news channel viz.
AajTak, Newshour, India TV, CNBC, Ndtv and IBN. P, who watches Newshour, is the mother – in – law of
L, who is wife of N. O is the father of Q and he does not watch AajTak or IBN. M watches Ndtv and is the
sister of Q, who watches India TV. N does not watch IBN.

Q 16) Who among the following watches AajTak?


a) N b) O c) L d) Data inadequate

Q 17) How is Q related to P?


a) Daughter b) Brother c) Son d) Data inadequate

Q 18) Which of the following is one of the married couples?


a) O – M b) O – P c) M – Q d) P – Q

Q 19) Which of the following news channel does L watches?


a) AajTak b) Ndtv c) CNBC d) None of these

Q 20) How many sons does P have?


a) Four b) Three c) Two d) One

Q (21 – 25) A, B, C, D, E, F & G are seven members of a family. Each of them has a different profession –
Businessman, Teacher, Advocate, Engineer, Architect, Chartered Accountant & Banker – and their incomes
are different. There are two married couples in a group. C is the Businessman and earns more than the
Engineer and the Advocate. E is married to Chartered Accountant and she earns the least. No lady is either
Advocate or Engineer. B, the Teacher, earns less than A, the Banker. G is married to B and he earns more than
D & A. F is not Advocate. The Chartered Accountant earns less than the Advocate but more than the Banker.

www.ibpsguide.com | www.ibpsguide.in | mock.ibpsguide.in | www.sscexamguide.com


Copyright © 2016 IBPS Guide 113
yoursmahboob.wordpress.com

Q 21) Who earns the maximum in the family?


a) F b) G c) C d) D

Q 22) Which of the following is a pair of married couple?


a) CE b) FE c) BE d) None of these

Q 23) What is A‘s position from the top when they are arranged in descending order of their incomes?
a) Second b) Fourth c) Third d) None of these

Q 24) What is the profession of F?


a) Engineer b) Chartered Accountant
c) Engineer or Chartered Accountant d) Data inadequate

Q 25) At least how many male members are there in the family?
a) Two b) four c) three d) Five

Q (26 – 30) In a family of three generations, there are two pairs of couples and two pairs of brothers and
sisters. There are three Reporters, Two Lawyers, one Architect and one Doctor in the family. The name of the
persons in the family are D, E, F, G, H, I & J.
1 – No females in the family are Reporter.
2 – J is not a Doctor
3 – No married persons in the family are Lawyers.
4 – Ratio of the monthly salaries of E : F : G is 1 : 3 : 5
5 – Ratio of the monthly salaries of D : H : I is 2 : 1 : 3
6 – Ratio of the monthly salaries of E : D : J is 1 : 4 : 7
7 –D is the son of E. H is the son of F and his wife is E. F is a Reporter. F & J are the eldest couple in the
Family. G is the daughter of H . I is the sister – in – law of E.

Q 26) How is E related to J?


a) Mother – in – Law b) Daughter – in – Law c) Daughter d) Can‘t say

Q 27) Who among the following persons is not a Reporter?


a) D b) F c) H d) I

Q 28) Who among the following gets the least monthly salary?
a) D b) F c) H d) E

Q 29) Which of the following shows the correct order of the salaries of all the peersons?
a) J>I>G>D>F>H>E b) J>I>G>D>F>E>H c) J>I>G>D>E>H>F d) J>I>G>E>F>H>D

Q 30) Which pair given below are the sons of E?


a) E & F b) E & C c) D & H d) E & D

www.ibpsguide.com | www.ibpsguide.in | mock.ibpsguide.in | www.sscexamguide.com


Copyright © 2016 IBPS Guide 114
yoursmahboob.wordpress.com

Q (31 – 33) Ramya& Rajesh are a married couple having two daughters named Reetika&Devika. Devika is
married to a man who is the son of Siya and Jeevan. Saumya is the daughter of Arav. Ganga who is Arav‘s
sister is married to Sunil and has two sons Ayush&Ankit. Ankit is the grandson of Siya and Jeevan.

Q 31) How is Ganga related to Devika?


a) Sisiter – in – Law b) Sister c) Aunt d) None of these

Q 32) What is the relationship between Ayush&Saumya?


a) Uncle – Niece b) Father – Daughter c) Husband – Wife d) Cousins

Q 33) Which of the following is true?


a) Ayush is the son of Reetika b) Saumya is the cousin of Ganga
c) Siya is Sunil‘s mother – in – Law d) Jeevan is Devika‘s Maternal Uncle

Q 34 – 36) Rita, Ramesh&Rajender are children of Mr. and Mrs. Sharma.


a) Renuka, Rajeev &Santosh are children of Mr& Mrs. Srivastava.
b) Santosh& Rita are a married couple and Kumar &Sanjeev are their children.
c) Geetika&Lokesh are children of Mr& Mrs. Kohli.
Geetika is married to Rajender and has three children named Reema, Abhay& Ravi.

Q 34) How is Ramesh related to Kumar?


a) Father – in – Law b) Brother – in – Law c) Cousin d) Maternal Uncle

Q 35) What is Sanjeev‘s surname?


a) Kohli b) Srivastava c) Sharma d) Rajender

Q 36) Renuka is Sanjeev‘s….


a) Sister – in – Law b) Sister c) Cousin d) Aunty

Q (37 – 41) In a group there are 6 persons U, V, W, X, Y & Z. W is the sister of Z. V is the brother of Y‘s
husband. X is the father of U and grandfather of Z. There are two fathers, three brothers and a mother in the
group.

Q 37) Who is the mother?


a) U b) V c) X d) Y

Q 38) Who is Y‘s husband?


a) V b) W c) U d) Y

Q 39) How many male members are there in the group?


a) 3 b) 1 c) 2 d) 4

Q 40) How is Z related to Y?

www.ibpsguide.com | www.ibpsguide.in | mock.ibpsguide.in | www.sscexamguide.com


Copyright © 2016 IBPS Guide 115
yoursmahboob.wordpress.com

a) Son b) Uncle c) Husband d) Daughter

Q 41) Which of the following is a group of brothers?


a) UVZ b) UVX c) VZW d) VXZ

Q (42 – 45) SudhirAnand has threechildren: Radhika, Raman and Sudeep. Sudeep married Rupa, the eldest
daughter Mr. and Mrs. Malhotra. The Malhotras married their youngest daughter to the eldest son of Mr. and
Mrs. Bushan and they had two children Rajveer and Sameeksha. The Malhotras have two more children,
Rakesh and Rukmani, both elder to Shiela. Sahil and Abhijeet are sons of Sudeep and Rupa. Rita is the
daughter of Rajveer.

Q 42) How is Mrs. Malhotras related to Sudeep?


a) Aunt b) Mother – in – Law c) Mother d) Sister – in – Law

Q 43) What is the surname of Sahil?


a) Bushan Malhotra c) Anand d) Raman

Q 44) What is the surname of Rita?


a) Bushan b) Malhotra c) Anand d) None of these

Q 45) How is Abhijeet related to Rupa‘s father?


a) Son – in – Law b) Cousin c) Son d) Gradnson

Q (46 – 50) U, V, W, X, Y & Z are members of a family consisting of three generations. The family consists
of two pairs of couples. The family consists of only three females. The oldest member in the family is a
female but the youngest one is a male. No two persons are of the same age. One day they visited a restaurant
and sat around a table facing towards the centre. X sat opposite the person who occupies the third place when
their ages are considered in descending order. U is the grandfather of Z and sat on the immediate right of X,
who is the father of W but not the husband of Y. Persons of the same generation sat opposite each other. V sat
on the immediate right of a female. Z is not the youngest.

Q 46) The youngest person sat between which of the following two persons?
a) X and Y b) V and U c) X and V d) Data inadequate

Q 47) The only person who sat between Y and Z is not the person who is
a) The mother of Z b) The daughter – in – Law of U
c) The wife of X d) The second in position in descending order of age

Q 48) Which of the following shows the correct order rightward from the position of U?
a) U, Z, V, Y, X, W b) U, X, Y, W, V, Z c) U, X, W, Y, V, Z d) None of these

Q 49) The person who are adjacent to Y, are


a) V and Z b) W and X c) W and V d) Data inadequate

www.ibpsguide.com | www.ibpsguide.in | mock.ibpsguide.in | www.sscexamguide.com


Copyright © 2016 IBPS Guide 116
yoursmahboob.wordpress.com

Q 50) Which of the following statements is/are not true?


1 – One of the couples is U and Y
2 – X is the son of Y whereas V is the wife of X
3 – W is the brother of Z
4 – Y, V and Z are females.
a) All 1, 2, 3 and 4 b) Only 3 c) Only 3 and 4 d) None

BLOOD RELATIONS (Answers with Detailed Explanation)

Q (1 – 5) The intelligence of seven members of the family in descending order as per the IQ test is given
below:
Priti(P) > Jayesh(J) > Deepika(D) > Anuj(A) > Raman(R) > Shivika(S) > Vijay(V)

Male represented by ‗+‘ Female represented by ‗–‗

Q 1) b) Q 2) b) Q 3) a) Q 4) c) Q 5) a)

Q 6) b)

Q 7 – 11) The tree diagram to show the relations between the members is given below:

www.ibpsguide.com | www.ibpsguide.in | mock.ibpsguide.in | www.sscexamguide.com


Copyright © 2016 IBPS Guide 117
yoursmahboob.wordpress.com

Hence the table of who reads which newspaper is given below with positive(+) and negative(-) sign showing
male and female members of the family.
WAJ – Wall street, WSJ – Washington journal, FT – Financial times, ET – Economic times,
BS –Business standard, IE –Indian express, TH – The Hindu, DC – Deccan Herald

7) c) 8) c) 9) d) 10) b) 11) b)

Q (12-15):

Q 12) c) Q13)c) Q14)a) Q 15) c)

Q 16 – 20) The blood relation of the six members are given below:

www.ibpsguide.com | www.ibpsguide.in | mock.ibpsguide.in | www.sscexamguide.com


Copyright © 2016 IBPS Guide 118
yoursmahboob.wordpress.com

Q 16) a) Q 17) c) Q 18) b) Q 19) d) Q 20) c)

Q 21 – 25) On the basis of the given statements we can conclude that

The two married couples are 1) G(+) B(–) and 2) E and F or D( not sure as per the info
given). The table below shows the profession and specifies the male and female members of the family.

Q 21) c) Q 22) d) Q 23) d) Q 24) c) Q 25) b)

Q 26 – 30) Let us arrange the information given in statement 7. D and H are the sons of E and F respectively.
Hence D and H are males. E is the wife of H, i.e. E is the mother of of D. F and J are eldest couple but we
have still no information about the sex of F and J. From clue 1, no female is a Reporter; hence F is a male and
J is a female. I is sister – in – law of E, i.e. I is the sister of H. We get this tree drawn below:

www.ibpsguide.com | www.ibpsguide.in | mock.ibpsguide.in | www.sscexamguide.com


Copyright © 2016 IBPS Guide 119
yoursmahboob.wordpress.com

Q 26) b) Q 27) d) Q 28) d) Q 29) a) Q 30) c)

Q (31 – 33) The following diagram drawn below shows the relations of the given people:

Q 31) a) Since, Devika is wife of Arav and Ganga is sister of Arav, therefore, Ganga is the siter – in – law of
Arav, therefore, Ganga is the sister – in – law of Devika.

Q 32) d) Since, Ayush is son of Ganga and Saumya is daughter of Arav and Arav is brother of Ganga,
therefore Ayush and Saumya are cousins.

Q 33) c) Since, Siya is mother of Ganga who is the wife of Sunil. Therefore, Siya is Sunil‘s mother – in – law.

www.ibpsguide.com | www.ibpsguide.in | mock.ibpsguide.in | www.sscexamguide.com


Copyright © 2016 IBPS Guide 120
yoursmahboob.wordpress.com

Q 34) d) Q 35) b) Q 36) d)


Q (37 – 41) The given information can be summarized as follows:

Q 37) d) U‘s wife Y is the mother.


Q 38) c) U is the husband of Y.
Q 39) d) Clearly there are four male members U, V, X andZ
Q 40) c) Clearly, Z is the son of Y
Q 41) a) U, V are brothers of each other while Z is the brother of W.

Q (42 – 45)

www.ibpsguide.com | www.ibpsguide.in | mock.ibpsguide.in | www.sscexamguide.com


Copyright © 2016 IBPS Guide 121
yoursmahboob.wordpress.com

Q (46 – 50) From the given information we can make our task easy. First we find that if U is the grandfather
of Z, it means Z is a person of the lowest generation. Again, since persons of same generation sat opposite
each other, X, who sat adjacent to U, is not from the generation of U. But X is the father of X. This implies X
is from the middle generation and W from the lowest generation. Again, since X is not the husband of Y, U is
the husband of Y and X is the husband of V. Thus, we get the following tree.

As per the information it is given that there are three females in the family. This implies that either W or Z is a
female. Now, let us fix their seating arrangement. From the given information we can also conclude that U sat
on the immediate right of X. Again, since V and X are of the same generation, this implies that V sat opposite
each other. Thus, we get the following arrangement.

Again, from the given information we get that Z is not that youngest. This implies W is the youngest(because
only W and Z are from lowest generation). Now, we also find out from the given information that the
youngest one is male. This implies that W is a male and Z is a female. Now, let us arrange the persons in
descending order of their ages. Since the oldest member is a female, this implies Y is older than U. Again, we
get that V occupies the third position. This implies v is older than X. And, since W is the youngest. Hence Z is
older than W.
Y>U>V>X>Z>W
Thus, we can conclude that V sat on the immediate right of Z(because it is given that V sat on the immediate
right of a female) and W sat on the immediate left of X.
Q 46) a) Q 47) d) Q 48) c) Q 49) d) Q 50) d)

10. CRITICAL REASONING

Q 1) In the country A, about $5,200 per person per year is spent on health care, while in country B the amount
is about half that. A recent study indicated that middle-aged white Americans have a significantly higher rate
of diabetes and heart disease than do middle-aged white Britons. Even after eliminating from the study the
lifestyle differences of diet, exercise, smoking, and drinking, the data showed that the Americans have poorer
health than their British counterparts. The statements above, if true, best support which of the following
assertions?
a) Health care spending in the country A should be reduced by 50%.
b) More expensive health care causes a higher incidence of certain diseases.
c) The money spent on health care in the country A is not being used effectively.
d) Something other than diet, exercise, smoking, and drinking must account for the difference in health for
the twogroups in the study.

www.ibpsguide.com | www.ibpsguide.in | mock.ibpsguide.in | www.sscexamguide.com


Copyright © 2016 IBPS Guide 122
yoursmahboob.wordpress.com

Q 2) In 2003, the Rendezvous Hits Record Company spent 40% of its total budget on the production of ten
albums, 30% of its budget on the marketing of these albums, and the remainder of its budget on overhead
costs. In the same year, the Song K-T Record Company spent 20% of its total budget on the production of 10
albums and 60% of itsbudget on the marketing of these albums. Making Hits sold a total of 800,000 copies of
the ten records it producedin 2003, while the Song K-T sold a total of 1,600,000 copies of the ten records it
produced in 2003. Assumingeach company met its budget, which of the following conclusions is best
supported by the information givenabove?
a) The amount of money spent on marketing is directly related to the number of copies sold.
b) Rendezvous Hits spent more money on the production of its albums in 2003 than did the Song K-T.
c) Song K-T‘s total revenue from the sale of albums produced in 2003 was higher than that of Rendezvous
Hits.
d) In 2003, Rendezvous Hits spent a larger percentage of its budget on overhead costs than did the Song K-T.

Q 3) A recent research study of undergraduate students analyzed the effects of music on human emotions.
Each of the 200 participants attended at least 1 two-hour concert of classical music per week over the course
of 12 weeks oftheir spring semester. At the end of the experiment, all of the students filled out a questionnaire
assessing their emotional state. Based on the results of the questionnaires, all of the 10 students who attended
the greatestnumber of concerts reported lower stress levels and higher satisfaction with their lives. Also, most
of the 20students who attended the fewest number of concerts reported below-average levels of emotional
comfort. Whichof the following must be true based on the evidence presented above?
a) Most of the 200 participants improved their emotional state and lowered their stress levels.
b) During each week of the experiment, the participants spent at least 2 hours less on their academic work as
a result of concert attendance.
c) Listening to classical music for at least 2 hours per week improves the emotional well-being of the
majority ofyoung adults.
d) More than 6 participants attended at least 14 concerts during the course of the experiment.

Q 4) In Greenland, from 1999 to 2004, the total consumption of fish increased by 4.0 percent, and the total
consumption of poultry products increased by 8.0 percent. During this time, the population of Greenland
increased by 6 percent,in part due to new arrivals from surrounding areas. Which of the following can one
infer based on the statementsabove?
a) For new arrivals to Eastland between 1999 and 2004, fish was less likely to be a major part of families‘
diet than was poultry
b) In 1999, the residents of Greenland consumed twice as much poultry as fish.
c) The per capita consumption of poultry in Greenland was higher in 2004 than it was in 1999.
d) Between 1999 and 2004, both fish and poultry products were a regular part of the diet of a significant
proportion of Greenland residents.

Q 5) Everyone who has graduated from Top Notch school has an intelligence quotient (IQ) of over 120.
Moststudents with an IQ of over 120 and all students with an IQ of over 150 who apply to one or more
Vidyamandiruniversities are accepted to at least one of them. The statements above, if true, best support which
of thefollowing conclusions?
a) Every graduate of TopNotch School with an IQ of 150 has been accepted to at least one of the
Vidyamandir schools.
b) If a person is a high-school graduate and has an IQ of less than 100, he or she could not have been a
student at Top Notch school.
c) If a person has an IQ of 130 and is attending an Ivy-League school, it is possible for him or her to have
graduated from TopNotch High School.
d) At least one graduate from TopNotch high school who has applied to at least one VidyaMandir university
has been accepted to one of them

www.ibpsguide.com | www.ibpsguide.in | mock.ibpsguide.in | www.sscexamguide.com


Copyright © 2016 IBPS Guide 123
yoursmahboob.wordpress.com

Q 6) Advocates insist that health savings accounts are an efficient method to reduce medical expenses.
However, widespread adoption of these accounts will soon undermine the public‘s health. One reason for this
is that most people will be reluctant to deplete their accounts to pay for regular preventive examinations, so
that in many cases a serious illness will go undetected until it is far advanced. Another reason is that poor
people, who will not be able to afford health savings accounts, will no longer receive vaccinations against
infectious diseases. The statements above, if true, most support which of the following?
a) Wealthy individuals will not be affected negatively by health savings accounts.
b) Private health insurance will no longer be available.
c) Most diseases are detected during regular preventive examinations.
d) Some people without health savings accounts are likely to contract infectious diseases.

Q 7) Indira Gandhi Airport and Shivaji Airport have the same number of flight departures each day. Indira
Gandhi Airport experiences 26 departure delays per 100 flights, while Shivaji Airport experiences 20
departure delays per 100flights. When delays caused by bad weather are disregarded, Indira Gandhi Airport
has 5 fewer departure delays per 100 flights than Shivaji Airport does. Which of the following conclusions is
best supported by the informationgiven above?
a) Bad weather causes a greater number of departure delays at Indira Gandhi Airport than at Shivaji Airport.
b) On average, the weather at Indira Gandhi Airport is worse than it is at Shivaji Airport.
c) Mechanical problems cause a greater number of delays at Shivaji Airport than at Indira Gandhi Airport.
d) The fleet of airplanes leaving from Shivaji Airport is better equipped to handle inclement weather than the
fleet of airplanes leaving from Indira Gandhi Airport.

Q 8) The head baker at Paradise can either purchase flour in-person from the local flour mill, Charles Local
Mill, or order a shipment of flour from an out-of-state mill, Isildiur‘s Mill. The cost of the flour from
Isildiur‘sMill is 10 percent less than the cost of the flour from Charles Local Mill. Even after shipping and
handlingfees are added, it is still cheaper to order flour that has to be shipped from Isildiur‘s than to buy flour
locally fromCharles. The statements above, if true, best support which of the following assertions?
a) Production costs at Isildiur‘s Mill are 10 percent below those at Charles Local Mill.
b) Buying flour from Isildiur‘sMill will eliminate 10 percent of the local flour mill jobs.
c) The shipping and handling fees for a batch of flour purchased from Isildiur‘s Mill are less than 10 percent
of the cost of an identical batch of flour purchased from Charles Local Mill.
d) The shipping and handling fees for a batch of flour purchased from Isildiur‘s Mill are more than 10
percent of the cost of Isildiur‘s flour.

Q 9) Box office receipts for independent movies for the first half of this year have increased by 20 percent
over the total receipts for independent movies for all of last year. Last year, 50 independent movies were
released, while so far this year only 20 independent movies have been released. The number of independent
movies slated for release in the second half of this year is roughly equal to the number released so far. If the
statements above are true, which of the following must be true?
a) The total box office receipts for independent movies this year will be significantly more than 20 percent
greater than the receipts for independent movies last year.
b) The number of independent movies released in the first half of this year is equal to the number released in
the first half of last year.
c) The price of a movie ticket has not increased since last year.
d) The average revenues of the independent films released during the first half of this year is greater than that
of all independent films released last year.

Q 10) As many as 90,000 people die each year due to medical error. In a campaign to reduce lethal errors,
thousands of hospitals introduced six key changes, including rapid-response teams, re-checks of patient
medication, and new guidelines for preventing infection. The campaign estimated that, over an 18-month

www.ibpsguide.com | www.ibpsguide.in | mock.ibpsguide.in | www.sscexamguide.com


Copyright © 2016 IBPS Guide 124
yoursmahboob.wordpress.com

period, more than 150,000 lives were saved as a direct result of the program. Which of the following can be
most properly inferred from theabove statements?
a) Doctors and nurses should be more careful when doing their jobs.
b) The campaign saved all of the people who otherwise would have died due to medical error in that time
period.
c) In the future, no one will die because of medical error.
d) If the campaign had not been implemented, more than 150,000 people might have died during the 18-
month period due to medical error.

Q 11) Educator: Like any other difficult pursuit, music requires intense study and practice in order for one to
become proficient. But many school music programs encourage only children who demonstrate early aptitude
to continue studying music, while children who are not especially musical are directed towards other
activities. Having learned to think of themselves as musically inept, these children do not devote any time to
music and thus deprive themselves of the opportunity to develop a latent talent. The educator‘s statements, if
true, would best support which of the following conclusions?
a) Music education should not devote special attention to talented students.
b) Everyone has the potential to learn music.
c) Talent is not always apparent at an early age.
d) Children are particularly sensitive to criticism from adults.

Q 12) Charter schools are independent public schools that are given greater autonomy in exchange for
increasedaccountability. Charter school operators are freed from many of the regulations of the traditional
public schoolbureaucracy, thereby allowing them to pursue more innovative educational ideas than non-
charter public schoolscan pursue. At the same time, charter schools are held accountable for achieving specific
educational outcomes and are closed down if those outcomes are not met. Which of the following, if true, best
supports the assertion that students attending charter schools will, on average, perform better on assessments
of writing ability than students attending traditional public schools?
a) Students who attend schools that emphasize order and discipline perform worse on assessments of
writing abilitythan students who attend schools that do not emphasize order and discipline.
b) The majority of students who score in the 99th percentile on assessments of writing ability attend
charter schools.
c) Public schools that operate outside of the traditional public school bureaucracy spend more time
teaching studentswriting than do traditional public schools.
d) Students who attend schools that are allowed to experiment with their writing curricula perform better
onassessments of writing ability than students who attend schools that have less flexible curricula.

Q 13) The new heart scans offer patients significant benefits. They can be completed in a fraction of the time
required for an angiogram, with no recovery time necessary. Furthermore, the scans are more sensitive and
can identify problem areas that an angiogram might not perceive. However, heart scans use more radiation
than most diagnostic procedures, and can cause undue concern over and treatment for the harmless
abnormalities often picked up by such sensitive technology. Which of the following conclusions is best
supported by the statements above?
a) A heart scan is safer than an angiogram procedure.
b) Patients should not be concerned about heart abnormalities that appear in a heart scan.
c) A heart scan could result in indirect harm by causing a patient to undergo risky, unnecessary procedures.
d) An angiogram is the more appropriate of the two procedures for most patients.

Q 14) In the Last April, in an attempt to lower the number of traffic fatalities, the state legislature passed its
―Click It or Ticket‖ law. Under the new law, motorists can be pulled over and ticketed for not wearing their
seat belts, even if an additional driving infraction has not been committed. Lawyers and citizens‘ groups are
already protesting the law, saying it unfairly infringes on the rights of the state‘s drivers. Law enforcement
www.ibpsguide.com | www.ibpsguide.in | mock.ibpsguide.in | www.sscexamguide.com
Copyright © 2016 IBPS Guide 125
yoursmahboob.wordpress.com

groups counter these claims by stating that the new regulations will save countless additional lives. Which of
the following inferences is best supported by the passage above?
a) Prior to the ―Click It or Ticket‖ law, motorists could not be stopped simply for not wearing a seat belt.
b) The ―Click It or Ticket‖ law violates current search and seizure laws.
c) Laws similar to ―Click It or Ticket‖ have effectively reduced traffic fatalities in a number of states.
d) The previous seatbelt laws were ineffective in saving lives.

Q 15) The popular notion that a tree‘s age can be determined by counting the number of internal rings in its
trunk is generally true. However, to help regulate the internal temperature of the tree, the outermost layers of
wood of the Brazilian ash often peel away when the temperature exceeds 95 degrees Fahrenheit, leaving the
tree with fewer rings than it would otherwise have. So only if the temperature in the Brazilian ash‘s
environment never exceeds 95 degrees Fahrenheit will its rings be a reliable measure of the tree‘s age. Which
of the following is an assumption on which the argument above depends?
a) The growth of new rings in a tree is not a function of levels of precipitation.
b) Only the Brazilian ash loses rings because of excessive heat.
c) Only one day of temperatures above 95 degrees Fahrenheit is needed to cause the Brazilian ash to lose a
ring.
d) The number of rings that will be lost when the temperature exceeds 95 degrees Fahrenheit is not
predictable.

Q 16) The Department of Homeland Security has proposed new federal requirements for driver‘s licenses that
would allow them to be used as part of a national identification system. Using licenses for purposes not
directly relatedto operating a motor vehicle is un-American because it would require U.S. citizens to carry the
equivalent of ―papers.‖ Such a requirement would allow the government to restrict their movements and
activities in the mannerof totalitarian regimes. In time, this could make other limits on freedom acceptable.
The author assumes which ofthe following?
a) The next presidential election will be dishonest, as has happened in eastern European countries.
b) The government will soon start curtailing the activities of those it considers ―dissidents.‖
c) Blanket restrictions on law-abiding individuals are contrary to the traditions of American culture and law.
d) The majority of Americans are not willing to give up their right to travel and move about without
identification.

Q 17) Since the new publisher took control, a news magazine‘s covers have featured only models and movie
stars. Previously, the covers had displayed only politicians, soldiers, and business leaders. A leading gossip
columnist claimed that the changes made the magazine relevant again. However, many newspaper editorials
disagreed and suggested that the new publisher is more interested in boosting sales than in reporting important
news events. Which of the following is an assumption necessary for the argument made by the gossip
columnist‘s opponents?
a) The charitable activities of models and movie stars often focus public attention on pressing problems.
b) Final authority for choosing the cover subject of the magazine lies with the publisher.
c) A magazine can boost sales while highlighting the coverage of important world leaders.
d) Some of the movie stars featured are now running for political office.

Q 18) In response to the increasing cost of producing energy through traditional means, such as combustion,
many utility companies have begun investing in renewable energy sources, chiefly wind and solar power,
hoping someday to rely on them completely and thus lower energy costs. The utility companies claim that
although these sources require significant initial capital investment, they will provide stable energy supplies at
low cost. As a result, these sources will be less risky for the utilities than nonrenewable sources, such as gas,
oil, and coal, whose prices can fluctuate dramatically according to availability. The claim of the utility
companies presupposes which of the following?
a) The public will embrace the development of wind and solar power.
www.ibpsguide.com | www.ibpsguide.in | mock.ibpsguide.in | www.sscexamguide.com
Copyright © 2016 IBPS Guide 126
yoursmahboob.wordpress.com

b) No new deposits of gas, oil, and coal will be discovered in the near future.
c) Weather patterns are consistent and predictable.
d) The necessary technology for conversion to wind and solar power is not more expensive than the
technologyneeded to create energy through combustion.

Q 19) The media claim that the economy is entering a phase of growth and prosperity. They point to lower
unemployment rates and increased productivity. This analysis is false, though. The number of people filing for
bankruptcy has increased every month for the last six months, and bankruptcy lawyers report that they are
busier than they have been in years. Which of the following is an assumption on which the argument depends?
a) Unemployment rates are not useful indicators of growth and prosperity.
b) Economic growth cannot be measured in terms of productivity.
c) Legislation has not been recently passed to make legal bankruptcy easier to obtain.
d) There has not been an increase in the number of bankruptcy lawyers.

Q 20) In order to save money, some of Company X‘s manufacturing plants converted from oil fuel to natural
gas last year, when the cost of oil was more than the cost of natural gas. Because of a sudden, unexpected
shortage, however, natural gas now costs more than oil, the price of which has fallen steeply over the past
year. The cost of conversion back to oil would more than negate any cost savings in fuel. So Company X‘s
fuel costs this year will be significantly higher than they were last year. Which of the following is an
assumption on which the argument above depends?
a) Company X does not have money set aside for the increased costs of fuel.
b) The increase in the cost of fuel cannot be offset by reductions in other operating expenses.
c) The price of natural gas will never again fall below that of oil.
d) The cost of fuel needed by those of Company X‘s plants that converted to natural gas is not less than the
cost offuel needed by those plants still using oil.

Q 21) Advertisement: Hansraj Oculars are the newest in binocular technology for the professional bird
watcher. For a price comparable to that of traditional binoculars, Hansraj Oculars are specially designed with
the features demanded by birding enthusiasts: they are lightweight, compact and extremely durable. So, use
Hansraj Oculars on your next bird watching excursion, and see some of the world‘s rarest species in all of
their beautiful plumage. Which of the following is an assumption on which the author of the advertisement
relies?
a) Hansraj Oculars weigh less than traditional binoculars.
b) Hansraj Oculars should be used only by bird watchers.
c) The reader will travel through the habitats of rare species during his or her next bird watching trip.
d) Hansraj Oculars are similar in cost to traditional binoculars.

Q 22) Every year many people become ill because of airborne mold spores in their homes. After someone
becomes ill, specialists are often hired to eradicate the mold. These specialists look in damp areas of the
house, since mold is almost always found in places where there is substantial moisture. If one wishes to avoid
mold poisoning, then, one should make sure to keep all internal plumbing in good condition to prevent leakage
that could serve as a breeding ground for mold. Which of the following is an assumption on which the
argument depends?
a) Mold itself does not create moisture.
b) Most homeowners know enough about plumbing to determine whether theirs is in good condition.
c) Mold cannot grow in dry areas.
d) No varieties of mold are harmless.

Q 23) Smoking is a known cause of certain serious health problems, including emphysema and lung cancer.
Now, an additional concern can be added to the list of maladies caused by smoking. A recent study surveyed

www.ibpsguide.com | www.ibpsguide.in | mock.ibpsguide.in | www.sscexamguide.com


Copyright © 2016 IBPS Guide 127
yoursmahboob.wordpress.com

both smokers and nonsmokers, and found that smokers are significantly more anxious and nervous than
nonsmokers. Which of the following is an assumption on which the argument rests?
a) Anxiety and nervousness can lead to serious health problems.
b) Anxiety and nervousness do not make individuals more likely to start smoking.
c) Equivalent numbers of smokers and nonsmokers were surveyed for the study.
d) Smokers are aware of the various health problems attributed to smoking, including lung cancer and
emphysema.

Q 24) A recent article stated that only 5.5% of colleges in country A grant the majority of their degrees in the
liberal arts. Citing this, a reader wrote to lament that this was further evidence of the decline of academic rigor
in country A‘s high school education. Which of the following is an assumption on the part of the reader?
a) The percentage of country A colleges granting liberal arts degrees would continue to drop.
b) All colleges should grant the majority of their degrees in the liberal arts.
c) Most post-secondary scientific, engineering, and vocational training does not involve as much academic
rigor asliberal arts training.
d) Academic rigor is the most important aspect of post high school education.

Q 25) Parent: The city education department is unable to distinguish between annoyances and important
problems. For instance, prohibiting students from having cell phones is an overreaction. If a student uses one
and thus interferes with instruction, confiscate it. All in all, we need educational leadership that can solve
problems, not create them. Which of the following is an assumption made by the parent?
a) Students having cell phones does not constitute an important problem for the city schools.
b) Students have no need for cell phones in school.
c) Faculty and staff should be allowed to possess cell phones.
d) Students need to have cell phones because some of them have no stay-at-home parent.

Q 26) The Ergonomic Society conducted a study that indicated that many people develop severe back
problems during adulthood, and that virtually all such people who received chiropractic treatment showed
great improvement. Therefore, in order to minimize the proportion of the population that suffers from back
pain, the Ergonomic Society recommended that chiropractic treatment be directed toward those adults who
suffer from severe back problems. Which of the following is an assumption on which the argument depends?
a) Any person who receives chiropractic treatment for back pain may also benefit from other forms of
treatment.
b) Large insurance carriers cover chiropractic care for back problems to a lesser degree than they do other
medical treatments.
c) Individuals who receive chiropractic or other treatment prior to developing severe back problems are not
less likely to develop back pain than those who do not.
d) Chiropractic treatment is more effective in treating severe back problems when utilized over a long period
of time, as opposed to sporadically.

Q 27) Since the new publisher took control, a news magazine‘s covers have featured only models and movie
stars. Previously, the covers had displayed only politicians, soldiers, and business leaders. A leading gossip
columnist claimed that the changes made the magazine relevant again. However, many newspaper editorials
disagreed and suggested that the new publisher is more interested in boosting sales than in reporting important
news events. Which of the following is an assumption necessary for the argument made by the gossip
columnist‘s opponents?
a) The charitable activities of models and movie stars often focus public attention on pressing problems.
b) Final authority for choosing the cover subject of the magazine lies with the publisher.
c) A magazine can boost sales while highlighting the coverage of important world leaders.
d) Some of the movie stars featured are now running for political office.

www.ibpsguide.com | www.ibpsguide.in | mock.ibpsguide.in | www.sscexamguide.com


Copyright © 2016 IBPS Guide 128
yoursmahboob.wordpress.com

Q 28) For-profit colleges serve far fewer students than either public or private non-profit colleges. At the same
time, relative to non-profit colleges, for-profit colleges draw a disproportionate share of federal and state
financial aid, such as tuition grants and guaranteed loans, for their students. It must be, then, that for-profit
colleges enroll a greater proportion of financially disadvantaged students than do non-profit colleges. The
conclusion above depends on which of the following assumptions?
a) Public non-profit colleges and private non-profit colleges enroll a similar proportion of financially
disadvantaged students.
b) For-profit colleges do not engage in fraudulent practices in helping their students obtain unneeded federal
and state financial aid.
c) The number of students receiving federal and state financial aid at for-profit colleges is greater than the
number of students receiving federal and state financial aid at non-profit colleges.
d) For-profit colleges are of similar educational quality as non-profit colleges.

Q 29) Recent research has indicated that married people are not only happier than unmarried people, but also
healthier. This study has been widely reported by the media, with most commentators concluding that being
married is good for one‘s health and attitude. The conclusion of the media commentators depends on which of
the following assumptions?
a) The longer people are married, the happier and healthier they become.
b) Married couples who had a large, extravagant wedding are happier than those who had a small, simple
ceremony.
c) Married people cannot get depressed.
d) Single people with depression or health problems are just as likely to get married as are other single
people.

Q 30) When a company refuses to allow other companies to produce patented technology, the consumer
invariably loses. The company that holds the patent can charge exorbitant prices because there is no direct
competition. When the patent expires, other companies are free to manufacture the technology and prices fall.
Companies should therefore allow other manufacturers to license patented technology. The argument above
presupposes which of the following?
a) Companies cannot find legal ways to produce technology similar to patented technology.
b) Companies have an obligation to act in the best interest of the consumer.
c) Too many patents are granted to companies that are unwilling to share them.
d) The consumer can tell the difference between patented technology and inferior imitations.

Q 31) Inorganic pesticides remain active on the surfaces of fruits and vegetables for several days after
spraying, while organic pesticides dissipate within a few hours after application, leaving the surface of the
sprayed produce free of pesticide residue. Therefore, when purchasing from a farm that uses inorganic
pesticides, one must be careful to wash the produce thoroughly before eating it to prevent the ingestion of
toxins. But one need not worry about ingesting pesticides when purchasing from farms that use only organic
pesticides. The argument above assumes that
a) Consumers are aware of the origins of the produce they purchase.
b) Produce from farms that use organic pesticides reaches the consumer within hours after it is picked or
harvested.
c) No farm uses both organic and inorganic pesticides.
d) No pesticide is capable of penetrating the skin of a fruit or vegetable.

Q 32) The downturn in the economy last year has prompted many companies to make widely publicized
layoffs, resulting in thousands of lost jobs. Economists predicted that these layoffs would cause people
generally to cut back on their discretionary spending, even if their jobs were secure, in anticipation of coming
hard times. However, this prediction has not come to pass, since there has been no increase in the amount of
www.ibpsguide.com | www.ibpsguide.in | mock.ibpsguide.in | www.sscexamguide.com
Copyright © 2016 IBPS Guide 129
yoursmahboob.wordpress.com

money set aside by the general public in savings accounts. Which one of the following is an assumption on
which the argument depends?
a) The economy has not improved in recent months.
b) There has been no increase in the amount of money invested in stocks, certificates of deposit, or other
savingsVehicles.
c) Salaries have decreased as a result of the economic downturn.
d) No business sectors have seen growth in recent months.

Q 33) Because most hospitals suffer a chronic undersupply of physicians, patients must sometimes wait hours
in the emergency room to see a doctor. Nurses should therefore perform initial examinations in hospital
emergencyrooms to determine which patients merit immediate treatment and which can wait until the
emergency physicians have more time to see them. Which of the following is an assumption on which the
argument above is based?
a) Hospitals should expand their medical staffs.
b) Physicians cannot be trained to perform initial examinations themselves.
c) Emergency rooms will run more smoothly if initial examinations are performed.
d) Nurses are competent to judge the severity of patients‘ conditions.

Q 34) A certain baseball team has just completed its season. In stadiums that seat 20,000 or fewer people, the
team averaged 1 home run per game; in stadiums that seat between 20,000 and 40,000 people, the team
averaged 2 home runs per game; and, in stadiums that seat 40,000 or more people, the team averaged 3 home
runs per game. Obviously, the excitement of playing in front of large crowds motivated the team to hit more
home runs. Assuming that all stadiums during the season were filled to capacity, which of the following, if
true, most undermines the argument above?
A) The team‘s leading home run hitter hit more home runs in mid-sized stadiums than in large stadiums
B) The fans in the larger stadiums often cheered against the team.
C) The team averaged only 2 home runs per game when playing in the league‘s largest stadium.
D) In order to create seating for the additional fans, the outfield walls in the larger stadiums were constructed
closer to home base.

Q 35) Abraham will spend $4,000 to rent a booth at the town‘s annual county fair to promote his new Burger
Shoppe. In previous years, the average food booth at the fair served 400 customers. Thus, in order to cover his
costs, Abraham has calculated that he must sell 400 burgers for $10 each. Which of the following statements
casts the most doubt on Abraham‘s chances of breaking even at the county fair?
a) Abraham should not limit himself to 400 burgers; he should be prepared to sell more.
b) Abraham has not factored in the costs of raw materials or labor in calculating the amount of revenue he
needs to generate to break even.
c) If booths in previous years averaged 400 customers, that means some booths served more than 400 and
someserved fewer than 400.
d) Many people prefer other types of food to burgers.

Q 36) The people of Prohibitionland are considering banning the service of alcoholic beverages in restaurants
to curb unruly behavior on the part of its residents. Proprietors of restaurants in Prohibitionland are protesting
the ban on the grounds that it will reduce their revenues and profits. However, several provinces in
Prohibitionland enacted restrictions on alcoholic beverages last year, and the sales taxes paid by the
restaurants in those provinces rose by an average of 50 percent. In contrast, the sales taxes paid by restaurants
located in areas of Prohibitionland that did not have any restrictions rose by an average of 30 percent. Which
of the following, if true, supports the restaurant proprietors‘ economic stance against the ban?
a) In the provinces that restricted alcoholic beverages, there was a short-term negative impact on restaurant
visitation in the beginning of last year.

www.ibpsguide.com | www.ibpsguide.in | mock.ibpsguide.in | www.sscexamguide.com


Copyright © 2016 IBPS Guide 130
yoursmahboob.wordpress.com

b) The sales tax in Prohibitionland is lower on food and beverages than it is on other consumer goods, such
asclothing.
c) The consumption of alcoholic beverages in Prohibitionland has been on a gradual decline the last 20 years.
d) The restrictions on alcoholic beverages enacted last year allowed for the service of drinks beginning
arounddinnertime each evening.

Q 37) Market Analyst: Recent research confirms that the main cause of bad breath is bacteria build-up on the
tongue. The research also concludes that tongue scrapers, when used properly, can eliminate up to 40% of the
bacteria from the tongue. As the effectiveness of tongue scrapers becomes more widely known, the market for
less effective breath freshening products, such as mints, gums, and sprays, will decline significantly. Which of
the following provides the best evidence that the analyst‘s argument is flawed?
a) Some breath freshening products are advertised to eliminate up to 30% of the bacteria from the tongue.
b) Tongue scrapers have already been on the market for a number of years.
c) Many dentists recommend regular flossing, and not the use of the tongue scraper, to combat bad breath.
d) A recent survey shows that 94% of those who regularly purchase breath freshening products are aware of
the effectiveness of the tongue scraper.

Q 38) Companies that offer ―employer sponsored insurance‖ (ESI) pay a portion of employees‘ health care
costs. In the manufacturing sector last year, companies that offered ESI had worker absentee rates 22% lower,
on average, than those at companies that did not offer ESI. If, on the basis of the evidence above, it is argued
that ESI decreases worker absenteeism, which of the following, if true, would most seriously weaken that
argument?
a) Results similar to those cited for the manufacturing sector have been found in other sectors of
the economy where ESI is offered.
b) At companies that offer ESI, employees have access to preventative health care such as regular
check-ups, routine laboratory tests, and nutrition counseling.
c) Because initiating an ESI plan requires a lot of paperwork for the company, employees, and the
insurance provider, doing so is complex and time-consuming.
d) Many firms in the manufacturing sector have improved workplace safety and decreased the
occurrence of on-thejob injuries in the last five years, and most of these companies introduced ESI at the
same time.

Q 39) A researcher studying corporate executives found that they tend to have ―take charge‖ personalities,
with the predominant traits of assertiveness, decisiveness, and self-confidence. The researcher concluded that
people who are more ―take charge‖ than the average person are more likely to become corporate executives.
Which of the following, if true, most seriously weakens the researcher‘s conclusion?
a) Holding the job of a corporate executive causes people to develop ―take charge‖ personality traits.
b) When working on charitable or community projects, corporate executives often use their ability to
make decisions and lead people to make those projects successful.
c) Some people who are not executives have stronger ―take charge‖ personalities than some people who
currently serve as corporate executives.
d) Many people who aspire to become executives exhibit different management styles in their current
jobs.

Q 40) Because of a rare type of fungus that killed off many cacao trees in Brazil, there was an unusually
meager harvest of cacao beans this year. The wholesale price of cocoa solids and cocoa butter has increased
significantly and isunlikely to fall in the foreseeable future. As a result, the retail price of chocolate is certain
to increase within six months. Which of the following, if true, most seriously weakens the argument above?
www.ibpsguide.com | www.ibpsguide.in | mock.ibpsguide.in | www.sscexamguide.com
Copyright © 2016 IBPS Guide 131
yoursmahboob.wordpress.com

a) Consumers will purchase other sweets if the price of chocolate increases.


b) Researchers have discovered an effective method to kill the fungus.
c) Dark and bittersweet varieties of chocolate will be affected more seriously than milk varieties.
d) Most chocolate in stores is manufactured from cocoa that was purchased two years earlier.

Q 41) Recently in City K, developers have stopped buying land, contractors have found themselves going
without work for longer periods, and banks have issued fewer mortgages. There must be fewer new residents
moving to City K than there were previously. Which of the following indicates a flaw in the reasoning above?
a) This year several housing blocks have gone on the market after being held up for months by legal red
tape.
b) The average size of a new home has increased significantly over the past several years.
c) Re-sales of condominiums have increased over the past six months.
d) The cost of materials such as lumber and cement has decreased over the past year.

Q 42) The anticipated retirement of tens of thousands of baby boomers will create an unprecedented
opportunity to move significant numbers of people into career-track jobs at family-supporting incomes. Major
industries, from health care and construction to automotive repair, will soon face deep shortages of workers as
a result of projected growth and boomer retirements. Fortunately, many of these jobs have relatively low
barriers to entry and could be filled by out-of-work young people. To achieve this result, the city government
should convene employers and educators to determine how best to create paths of upward mobility in these
fields. Which of the following, if true, most weakens the argument?
a) Immigration reform will limit the pool of available workers.
b) Government efforts have been shown to affect employment trends only rarely.
c) The best available positions require skills not possessed by the vast majority of the unemployed.
d) A small proportion of baby boomers will not retire as soon as is anticipated.

Q 43) Two years ago, the cost of the raw material used in a particular product doubled after an earthquake
disrupted production in the region where the material is mined. Since that time, the company that makes the
product has seen its profit margins decline steadily. Aiming to improve profit margins, the company‘s head of
engineering has decided that he must find a new source for the raw material. Which of the following, if true,
would cast the most doubt on the validity of the head of engineering‘s decision?
a) New competitors have entered the market every six months for the past two years, resulting in price
wars that have progressively driven down revenues across the market.
b) Although the earthquake occurred two years ago, the region‘s mines have still not recovered to pre-
earthquake production capacity.
c) There are several other regions in the world where the raw material is mined, but those regions do not
produce as much of the raw material as the current source region.
d) The company could use a completely different raw material to make its product.

Q 44) The current administration and Congress have once again practiced bad public policy in failing to
increase Pell grants or at least limit their reduction for next year‘s budget. Pell grants improve access to higher
education for those who have historically been disadvantaged in our society by financial or other life
circumstances, therebyhelping recipients elevate themselves to the middle class. Without that access, the gap
between the rich and poor in this country will continue to widen, increasingly straining the stability of our
democracy. Which of the following, if true, most seriously weakens the conclusion of this argument?
a) Total spending on programs targeted at improving access to higher education for disadvantaged
students will increase in next year‘s federal budget.
b) The neediest candidates for Pell grants often lack information about their eligibility for such grants.
c) Congress recently authorized a bill that will increase after-school programs in urban communities.
d) On average, an individual Pell grant funds less than 15% of the full cost of attending a four-year
college or university.
www.ibpsguide.com | www.ibpsguide.in | mock.ibpsguide.in | www.sscexamguide.com
Copyright © 2016 IBPS Guide 132
yoursmahboob.wordpress.com

Q 45) Investment Advisor: It is well-known that investing in mutual funds reduces portfolio risk through
diversification. It is also true that past investment performance is often related to future investment prospects.
Therefore, to help my clients earn high returns with low risk, I select a group of mutual funds that meet the
client‘s objectives and then invest the client‘s assets in the fund that has delivered the highest returns in this
group over the past 2 years. Which of the following statements, if true, would demonstrate a serious flaw in
the approach of the Investment Advisor?
a) Managers of many mutual funds that have delivered the highest returns over the past several years
have already used up their best investment ideas and are unlikely to sustain this level of performance in
the future.
b) Mutual funds span a wide spectrum of investment styles and performance objectives and no single
fund is suitable for every investor.
c) Many individual investors choose to manage their own portfolios rather than consult an investment
advisor.
d) The funds that have had the strongest past performance tend to continue to outperform other funds
with similar objectives for many years in the future.

Q 46) Recently, many critics of the U.S. government have pointed out that this country is the only
industrialized nationwithout a national vaccine laboratory and suggested that this lack makes theAmerican
public more vulnerablethan other advanced nations to diseases such as avian flu or other flu epidemics. When
asked at a press conference, a government official said these critics were disloyal and thus wrong about the
public‘s vulnerability. To support hisclaim, he cited the international pre-eminence of American doctors and
hospitals as well as the middle ranking ofthe United States among United Nation member nations in the health
categories of infant mortality, life span, andnutrition. He also added that all of the Europeans that he knew
preferred to undergo major medical treatments inthe United States rather than in the socialized medical
systems in place in their home countries. All of the followingare flaws in the official‘s logic EXCEPT:
a) The official accepts that the quality of physicians and hospitals is a major factor, albeit not the only one,
affectingthe public‘s vulnerability to disease.
b) The critics could be disloyal but not wrong.
c) The Europeans that the official cited are overwhelmingly wealthy males over the age of fifty.
d) The UN health ranking that the official cited is based on an almost complete survey of the nations of the
world.

Q 47)Consumer advocates argue that the coating found on non-stick cookware contains harmful chemicals
that arereleased into the air when the cookware is heated above a certain temperature. The manufacturer of the
cookwareacknowledges this hazard but assures consumers that the temperature threshold is much higher than
would everbe needed for food preparation and therefore no special precautions need be taken in using the
cookware. Whichof the following, if true, would cast the most serious doubt on the claims of the
manufacturer?
a) The chemicals released by the coating can linger in the air for days.
b) Empty cookware left on the flame often reaches exceptionally high temperatures.
c) Several consumers have already claimed illness as a result of using the cookware.
d) There are effective non-stick coatings that do not release toxins when heated.

Q 48) State X recently decided to cut state funding for the public library system in County X. To help
counteract this cutin funding, the county library system has increased library late fees from $.10 per day to
$.12 per day. Since thefee increase, library administrators have seen no decline in the number of overdue
books. The director of thecounty library system concludes that the fee hike has helped to counteract the cut in
state funding. Which of thefollowing statements, if true, most strengthens the director‘s claim?
www.ibpsguide.com | www.ibpsguide.in | mock.ibpsguide.in | www.sscexamguide.com
Copyright © 2016 IBPS Guide 133
yoursmahboob.wordpress.com

a) Since the fee increase, library administrators have noted a significant decrease in the number of books
borrowedeach day.
b) The library system incurred minor costs to make its visitors aware of the late fee increase.
c) Since the fee increase, there has been no significant change in the average number of days that books
areoverdue before they are returned.
d) The library system in County X tracks its books through a very advanced database system, allowing
libraryadministrators to have access to very accurate statistics on the number of overdue books at any
given time.

Q 49) Historically, the drug industry promoted its products to physicians by educating them in their offices or
at industry conferences. In the last 10 years, it has become much more commonplace for drug companies to
advertise prescription drugs directly to consumers, via television advertising and other media. Some public
health advocates have become concerned that patients, encouraged by advertising, may pursue the use of
prescription drugs that may be inappropriate for the individual patient or situation. However, since physicians
must prescribe these medications, there is no reason for such concern. Which of the following pieces of
information would be most helpful in addressing the concern articulated by the public health advocates?
a) Certain over-the-counter medications are as effective for many common medical conditions as more
powerful nonprescription medications.
b) Prescription medication television advertisements directed at the general public only appear on certain
programs and are not seen by many potential consumers.
c) Physicians are also subject to prescription drug advertisements that are directed toward consumers.
d) Physicians are not susceptible to pressure from patients in determining appropriate courses of
treatment, including drug prescriptions.

Q 50) Studies in restaurants show that the tips left by customers who pay their bill tend to be larger when the
bill is presented with the server‘s name hand-written on the bill. Psychologists hypothesize that simply seeing
a handwritten name makes many consumers feel more of a personal identification with the server,
encouraging larger tips. Which of the following, if true, most strongly supports the psychologists‘
interpretation of the studies?
a) The effect noted in the studies applies to patrons paying with either credit cards or cash.
b) Nametags for servers have not been shown to have any effect on the size of the bill.
c) Greeting card companies have found that charities which send holiday cards with handwritten
signatures are more likely to receive donations than those which send cards with printed signatures.
d) The studies indicated much larger average tips if the customer ordered alcoholic beverages with his or
her meal.

CRITICAL REASONING (ANSWERS WITH DETAILED SOLUTIONS):

Q 1)
A) Reducing health care spending in the United States by 50% would equalize the amount of money spent on
health care in the United States and Britain. There is an assumption made that there is some reason to do so,
perhaps in the hopes that American health will consequently improve to the level of British health, or perhaps
simply to save money. The facts given in the argument are not sufficient to support this assertion.
B) Although the recent study indicated that the middle-aged white Americans have poorer health than their
British counterparts despite apparently more expensive health care, there is no evidence that the cost of the
health care is a cause of health or sickness for either group.
C) It is possible that health care in the United States costs more because the money is being used ineffectively.
This assertion is one potential explanation for the poorer health of the Americans in the study, despite

www.ibpsguide.com | www.ibpsguide.in | mock.ibpsguide.in | www.sscexamguide.com


Copyright © 2016 IBPS Guide 134
yoursmahboob.wordpress.com

apparently higher spending on health care. However, more information is needed to conclusively make this
assertion, such as proving that money is currently being wasted, and on what. Additionally, it is unclear
exactly how much money is spent on health care for the age group and race studied, so the poorer health of the
American patients does not necessarily tell us anything about the effectiveness of the money spent on them.
D) CORRECT. The study revealed some differences in the health of middleaged white Americans and
middle-aged white Britons. The study did not indicate the reason for the difference. However, the researchers
did eliminate the lifestyle differences of diet, exercise, smoking, and drinking as causes of the health
difference. Thus, it can be conclusively asserted that something else (other than diet, exercise, smoking and
drinking) must account for the difference in health for the two groups in the study.

2)
A) While we are given information about the percentage of the total budgets spent on marketing, we have no
information about the actual amount of money either company spent on marketing.
B) While we are given information about the percentage of the total budgets spent on production, we have no
information about the actual amount of money either company spent on production.
C) Because we have no information on the sale price per copy for either company, we cannot make any
conclusions about the revenue generated by either company. It‘s very possible that Rendezvous sold its copies
at twice the price of the K-T copies, in which case the revenues for the two companies would be the same.
D) CORRECT. Since Rendezvous Hits spent 40% of its budget on production, 30% on marketing, and the
rest on overhead, we can conclude that Rendezvous Hits spent 30% of its budget on overhead. Since the K-T
spent 20% of its budget on production and 60% on marketing, and met its budget, it could not have spent more
than 20% on overhead. Therefore, Rendezvous Hits spent a higher percentage of its budget on overhead than
did the K-T.

3)
A) This statement does not have to be true. The argument provides evidence about the emotional progress of
only 30 participants. The fact that we have no information about the vast majority of participants demonstrates
that the statement in this answer choice cannot be justified.
B) While each participant did spend at least 2 hours each week attending the concerts, there is no information
in the argument that would suggest that the students reduced their study time. For example, they could have
attended the concerts in their free time.
C) This statement does not have to be true, since we have no information about the emotional progress of the
vast majority of study participants. Note that even if the study did demonstrate a positive effect of classical
music on the majority of participants, it would still be uncertain whether this effect would hold for the
majority of young adults.
D) CORRECT. We know that 20 students attended the fewest number of concerts, 10 students attended the
greatest number of concerts, and the remaining 170 students attended some other number of concerts in
between. The term 'greatest' indicates that there are at least 3 different numbers of concerts attended by the
students (as opposed to 'greater' to distinguish between 2 different numbers). Since each of the participants
attended at least one concert per week during the 12 weeks of the experiment, all of the study participants
must have attended at least 12 concerts. Even if the 20 bottom students attended the smallest possible number
of concerts (i.e. 12), it must be the case that the next 170 students in the middle attended at least one more (i.e.
at least 13 concerts) and the 10 most active participants must have attended at least one more than the middle
group, i.e at least 14 concerts. Thus, it must be true that the 10 most active participants (i.e. more than 6
participants) attended at least 14 concerts, as stated in this answer choice. Note that if the students attended
more concerts than the minimum requirement, the number of students with at least 14 concerts attended will
be even greater, still validating the accuracy of this statement.

4)

www.ibpsguide.com | www.ibpsguide.in | mock.ibpsguide.in | www.sscexamguide.com


Copyright © 2016 IBPS Guide 135
yoursmahboob.wordpress.com

A) Though poultry consumption increased at a higher rate than fish consumption, there is no way to determine
if this is due to the dietary habits of the new arrivals in Greenland. It is also possible that consumption among
longtime residents of Greenland increased at a dramatically higher rate.
B) We are given information about the relative rate of increases, not the actual amounts of poultry or fish
consumed. As a result, there is no way to know if this statement is true.
C) CORRECT. As we are given that the population of Greenland increased by 6 percent, and the total
consumption of poultry increased by 8 percent in the same period, then it must be the case that the per capita,
or average, consumption of poultry rose from 1999 to 2004. For example, let's say that the population of
Greenland increased by 6 percent from 1000 to 1060 people, while the consumption of poultry increased by 9
percent from 100 to 109 units. The per capita consumption in 1999 would have been exactly 100/1000 while
the per capita consumption in 2004 would have been 109/1060, a slightly greater value.
D) Between 1999 and 2004, both fish and poultry products were a regular part of the diet of a significant
proportion of Greenland residents.

5)
A) The passage states that every student with an IQ of 150 who applies to the VidyaMandir will be accepted
by at least one of the universities. However, it is possible that a graduate of TopNotch High with an IQ of 150
did not apply, and thereby was not accepted, to any of the schools. Hence, this conclusion is not valid.
B) The passage states that every graduate of TopNotch School has an IQ of over 120. The conclusion only
states that the student is a Top Notch school graduate and that he has an IQ of less than 120. It does NOT state
that he or she was a graduate of TopNotch school. It is possible, however, that after attending TopNotch
school for a period of time, he or she graduated from another school. If this is the case, the situation does not
contradict the passage, but contradicts the conclusion (he or she was a student at TopNotch School). Hence,
this conclusion is not valid.
C) CORRECT. Nothing in the passage precludes a person who is a graduate of TopNotch School from
having an IQ of 130 and from attending anVidyaMandir university. Neither does anything in the passage
preclude a person who has an IQ of 130 and is attending anVindyaMandir school to have graduated from
TopNotch School. Therefore, it is possible for both situations to exist simultaneously, so the conclusion is
valid.
D) At least one graduate from TopNotch school who has applied to at least one VidyaMandiruniversity has
been accepted to one of them.

6)
A) The argument does not provide enough information to conclude that wealthy individuals will not be
affected negatively by health savings accounts. The argument never specifically mentions wealthy individuals,
just people in general.
B) The argument does not provide enough information to conclude that private health insurance will no longer
be available. In fact, private health insurance is never mentioned.
C) The author argues that people will not get regular preventive examinations, and will therefore not receive
medical attention until diseases are advanced. This logic, even if true, does not allow us to conclude that most
diseases are detected during regular preventive examinations.
D) CORRECT. The argument states that "poor people, who will not be able to afford health savings accounts,
will no longer receive vaccinations". Based on this statement, it is reasonable to conclude that some people
without health savings are likely to contract infectious diseases.

7)
A) CORRECT. In general, Indira Gandhi experiences a greater number of delays per 100 flights than Shivaji
does. However, when delays caused by bad weather are discounted, Shivaji has 5 fewer delays per 100 flights.

www.ibpsguide.com | www.ibpsguide.in | mock.ibpsguide.in | www.sscexamguide.com


Copyright © 2016 IBPS Guide 136
yoursmahboob.wordpress.com

Since the two airports run the same number of flights, bad weather must cause a greater number of delays at
Indira Gandhi Airport than at Shivaji Airport.
B) While we can conclude that Indira Gandhi experiences a greater number of delays caused by bad weather,
we cannot make any conclusions about the relative weather conditions at either airport. It is very possible that
the airports experience the same weather, but that for some reason Newcomb‘s airport is betterequipped than
Indira Gandhi‘s airport to handle inclement weather (e.g. the fleet ofaircraft at Shivaji is better suited to bad
weather, or the air traffic controllers at
Shivaji are more competent, etc.). Some other factor could cause the discrepancy in weather related delays
aside from the weather conditions being different.
C) While we can conclude that Indira Gandhi experiences a greater number of delays caused by bad weather,
we have no information about delays caused by mechanical problems. There may be other reasons aside from
either bad weather or mechanical problems that account for departure delays (e.g. human error).
D) We have no information regarding the quality of airplane fleets at either airport.

8)
A) Lower production costs could explain the lower price of the flour from Isidiur‘s Mill, but there may be a
variety of other reasons. We cannot state this conclusively.
B) It is possible that the number of local flour mill jobs would be decreased, but no evidence in the passage
leads to that conclusion.
C) CORRECT. This statement properly identifies the point that, for ordering from an out-of-state mill to be
less expensive, the shipping and handling fees must be less than the difference in the flour costs of the two
suppliers. Say, for example, that a batch of flour costs $100 from Charles‘s Local Mill. The passage tells us
that the same batch would cost $90 from Isildiur‘s Mill, yet when purchasing from Isildiur‘s, shipping and
handling fees would apply. We are told that Isildiur‘s total cost is cheaper than Charles‘s, so mathematically
that is: $90 + Shipping & Handling < $100, which means that Shipping & Handling < $10 = 10% of the cost
of flour from Charles‘s.
D) If shipping and handling fees were more than 10 percent, purchasing from the out-of-state supplier would
be more expensive, not less.

9)
A) There is no way to predict box-office receipts for the year.
B) There is no way to know how many movies were released in the first half of last year.
C) We cannot infer that the price of a movie ticket has not increased.
D) CORRECT. The average revenue per film = total revenues ÷ number of films

10)
A) While this might generally be true in the real world, the given information does not address whether
doctors and nurses are too careless in conducting their jobs.
B) While the campaign did save a large number of people, we cannot say that every single person who would
have died was saved; this answer choice is too extreme.
C) The argument does not provide information to make predictions about the future; in addition, this answer
choice is extreme. Common sense tells us that we cannot prevent every single medical error in the future.
D) CORRECT. This statement can be inferred from the original argument. If the campaign saved the lives of
people who otherwise would have died of medical error, then the absence of the campaign would have meant
that many of those people might not have been saved. Notice that this answer choice is more of a restatement
of the given information, rather than what we would consider a true conclusion in the real world.

11)
A) This choice states that music education should not devote special attention to talented students. This goes
beyond the scope of the educator's statements.

www.ibpsguide.com | www.ibpsguide.in | mock.ibpsguide.in | www.sscexamguide.com


Copyright © 2016 IBPS Guide 137
yoursmahboob.wordpress.com

B) This choice states that everyone has the potential to learn music. This goes beyond the scope of the
educator's statements. The educator is arguing against classifying students as musically inept at an early age
because they might have latent talent that is not showing itself. He is not necessarily saying that everyone has
the potential to learn music.
C) CORRECT. By referring to the latent talent that some children may beneglecting, the educator is implying
that not all talent shows its face at an early age.
D) The fact that children who are directed towards other activities have learned to think of themselves as
musically inept doesn't mean that children are particularly sensitive to criticism from adults. The being
"directed towards other activities" is not necessarily best characterized criticism, and furthermore, it is not just
children that tend to think themselves incapable of something if they don't partake in that activity.

12)
A) Incorrect. While the passage mentions that charter schools themselves are freed from many regulations, no
information is presented about any difference in emphasis with respect to order and discipline between charter
and non-charter public schools. As such, it is impossible to tell whether this information would support the
conclusion in the question.
B) Incorrect. This choice presents information only about those students who score at the very highest level
of the writing assessments. However, this presents no information about the difference, on average, between
all charter school students and non-charter public school students. It is possible, for example, that while the
students who perform at the highest level on writing assessments are those whoattend charter schools, on
average non-charter public school students perform better.
C) Incorrect. There is no necessary link between the amount of time spent teaching writing and student
performance on writing assessments. For example, a good teacher who spends one hour teaching writing may
have a more positive impact on student performance than a poor teacher who spends three hours teaching
writing.
D) Correct. The passage specifies that charter schools have more freedom to pursue innovative educational
ideas than non-charter public schools. It follows that charter schools are allowed to experiment with their
curricula to a greater degree than non-charter public schools. This choice links this difference to higher student
achievement on assessments of writing ability.

13)
A) States that a heart scan is safer than an angiogram. We are given no information about the relative safety of
the procedures. So Incorrect.
B) States that patients should not be concerned about heart abnormalities that appear in a heart scan. This
misrepresents the information in the passage. The passage simply stated that the scans may pick up harmless
abnormalities, not that all abnormalities are harmless. So Incorrect.
C) States that a heart scan could result in indirect harm by causing a patient to undergo risky unnecessary
procedures. This is supported by the passage, which states that the scans could result in undue concern and
treatment. So Correct.
D) States an angiogram is the appropriate treatment for most patients. The information in the passage does not
support this. In fact, if anything, the passage would seem to support the contradictory assertion that heart scans
are more appropriate for most patients. So Incorrect.

14)
A) CORRECT. The entire controversy is based on the new law that allows motorists to be cited, even in the
absence of an additional infraction. Thus, it follows that prior to the passage of this law, an additional driving
infraction must have been necessary in order to stop and cite an individual for not wearing a seat belt.
B) Incorrect. Search and seizure laws are never mentioned in the text. This answer choice is outside the scope
of the argument.
C) Incorrect. Laws in other states are never mentioned in the text. This answer choice is outside the scope of
the argument.
www.ibpsguide.com | www.ibpsguide.in | mock.ibpsguide.in | www.sscexamguide.com
Copyright © 2016 IBPS Guide 138
yoursmahboob.wordpress.com

D) Incorrect. Though the text states that the new regulation might save countless additional lives, the
effectiveness of the previous laws are never mentioned.

15)
A) Incorrect. The argument says nothing about precipitation. This answer choice is out of scope since it
would require a number of other assumptions to make it relevant to the argument's conclusion.
B) Incorrect. Whether other trees share this feature is irrelevant; the argument focuses only on the Brazilian
ash.
C) Incorrect. The number of days of excessive heat needed to cause the tree to lose rings is irrelevant.
D) Correct. The conclusion is that the rings will be a reliable measure only if the temperature never exceeds
95 degrees. This is true only if there is no way to predict how many rings would be lost when the temperature
does exceed 95 degrees. (If it were possible to predict this, one might be able to assess the age of a tree using
its rings even if the temperature had exceeded 95 degrees.)

16)
A) Incorrect. The author never mentions future presidential elections, or the role of then president in such a
national identification system. Therefore, the conclusion that the national identification system is un-American
does not depend on this assumption.
B) Incorrect. Whether the government will soon, or will ever, start curtailing the activities of dissidents is
irrelevant to this argument: that the national identification system is un-American simply because it restricts
the liberties of U.S. citizens. Even if the government does not abuse the power the national identification
system provides, the system could still be considered un-American.
C) CORRECT. This choice connects the concept of "un-American" policies to restrictions on liberties,
essentially defining blanket restrictions on citizens as un-American.
D) Incorrect. Whether Americans are willing to give up their right to travel freely is irrelevant to this
argument: that the national identification system is un-American simply because it restricts the liberties of
U.S. citizens. Even if Americans were willing to give up their right to move about without identification, the
system could still be considered un-American.

17)
A) Incorrect. This choice is irrelevant, as it is not connected to the conclusion. The activities of celebrities
have nothing to do with the publisher‘s interests.
B) CORRECT. Since the conclusion concerns the publisher‘s desires based on the content of the magazine
covers, the editorials have to assume that the publisher decides who is to be a cover subject. If not, there is no
connection between the covers and the publisher‘s interests.
C) Incorrect. This choice is the opposite of a necessary assumption. For the editorials to conclude that the
publisher prefers profits to reporting, they have to assume that the two are mutually exclusive.
D) Incorrect. ―Some‖ means ―at least one,‖ so this is not a powerful statement in any direction. Furthermore,
even if several such stars were running for political office, it is not at all necessary to assume that to conclude
that the publisher was more interested in profits.

18)
A) Incorrect. The utility companies' claim has to do with the supply risk of the new energy sources, not with
how these sources are received by the public.
B) Incorrect. If no new supplies of traditional energy sources are found, then it is true that perhapsthese
nonrenewable supplies will continue to fluctuate in price in a risky manner. However, the argument does not
depend upon any assumption about the future discovery of oil and gas supplies.
C) CORRECT. If we assume that weather patterns are consistent and predictable, then with the stated
premises, we can conclude that solar and wind power will be less risky than oil and gas. If, on the other hand,
weather patterns are not consistent and predictable, then solar and wind power are not reliable and thus will

www.ibpsguide.com | www.ibpsguide.in | mock.ibpsguide.in | www.sscexamguide.com


Copyright © 2016 IBPS Guide 139
yoursmahboob.wordpress.com

not provide "stable energy supplies at low cost." Thus, the argument's conclusion directly depends on this
assumption.
D) Incorrect. To reach the required conclusion, it is not necessary to assume that the conversion technology
for new sources is not more expensive than the present technology.

19)
A) Incorrect. This statement does not have to be true for the claim that the media are wrong about the
economy to hold. Even if unemployment rates are useful indicators of growth and prosperity, the media could
still be wrong about the economy (e.g., if there are other indicators that show problems in other areas)
B) Incorrect. This does not have to be true for the conclusion to hold. Productivity could be a good measure
of economic growth, but the media could still be wrong about the economy (e.g., if there are other indicators
that show problems in other areas).
C) CORRECT. This has to be true for the conclusion to hold. If legislation has recently been passed that
makes it easier to obtain bankruptcy, then the evidence cited would be less relevant. The increased number of
bankruptcies could have been the result of the easier process rather than of a poor economy.
D) Incorrect. This does not have to be true for the conclusion to hold. An increase in the number of
bankruptcy lawyers would not explain the increase in the number of bankruptcy filings.

20)
A) Incorrect. Whether Company X has the money to cover its costs does not affect the amount of those costs.
B) Incorrect. We do not need to assume that the costs cannot be offset by reducing expenditures in other
areas in order for Company X's costs to be higher.
C) Incorrect. We do not need to assume that gas will never be cheaper than oil in order for Company X's
costs to be higher.
D) CORRECT. The author does not take into account the fact that only "some" of the company's plants
converted to natural gas. Some of the plants, then, still use oil, which is now cheaper. So in order to conclude
that the company will have to spend more on fuel, the author must assume that the extra cost of the natural gas
for the plants that converted is at least as much as the cost of the oil for the plants that did not.

21)
A) Incorrect. The argument states that Hansraj Oculars are lightweight, but makes no direct comparison to
traditional binoculars.
B) Incorrect. Although the advertisement is geared toward bird watchers, nothing indicates that Hansraj
Oculars should be used only by bird watchers. Even if Hansraj Oculars were used by individuals other than
bird watchers, no information in the advertisement would be undermined.
C) CORRECT. The conclusion of the advertisement is that, by using Hansraj Oculars, the reader will see
some of the world‘s rarest bird species on his or her next bird watching trip. In order for this to be true, the
reader would have to have access to these rare bird species. Even the best bird watching technology would
prove useless to view a particular species of bird if the species were not present.
D) Incorrect. This choice is true, but it is not an assumption. Remember, an assumption is an unstated piece
of information that ties a conclusion to its premises. This particular answer choice is simply a stated premise
from the second sentence of the argument.

22)
A) CORRECT. The argument depends on the assumption that the reason mold and wetness are observed
together is that wet areas cause mold growth. If the reverse causation (mold causes wetness) were true, then
keeping all plumbing in good condition to prevent leakage would do little to prevent the growth of mold. This
choice eliminates the alternate causation.
B) Incorrect. If most homeowners know enough about plumbing to determine whether theirs is in good
condition, then the recommendation made in this argument would be more useful. However, this is not an
assumption on which the argument depends.
www.ibpsguide.com | www.ibpsguide.in | mock.ibpsguide.in | www.sscexamguide.com
Copyright © 2016 IBPS Guide 140
yoursmahboob.wordpress.com

C) Incorrect. Even if mold could grow in dry areas, the fact that mold is almost always found in wet areas is
still valid. This is the fact upon which the argument is based, so the argument does not depend on the
unnecessarily absolute assertion that mold cannot grow in dry areas.
D) Incorrect. Even if some varieties of mold are harmless, the conclusion of this argument, that ―one should
make sure to keep all internal plumbing in good condition to prevent leakage‖ and minimize mold growth,
could still be valid. Therefore, this argument does not depend on the unnecessarily absolute assertion that no
varieties of mold are harmless.
23)
A) Incorrect. The argument concludes that smoking causes anxiety and nervousness. Whether these maladies
lead to more serious health problems is not relevant to the conclusion.
B) CORRECT. For smoking to be the cause of anxiety and nervousness (i.e., that A caused B) it must be true
that these individuals were not more likely to be anxious and nervous before they started smoking. If smokers
had these preconditions, which contributed to their decision to begin smoking (i.e., that B caused A), our
conclusion – that smoking causes these maladies – would be incorrect.
C) Incorrect. The argument concludes that smoking causes anxiety and nervousness. The number of survey
respondents is not relevant to the conclusion.
D) Incorrect. The argument concludes that smoking causes anxiety and nervousness. The awareness of the
health problems related to smoking is not relevant to the conclusion.

24)
A) Incorrect. It is not necessary for the reader to assume that the percentage will continue to drop. The
reader's conclusion concerns the present. Assumptions must be both unstated and necessary.
B) Incorrect. This extreme statement is not a necessary assumption. The reader does not have to assume that
all colleges should do so; the conclusion only relies on an assumption that 5.5% is too low.
C) Correct. To conclude that the low percentage of colleges granting the majority of their degrees in the
liberal arts indicates a decline in academic rigor, the reader must assume that other degree programs required
less academic rigor. If not, this evidence would not indicate a decline in academic rigor.
D) Incorrect. This is not a necessary assumption. The relative importance of academic rigor is irrelevant to
the reader‘s claim. That claim only asserts that academic rigor, in isolation, is in decline. The claim has
nothing to do with its importance relative to other attributes.

25)
A) Correct. In order to use the issue of students having cells phones as an example of how the educational
leadership creates problems where none exist, the author must assume that students having cell phones is not
an important problem.
B) Incorrect. The argument and its conclusion are not about students‘ needs. They concern the quality of
educational leadership. Thus, this choice is irrelevant; it is not necessary for the argument to assume anything
about students‘ needs.
C) Incorrect. This choice is irrelevant; the argument does not concern faculty and staff cell phones, and thus
no assumption about them is necessary. Assumptions must be both unstated and necessary to the conclusion.
D) Incorrect. The argument and its conclusion are not about students‘ needs. They concern the quality of
educational leadership. Thus, this choice is irrelevant; it is not necessary for the argument to assume anything
about students‘ needs.

26)
A) Incorrect. The argument does not depend upon whether or not chiropractic treatment can or cannot be
used in conjunction with other medical treatments. The fact that people could benefit from other forms of
treatment aside from chiropractic care weakens the argument to a slight degree, and does not act as a
supportive assumption.

www.ibpsguide.com | www.ibpsguide.in | mock.ibpsguide.in | www.sscexamguide.com


Copyright © 2016 IBPS Guide 141
yoursmahboob.wordpress.com

B) Incorrect. The relative degree of insurance coverage of chiropractic care compared with other medical
treatments is not vital to the argument. Also, that insurance carriers cover chiropractic care to a lesser degree
weakens the argument marginally, and does not act as a supportive assumption.
C) Correct. This statement rules out the possibility that chiropractic care or other medical treatments could
effectively prevent or lessen back pain among those people who have not yet developed severe back problems.
D) Incorrect. That chiropractic treatment is more effective over time is irrelevant to the argument.

27)
A) Incorrect. This choice is irrelevant, as it is not connected to the conclusion. The activities of celebrities
have nothing to do with the publisher‘s interests.
B) Correct. Since the conclusion concerns the publisher‘s desires based on the content of the magazine
covers, the editorials have to assume that the publisher decides who is to be a cover subject. If not, there is no
connection between the covers and the publisher‘s interests.
C) Incorrect. This choice is the opposite of a necessary assumption. For the editorials to conclude that the
publisher prefers profits to reporting, they have to assume that the two are mutually exclusive.
D) Incorrect. ―Some‖ means ―at least one,‖ so this is not a powerful statement in any direction. Furthermore,
even if several such stars were running for political office, it is not at all necessary to assume that to conclude
that the publisher was more interested in profits.

28)
A) Incorrect. The conclusion makes a claim about the differences between for-profit and non-profit colleges.
Differences among non-profit colleges – such as public vs. private – are irrelevant to the argument.
B) Correct. One alternative reason that might explain the disproportionate aid distribution is that for-profit
colleges engaged in fraudulent practices to obtain unneeded financial assistance for their students. If this were
true, then much of the aid was distributed based not on the actual financial situation of the students but on the
ability of colleges to defraud federal and state governments. This answer choice asserts that this was NOT in
fact the case, thereby eliminating this alternative explanation and highlighting a key assumption upon which
the argument rests.
C) Incorrect. The argument's claim is centered on proportions. The actual number of students receiving aid at
for-profit vs. non-profit colleges is irrelevant to the conclusion.
D) Incorrect. The relative educational quality of for-profit vs. non-profit colleges lies outside the scope of the
argument, which is focused solely on differences in financial aid distribution.

29)
A) Incorrect. The research compared married people to unmarried people. Neither the researchers nor the
media commentators made any distinction between newlyweds and those who had been married a long time,
so this assumption is not necessary.
B) Incorrect. The type of wedding is outside the scope of this argument. The research compared married
people to unmarried people, but made no distinction based upon the type of wedding. Thus, this assumption is
unnecessary.
C) Incorrect. At first, this statement may seem necessary—after all, if the commentators conclude that
marriage causes happiness, a lack of depression in married people would certainly support that conclusion.
However, the statement is too extreme. One depressed married person does not invalidate the research
indicating that, on average, married people are healthier and happier than non-married people.
D) Correct. This statement eliminates the alternative interpretation of the research findings—that being happy
and healthy makes a person more likely to get married.

30)
A) Incorrect. This does not address the moral obligation to the consumers (i.e. ―should‖) of the companies
who produced the patented technology, the main point of the conclusion. Furthremore, even if companies

www.ibpsguide.com | www.ibpsguide.in | mock.ibpsguide.in | www.sscexamguide.com


Copyright © 2016 IBPS Guide 142
yoursmahboob.wordpress.com

could find legal ways to produce similar technologies, the patented technology could still command exorbitant
prices, thereby harming the consumer.
B) Correct. The conclusion only makes sense if companies have an obligation to act in the best interest of the
customer, as this choice states.
C) Incorrect. This generally follows along with the author's claim, but we are not required to assume this in
order to reach the conclusion that companies who are granted patents are obligated to look out for the best
interests of their customers.
D) Incorrect. This addresses a tangential issue of whether or not consumers could notice the difference
between a new patented technology and a possible imitation. This does not address the core issue of the
obligation to the consumer.

31)
A) Incorrect. This is unrelated to the argument since the conclusion speaks about nothaving to worry about
ingesting produce on which only organic pesticides were known to be used.
B) Incorrect. If anything, this statement runs counter to what the argument is saying. If produce that has been
sprayed with organic pesticide reaches the final consumer within hours after it is picked, it is possible that the
consumer does need be concerned about the pesticides.
C) Incorrect. The conclusion of the argument is already limited to those farms which use "only organic
pesticides.‖
D) Correct. If a pesticide is capable of penetrating the skin of a fruit or vegetable then, while the organic
pesticide will dissipate from the surface of the fruit in a few hours, it may remain inside the fruit. The author
of this argument assumes that the pesticides cannot penetrate the skin.
32)
A) Incorrect. The state of the economy in the last few months has no bearing on the claim that a lack of extra
money deposited in consumer savings accounts is evidence that consumer spending has not decreased.
B) Correct. If there was an alternate explanation for the lack of increase in savings accounts, the claim that
the decrease in spending has not taken place would be greatly weakened. This assumption guarantees us that
at least one other possible explanation for the lack of increase in savings (i.e. an increase in stocks, certificates
of deposit, or other savings vehicles) is NOT true.
C) Incorrect. Assuming that salaries decreased would actually weaken this argument. If salaries decreased,
that may explain the decrease in the amount of money being put aside in savings without necessarily implying
a decrease in spending. People could be making less, spending the same, and therefore saving less.
D) Incorrect. The lack of growth in business sectors has nothing to do with what consumers are doing with
their money during the same time period.

33)
A) Incorrect. The idea of having nurses make the initial examination does not depend on increasing the
medical staff.
B) Incorrect. The main premise for the conclusion was that patients ended up waiting due to an undersupply
of doctors. There weren't enough doctors to perform the initial examination. If the doctors perform the initial
examinations there will be no time saved.
C) Incorrect. The conclusions rests on whether or not the nurses would be able to perform the examinations,
not on what the result of them doing the examinations would be.
D) Incorrect. This argument is valid only if we assume that nurses are competent to determine which patients
merit immediate treatment.

34)
A) Incorrect. The argument makes a claim about the collective behavior of the team. This collective claim
does not preclude certain individuals from hitting fewer home runs in larger stadiums.
B) Incorrect. The claim made in the argument is based on the size of the crowd in each stadium. For whom
the fans cheered is irrelevant to the argument.
www.ibpsguide.com | www.ibpsguide.in | mock.ibpsguide.in | www.sscexamguide.com
Copyright © 2016 IBPS Guide 143
yoursmahboob.wordpress.com

C) Incorrect. Similar to answer choice A, this choice cites one specific example of contradictory information,
while the argument is based on the average behavior of the team throughout the entire season. The does not
strongly undermine that, on average, the team was motivated by larger crowds.
D) Correct. This choice explains that the larger stadiums actually have different dimensions from the smaller
stadiums. In order to accommodate a larger number of fans, the outfield walls are closer to the batters. Thus, it
is very possible that the greater number of home runs is due to the fact that the ball does not have to travel as
far in larger stadiums.

35)
A) Incorrect. Though this is certainly true in a practical sense, it does not show that Abraham‘s plan will not
work and that he will not stay within his budget. In fact, if he could sell more than 400 burgers, it is likely that
this would help him break even.
B) Correct. Each sandwich costs some amount of money to make and sell, so even if Abraham sells the 400
burgers as planned, his net income will certainly be less than $4,000. Based on this statement, Abraham
definitely will not break even.
C) Incorrect. Though it is true that an average number of customers means some booths served more and
some served fewer, this statement does not tell us that Abraham will have fewer than 400 or that his plan
cannot work. Consider also that each customer might buy more than one burger, so Abraham could have fewer
customers but still sell enough burgers to cover his costs.
D) Incorrect. Although some people will buy other types of food at the fair, this statement fails to
conclusively weaken Abraham‘s logic. Abraham relied on information about the average number of customers
for food booths in previous years, but those booths may have sold burgers or other foods – we simply don‘t
have enough information to judge whether this statement strengthens or weakens Abraham‘s conclusion.

36)
A) Incorrect. This answer choice may seem to strengthen the argument that banning the service of alcoholic
beverages would have an adverse impact on restaurants. However, as the evidence involves data for the entire
year, citing a short-term negative impact on restaurant visitation at the beginning of the year does not
measurably strengthen the argument.
B) Incorrect. The relative tax rate on food and beverages as compared to other consumer good is irrelevant
here.
C) Incorrect. A gradual decline in alcohol consumption over the past 20 years would suggest that over time,
any ban on alcohol would have an increasingly small impact on restaurant visitation, weakening the
proprietors‘ argument.
D) Correct. This statement calls the evidence into question by indicating that any measured increase in sales
taxes and, presumably, revenues for restaurants that have been operating under the restrictions last year
enacted is irrelevant, as the restrictions could be argued to be completely different than the total ban that is
being proposed. This answer choice substantially strengthens the proprietors‘ argument by threatening to
make the cited evidence irrelevant.

37)
A) Incorrect. This statement does not attack either one of the assumptions. In fact, it may actually strengthen
the argument by pointing out that some breath freshening products are less effective than tongue scrapers at
eliminating bacteria.
B) Incorrect. This statement could weaken the argument by attacking the second assumption (consumers are
currently unaware of the tongue scraper). If the tongue scraper has been on the market for a while, maybe
consumers are already aware of its effectiveness. However, just because the product is on the market doesn't
mean consumers are aware of its effectiveness. The analyst‘s conclusion states that as consumers learn about
tongue scrapers, the market for breath freshening products will decline, regardless of whether the tongue
scraper has already been on the market.

www.ibpsguide.com | www.ibpsguide.in | mock.ibpsguide.in | www.sscexamguide.com


Copyright © 2016 IBPS Guide 144
yoursmahboob.wordpress.com

C) Incorrect. While this certainly doesn‘t help the market analyst‘s case, we don‘t know what percentage of
dentists recommend flossing over the tongue scraper, and we can‘t be sure how this recommendation affects
the consumption of breath freshening products.
D) Correct. This statement weakens the argument by attacking one of the main assumptions of the argument:
people who use breath freshening products don‘t already know about the effectiveness of the tongue scraper.
If 94% of those who consume breath freshening products already know about the tongue scraper, and if these
consumers have continued to purchase breath freshening products, then only 6% of those who consume breath
freshening products could decide to stop purchasing these products upon learning about the tongue scrapers
effectiveness in fighting bad breath. Even if all 6% stopped purchasing the products, this would hardly create a
―significant decline‖ in the market.

38)
A) Incorrect. Similar findings in other sectors of the economy strengthen rather than weaken the argument.
B) Incorrect. If workers have access to preventative health care as a result of the ESI plan, they might be
healthier and would miss fewer days of work due to illness. This point supports the argument.
C) Incorrect. The difficulty of initiating an ESI plan is irrelevant to a conclusion about what happens after the
plan is established.
D) Correct. If there are fewer on-the-job injuries, then workers will miss fewer days of work. This is an
alternate explanation for the decrease in absenteeism, and thus weakens the argument that the decrease in
absenteeism must have been due to the availability of ESI.

39)
A) Correct. If a ―take charge‖ personality is observed in people who are corporate executives because the job
itself causes those traits to emerge, then the researcher‘s conclusion that the personality traits lead to the job
would be weakened.
B) Incorrect. The behavior of corporate executives outside of the corporate world is not relevant to the
argument.
C) Incorrect. The fact that some non-executives have stronger "take charge" personalities than some people
who are corporate executives does not substantially weaken the argument, as the existence of people who
embody the extremes of a "take charge" personality does not disprove that those with a "take charge"
personality are more likely to move into the role of corporate executive. Moreover, the personalities of non-
executives are not relevant as counterexamples to the researcher‘s argument about executives.
D) Incorrect. The argument concerns ―take charge‖ personality traits, not other management styles. It is
possible that the people who aspire to become executives have ―take charge‖ personalities, but have to adjust
their management style according to the demands of their current jobs. Thus, this statement does not weaken
the researcher‘s conclusion.

40)
A) Incorrect. The argument concludes that the retail price of chocolate is certain to increase - what consumers
will or will not do in response to the predicted price increase is not relevant.
B) Incorrect. If researchers have discovered a method to kill the fungus, then the cacao bean crop might
recover, although this statement does not provide any indication that that will happen. This statement does not
mention anything about when the fungicide method would be implemented, or how long the supposed cacao
bean crop recovery would take. Therefore, this statement fails to weaken the conclusion that the retail price
will increase within six months.
C) Incorrect. The argument does not discuss the differences between types of chocolates, so the statement
that dark and bittersweet varieties will be more seriously affected than milk varieties is irrelevant.
D) Correct. This statement contradicts the assumption that the retail price of chocolate immediately reflects
the wholesale price of cocoa solids and cocoa butter. If the chocolate currently sold in stores was made from

www.ibpsguide.com | www.ibpsguide.in | mock.ibpsguide.in | www.sscexamguide.com


Copyright © 2016 IBPS Guide 145
yoursmahboob.wordpress.com

cocoa purchased two years ago, then it may be quite some time before the current wholesale price increase is
passed on to retail chocolate consumers.

41)
A) Correct. This suggests that there might be another reason for the decline in home construction: the supply
of available housing has been increased through the release of many previously built homes. Therefore, the
reasoning in the argument is flawed.
B) Incorrect. The size of homes, by itself, does not point to any flaw in the argument.
C) Incorrect. The argument centers on new homes, so re-sales of condominiums are not directly related.
D) Incorrect. If materials cost less, it seems more likely that any decrease in new home construction could be
attributed to the stated causes.

42)
A) Incorrect. If anything, this choice strengthens the argument. If immigration does not provide a labor pool,
it is more likely that a shortage will ensue.
B) Correct. The argument assumes that it is feasible to affect employment patterns by government
encouragement and/or action. If that assumption is denied, the conclusion is weakened, as the conference
would be pointless.
C) Incorrect. This choice makes an irrelevant distinction. It doesn‘t matter if the best positions require skills,
as long as the majority are available to the unskilled unemployed in question.
D) Incorrect. Knowing that a small proportion of baby boomers will not retire on schedule does not
significantly weaken the argument. The argument relies on general estimates, not on exact numbers.

43)
A) Correct. New competitors have caused a steady two-year decline in revenues. This weakens the engineer's
contention that the one-time doubling of costs for the raw material is the cause of the steady two-year decline
in profit margins .
B) Incorrect. The fact that the region's mines are producing less than they did before the earthquake does not
indicate anything about the cost of the raw material; it would be necessary to show that cost is not reduced in
order to weaken the engineer's conclusion.
C) Incorrect. The amount of raw material produced by other regions does not indicate anything about the cost
of the raw material; it would be necessary to show that cost is not reduced in order to weaken the engineer's
conclusion.
D) Incorrect. The use of a different raw material does not indicate anything about the cost of that raw
material; it would be necessary to show that cost is not reduced in order to weaken the engineer's conclusion.

44)
A) Correct. If total spending on access to higher education will increase, then the federal government has
addressed the issue that the author cites, albeit through means other than Pell grants.
B) Incorrect. Whether candidates for Pell grants are aware of their eligibility is irrelevant to the claim that the
government has practiced bad public policy.
C) Incorrect. This choice may sound like a counterargument (that Congress is somehow practicing good
public policy by authorizing a bill that will increase after-school programs in urban communities) to the
argument presented (that the government is practicing bad public policy by failing to safeguard Pell grants).
However, we have no evidence that after-school programs in urban communities help low income students
afford higher education, so this does not weaken the argument presented by the author.
D) Incorrect. The dollar amount of the Pell grants is irrelevant. To this argument, it matters only that they
provide some help at all.

45)

www.ibpsguide.com | www.ibpsguide.in | mock.ibpsguide.in | www.sscexamguide.com


Copyright © 2016 IBPS Guide 146
yoursmahboob.wordpress.com

A) Correct. This answer choice demonstrates a serious flaw in the logic of the investment advisor. If it is true
that the best-performing fund managers have already used their strongest ideas and are unlikely to sustain this
level of performance in the future, then the advisor‘s winner-oriented strategy is unlikely to deliver high
returns.
B) Incorrect. Since the investment advisor selects the fund from the group that meets the client‘s objectives,
this statement does not weaken the advisor‘s strategy.
C) Incorrect. Since the advisor‘s strategy is oriented only towards her clients rather than the public in general,
the fact that many investors choose to manage their own portfolios is outside the scope of the argument
D) Incorrect. This statement, if true, would support rather than weaken the advisor‘s strategy. If the funds
with strongest past performance continue to outperform others, the advisor‘s strategy is likely to yield high
future returns.

46)
A) Correct. This choice states that the official accepts that the quality of physicians and hospitals is a major
factor, albeit not the only one, affecting the public's vulnerability to disease. This does not contradict anything
in the official's argument, nor does it make light of any flaws in the official's logic.
B) Incorrect. This choice highlights the official's logical jump from "disloyal" to "wrong" in the phrase "these
critics are disloyal and thus wrong about the public's vulnerability." There is no connection between disloyalty
and wrongness. This is a flaw.
C) Incorrect. If the Europeans that the official cited are overwhelmingly wealthy men over the age of fifty,
the official relied on an unrepresentative sample to justify his claim. What is true of wealthy older European
men is not necessarily true of Europeans or non- Americans generally. This is a flaw.
D) Incorrect. The fact the official relies on health statistics that are based on a complete accounting of deaths,
injuries, and illnesses suffered by the American public from all causes, including the ten percent attributable to
infectious diseases means that the official is not limiting his evidence to cases relating to vaccines. Other
causes of death are not relevant to the argument. This is a flaw.

47)
A) Incorrect. The fact that chemicals can linger for days does not affect the claim; if the chemicals are not
released in the first place, this is irrelevant.
B) Correct. If "empty cookware left on the flame often reaches exceptionally high temperatures," then there
may indeed exist circumstances under which the cookware will pose a danger. The manufacturer's claim that
no precautions need be taken is greatly weakened.
C) Incorrect. The fact that several consumers have claimed illness as a result of using the cookware does not
mean that their illnesses were in fact from the cookware; the food may have been contaminated or the illness
may have resulted from something entirely unrelated to cooking. Without proof of the claim, this choice is not
relevant.
D) Incorrect. The existence of other non-stick coatings that do not release toxins has little to do with the
manufacturer's claim here about a non-stick coating that could release toxins.

48)
A) Incorrect. A decrease in the number of borrowed books has no bearing on the revenue generated from late
fees if the number of overdue books remains unchanged. The question explicitly states that the number of
overdue books has not changed.
B) Incorrect. If anything, this statement weakens the argument. The costs incurred to implement the new fees
would cut into the revenue generated from these new fees.
C) Correct. This statement rules out the possibility that the library system is losing revenue as a result of
borrowers returning overdue books earlier than they would otherwise.
D) Incorrect. The argument has explicitly stated that the number of overdue books has not changed. This is a
stated premise that we must take as factual information, regardless of the quality of the database being used to
track such information.
www.ibpsguide.com | www.ibpsguide.in | mock.ibpsguide.in | www.sscexamguide.com
Copyright © 2016 IBPS Guide 147
yoursmahboob.wordpress.com

49)
A) Incorrect. The clinical efficacy of certain over-the-counter medications does not address the public health
advocates‘ concern regarding patient pursuit of inappropriate prescription drugs.
B) Incorrect. The public health advocates‘ concern does not rely on every possible consumer to see the
advertisements for prescription medications. It is enough that some consumers see the advertisements.
C) Incorrect. This answer choice states the obvious possibility that physicians may also see the
advertisements for prescription drugs directed toward consumers. This is irrelevant to the concern expressed
by the public health advocates.
D) Correct. This answer choice directly addresses the public health advocates‘ concern by establishing that
physicians are not susceptible to patient pressure in prescribing inappropriate drugs. As a result, drugs will be
prescribedaccording to the objective clinical judgment of the prescribing physician, mitigating the danger of
inappropriate use.

50)
A) Incorrect. The fact that the effect applies equally regardless of the method of payment is not relevant to
the conclusion.
B) Incorrect. The argument does not address the size of the bill; rather, it addresses the size of the tip. This
choice is irrelevant.
C) Correct. This answer choice provides further evidence that a handwritten name or signature generates a
greater form of personalization and emotional connection among recipients, leading to more donations.
D) Incorrect. The impact of alcoholic beverages on tipping behavior is irrelevant.

www.ibpsguide.com | www.ibpsguide.in | mock.ibpsguide.in | www.sscexamguide.com


Copyright © 2016 IBPS Guide 148

You might also like